P формула в физике: Из формулы p=A\t выразите A,t

Как легко выучить формулы по физике для ЕГЭ 2023 в 11 классе — 4 способа запоминания физических формул из кодификатора

В кодификаторе ЕГЭ по физике около 180 формул. Сначала создаётся ощущение, что в течение года можно разбить формулы по блокам и изучать их в спокойном темпе. Однако это не очень хорошая идея: подготовка к ЕГЭ должна быть системной, зубрёжка не поможет. В этой статье преподавательница физики в Вебиуме Снежа Планк рассказывает, как правильно учить формулы по физике.

Снежа Планк, преподавательница физики в Вебиуме

Некоторые формулы мы выводим из задач и со временем понимаем, откуда появлялись те или иные величины. Но есть базовые формулы, которые невозможно вывести в рамках школьной программы по физике. Поэтому приходится их запоминать.

Метод 1: придумывайте ассоциации

Например, формула силы Ампера F = B * I * L * sina = Ампер бил сына.

На курсе в Вебиуме мы с ребятами запоминали формулу напряженности и разницу потенциалов U = E * d словом «еда». А при нахождении гидростатического давления нужно обязательно кому-то родить

p = p*g*h — «ро жэ аш», рожаешь.

Если вы умеете рисовать, то можете изображать свои ассоциации. Написали формулу, придумали ассоциацию и нарисовали её. 

Формулу циклической частоты мы запоминали со студентами как «две птицы», поскольку «омега» похожа на птичку. И такую иллюстрацию сможет нарисовать каждый.

Как не потерять баллы на лёгких заданиях и избежать ошибок в ЕГЭ по физике, рассказали в этой статье.

Метод 2: сделайте карточки с формулами 

Сделайте карточки и развесьте их по всей квартире. Например, идёте ночью к холодильнику и повторяете кпд тепловой машины. Ставите разогревать еду в микроволновке — а там у вас формула электромагнитной волны. 

Может ли девушка получить специальность инженера и получить работу в строительной компании? Снежа рассказала о своём профессиональном пути.

Метод 3: используйте мнемонические правила

Чтобы запомнить, в каком порядке идёт увеличение частоты волн видимого света, выучите фразу «Каждый охотник желает знать, где сидит фазан!». Самая высокая частота у фиолетового цвета, а у самая низкая — у красного. 

Сделали подборку сборников и пособий для подготовки к ЕГЭ по физике.

Метод 4: используйте метод Фейнмана

Представьте, что вы объясняете тему восьмилетнему ребёнку. Вы не можете использовать терминологию, иначе он вас не поймёт. Придётся описывать формулу своими словами: максимально просто и доступно. 

Например, опишем этим методом формулу мощности. Представьте, вы поехали отдыхать на дачу к бабушке. Но приехали в самый неудачный момент — пора выкапывать картошку. Утром вы начинаете усердно копать картошку, то есть, заниматься какой-то работой. Вы копали в течение двух часов. В итоге, бабушка сказала, что вы «супер мощная машина по выкапыванию картошки».

Для формулы мощности необходимо понять, какую работу вы выполняете в единицу времени. 

Если вы только начинаете путь подготовки к ЕГЭ по физике, то первым делом изучите демоверсию. А наш подробный гайд по экзамену поможет вам стать высокобалльником в этом году.

Если вы нашли ошибку, пожалуйста, выделите фрагмент текста и нажмите Ctrl+Enter. Мы обязательно поправим!

определения, правила, таблица с пояснениями, примеры

Содержание:

  • Физика для 7-9 класса — какие основные темы изучаются
  • Все формулы по физике за 7 класс
    • Формула для нахождения скорости равномерного движения
    • Формула средней скорость неравномерного движения
    • Формула плотности вещества
    • Формула силы тяжести
    • Формула для нахождения равнодействующих сил, которые направлены в единую сторону
    • Формула для веса тела
    • Формула для вычисления давления
    • Формула для измерения давления жидкости
    • Формула силы Архимеда
  • Формулы по физике за 8 класс
    • Формула для вычисления количества теплоты при охлаждении (нагревании)
    • Формула количества теплоты при сгорании топлива
    • Формула количество теплоты плавления (кристаллизации)
    • Формула КПД теплового двигателя
    • Формула силы тока
    • Формула электрического напряжения
    • Формула закона Ома для участка цепи
    • Формула последовательного соединения проводников
    • Формула параллельного соединения проводников
    • Формула мощности электрического тока
    • Формула для закона преломления света
  • Формулы по физике за 9 класс
    • Формула проекции вектора перемещения
    • Формула скорости равномерного движения
    • Формула движения тела по окружности
    • Закон всемирного притяжения
    • Формула импульса тела
    • Формула связи между периодом и частотой колебаний
    • Формула скорости волны
    • Формула электрической емкости конденсатора
    • Формула энергии связи (или формула Эйнштейна)

Содержание

  • Физика для 7-9 класса — какие основные темы изучаются
  • Все формулы по физике за 7 класс
    • Формула для нахождения скорости равномерного движения
    • Формула средней скорость неравномерного движения
    • Формула плотности вещества
    • Формула силы тяжести
    • Формула для нахождения равнодействующих сил, которые направлены в единую сторону
    • Формула для веса тела
    • Формула для вычисления давления
    • Формула для измерения давления жидкости
    • Формула силы Архимеда
  • Формулы по физике за 8 класс
    • Формула для вычисления количества теплоты при охлаждении (нагревании)
    • Формула количества теплоты при сгорании топлива
    • Формула количество теплоты плавления (кристаллизации)
    • Формула КПД теплового двигателя
    • Формула силы тока
    • Формула электрического напряжения
    • Формула закона Ома для участка цепи
    • Формула последовательного соединения проводников
    • Формула параллельного соединения проводников
    • Формула мощности электрического тока
    • Формула для закона преломления света
  • Формулы по физике за 9 класс
    • Формула проекции вектора перемещения
    • Формула скорости равномерного движения
    • Формула движения тела по окружности
    • Закон всемирного притяжения
    • Формула импульса тела
    • Формула связи между периодом и частотой колебаний
    • Формула скорости волны
    • Формула электрической емкости конденсатора
    • Формула энергии связи (или формула Эйнштейна)

Физика для 7-9 класса — какие основные темы изучаются

Физика — одна из естественных наук, изучающая общие и фундаментальные закономерности структуры, развития и эволюции мира вокруг нас. Изучением физики занимались многие известные ученые. Имена самых известных физиков: Альберт Эйнштейн, Исаак Ньютон, Ж.И. Алферов, Л.Д. Ландау, Никола Тесла и многие другие.

Так выглядел Никола Тесла (открытия в области электричества):

Источник: ru.wikipedia.org

Практически все физические задачи, явления рассчитывают при помощи формул. Рассмотрим самые основные формулы по физике за 7-9 классы.

Осторожно! Если преподаватель обнаружит плагиат в работе, не избежать крупных проблем (вплоть до отчисления). Если нет возможности написать самому, закажите тут.

Все формулы по физике за 7 класс

В 7 классе ученики школы знакомятся с формулами для вычисления следующих физических величин: скорость равномерного движения; плотность вещества; средняя скорость неравномерного движения; сила тяжести; равнодействующая сила, которая направлена в одну сторону; вес тела; давление; давление жидкости; сила Архимеда.   

Формула для нахождения скорости равномерного движения

Скорость равномерного прямолинейного движения — неизменяемая скорость объекта в процессе движения по прямой линии. Такая скорость будет одинаковой в каждый момент движения тела.

Рассчитать скорость равномерного движения можно так:

Формула 1

\(V=\frac{S}{t}\)

В данной формуле V является скоростью тела, которую ищем, показатель S является путем, который проходит объект, а t является временем, за которое путь был пройден.  

Скорость обычно измеряют в километрах в час (обозначают как км/ч) в случае больших расстояний, а в случае маленьких — в метрах в секунду (обозначают как м/с).

Формула средней скорость неравномерного движения

Средняя скорость — скорость, которой мог бы обладать объект, если бы смог преодолеть тот же путь за это же время при условии равномерного движения.

Средняя скорость зависит от параметров, которые применяются к скорость равномерного движения: t и S. Для расчета средней скорости движения необходимо использовать следующую формулу:

Формула 2

\(V=\frac{S_{1}+S_{2}}{t_{1}+t_{2}} S_{1}\)

В данной формуле V будет искомой средней скоростью, \(S_{1}\), \(S_{2}\) — участники пути, из которых складывается полноценный путь тела, \(t_{1}\) — время, которое тело потратило на преодоление первого участка пути, а \(t_{2}\) — время, которое тело потратило на преодоление второго участка пути. Среднюю скорость изменяют в километрах в час.

Формула плотности вещества

Плотность вещества — физическое значение, которое указывает на зависимость массы вещества от объема вещества. Формула для вычисления плотности вещества будет следующей:

\(p=\frac{m}{V}\)

В данной формуле p будет плотностью, m будет массой вещества, а V является объемом вещества. Единицами измерения плотности является килограмм на кубический метр.

Формула силы тяжести

Сила тяжести — такая сила, при помощи которой все тела на планете Земля притягиваются к поверхности.

Можно вычислить по формуле:

\(F=g\times{m}\)

Здесь F будет силой тяжести, m — массой тела, g будет являться коэффициентом силы тяжести (он равен 9,8 метров в секунду). Единицей измерения силы тяжести является ньютон.

Формула для нахождения равнодействующих сил, которые направлены в единую сторону

Равнодействующая сила — векторная сумма сил, которые действуют на объект или частицу.

Если силы, которые воздействуют на объект, направляются по единой прямой в одну сторону, равнодействующая данных сил будет направлена в такую же сторону, модуль этой силы будет равняться сумме модулей данных сил.

Равнодействующую силу можно вычислить по формуле:

\(R=F_{1}+F_{2}\)

В данной формуле R является равнодействующей сил \(F_{1}\) и \(F_{2}\), которые воздействуют на объект. Единица измерения — Ньютоны.

Формула для веса тела

Вес тела — сила, с которой объект влияет на подвес или опору под ним из-за притяжения к поверхности Земли.

Вес тела вычисляют по такой формуле:

\(F=g\times{m}.\)

Единицы измерения веса тела — Ньютоны.

Формула для вычисления давления

Давление — физическая величина, которая характеризует величину воздействия силы, которая действует перпендикулярно поверхности на площадь данной поверхности.

Формула давления:

\(P=\frac{F}{S}\)

В данной формуле P является давлением, F — силой, которая направлена перпендикулярно площади поверхности, S является площадью поверхности, на которую влияет сила. Единицей измерения давления являются паскали.

Формула для измерения давления жидкости

Обычно величина давления в газе или жидкости напрямую зависит от двух факторов:

  1. От уровня газа или жидкости в сосуде.
  2. От уровня плотности газа или жидкости. Чем больше показатель плотности, тем больше будет показатель давления.

Формула для вычисления давления жидкости выглядит так:

Формула 3

\(P=p\times{g}\times{h}\)

В данной формуле P является давлением в жидкости, p является плотностью жидкости, g является коэффициентом силы тяжести, который равен 9,8 метров в секунду, h является высотой столба жидкости в емкости. Единицей измерения давления в жидкости является паскаль.

Примечание 1

По закону Паскаля давление в газах и жидкости передает одинаково по всем направлениям.

Формула силы Архимеда

Сила Архимеда — сила выталкивания, которая воздействует на объект, который погружают в газ или жидкость. Данная сила постоянно направлена вверх, равна весу жидкости по модулю.

Формула силы Архимеда выглядит так:

\(F_{a}=p\times{g}\times{V}\)

В данной формуле \(F_{a}\) является силой Архимеда, p является плотностью газа или жидкости, g является коэффициентом силы тяжести, а V является объемом объекта, который погружен в жидкость. Единицей измерения силы Архимеда является ньютон.

Формулы по физике за 8 класс

В 8 классе ученики знакомятся с формулами, с помощью которых можно вычислить следующие величины: количество теплоты в случае нагревания (или охлаждения), количество теплоты в случае сгорания топлива, количество теплоты в случае плавления (кристаллизации), коэффициент полезного действия (КПД) теплового двигателя, сила тока, электрическое напряжение, закон Ома для определенного участка цепи, последовательное соединение проводников, параллельное соединение проводников, мощность электрического тока, а также закон преломления света.

Формула для вычисления количества теплоты при охлаждении (нагревании)

Количество теплоты — физическая величина, которая характеризуется количественным значением энергии, которую объект получает в случае нагревания и отдает в случае охлаждения.

Формула количества теплоты:

\(Q=c\times{m}\times{\triangle{t}}\)

В данной формуле Q является количеством теплоты, m — массой тела объекта, c — показатель удельной теплоемкости вещества, из которого состоит объект, \(\triangle{t}\) является показателем изменения температуры тела объекта. Единицей измерения являются джоули.

Примечание 2

Если Q>0, тогда происходит процесс нагревания, а если Q<0, тогда тело остывает.

Формула количества теплоты при сгорании топлива

Количество теплоты при сгорании топлива — физическая величина, равная количеству энергии, которая выделяется в случае полного сгорания топлива.

\(Q=q\times{m}\)

В данной формуле Q является количеством теплоты в процессе сгорания топлива, q является удельной теплотой сгорания топлива (то есть количество энергии, которое выделяется в случае сгорания килограмма топлива), m является массой топлива. Единицей измерения являются джоули.

Формула количество теплоты плавления (кристаллизации)

Количество теплоты плавления или кристаллизации — количества теплоты, которое нужно для плавления объекта, который находится в условиях температуры плавления и нормальном атмосферном давлении.

Формула вычисления количества теплоты плавления такова:

\(Q=\lambda\times{m}\)

Формула вычисления количества теплоты кристаллизации такова:

\(Q=-\lambda\times{m}\)

В данных формулах Q будет количество теплоты плавления\кристаллизации, m будет массой тела, \(\lambda\) будет удельной теплотой плавления (то есть такой величиной теплоты, которая нужна для плавления килограмма вещества). Единицей измерения этой формулы является джоуль.

Формула КПД теплового двигателя

КПД (или же коэффициент полезного действия) теплового двигателя — знаковый показатель, который зависит от работы, которая совершается двигателем за единый цикл, а также количества теплоты, которое получает объект от нагревателя.

Формула КПД такова:

\(\eta=\frac{A}{Q_{1}}\times100%\]\)

В данной формуле \(\eta\) является КПД, A является полезной работой, \(Q_{1}\) является количеством теплоты, которое тело получило от нагревателя.

Существует и другой тип формулы:

Формула 4

\(\eta=\frac{Q_{1}-Q_{2}}{Q_{1}}\times100%\)

В данной формуле \(Q_{1}\) является количеством теплоты, которое получает тело от нагревателя, \(Q_{2}\) является количеством теплоты, которое отдается холодильнику. КПД измеряют обычно в процентах.

Формула силы тока

 

Сила тока — физическая величина, характеризующая заряд, который проходит через проводник за единицу времени.

Формула силы тока такова:

\(I=\frac{q}{\triangle{t}}\)

В данной формуле I является силой тока в проводнике, q является электрическим зарядом, который проходит через поперечное сечение проводника, а \(\triangle{t}\) является временем прохождения заряда. Единицей измерения силы тока является ампер.

Формула электрического напряжения

Электрическое напряжение — физическая величина, которая характеризует действие электрического поля на заряженные частицы.

Формула для нахождения электрического напряжения:

\(U=\frac{A}{q}\)

В данной формуле U является напряжением на участке цепи, A является работой электрического поля, q является величиной заряда на участке цепи. Единицей измерения являются вольты.

Формула закона Ома для участка цепи

Теорема

Закон Ома сформулирован так: сила тока на конкретном участке электрической цепи прямо пропорционально напряжению на данном участке, а также обратно пропорциональна сопротивлению данного участка электрической цепи.

Формула закона Ома такова:

\(I=\frac{U}{R}\)

В данной формуле I является сила тока на конкретном участке цепи, U является напряжением на данной участке электрической цепи, R является сопротивлением конкретного участка электрической цепи. Единицей измерения является ампер.

Формула последовательного соединения проводников

Последовательное соединение проводников в электрической цепи — соединение элементов, при котором конец одного элемента соединяется с началом другого.

Для последовательного соединения характерны закономерности, при помощи которых можно вычислить главные параметры электрической цепи, такие как сила тока, сопротивление и напряжения.

формула 5

Теорема

\(I=I_{1}=I_{2}\)

\(U=U_{1}+U_{2}\)

\(R=R_{1}+R_{2}\)

В данных формулах \(I_{1},U_{1},R_{1}\) является электрическими характеристиками первого участка цепи, а \(I_{2},U_{2},R_{2}\) является электрическими характеристиками второго участка цепи. Единицами измерения силы тока является ампер, напряжения — вольт, сопротивления — ом.

Формула параллельного соединения проводников

Параллельное соединение — вид соединения проводников, при котором начала проводников присоединяются к единой точке цепи, концы — к иной точке цепи.

В случае параллельного соединения основными характеристики электрической цепи вычисляются по таким формулам:

\(I=I_{1}+I_{2}\)

\(U=U_{1}=U_{2}\)

\(R=\frac{R_{1}\times{R_{2}}}{R_{1}+R_{2}}\)

В данной формуле \(I_{1},U_{1},R_{1}\) являются электрические характеристики первого участка цепи, а \(I_{2},U_{2},R_{2}\) являются электрические характеристики второго участка цепи. Единицы измерения: ампер, вольт и ом соответственно.

Формула мощности электрического тока

Мощность электрического тока — физическая величина, которая определяет, какую работу совершит ток за конкретный промежуток времени.

Мощность тока можно вычислить по следующей формуле:

\(P=\frac{A}{t}\)

В данной формуле P является мощностью тока, A является работой электрического тока на участке цепи, t является временем, в течение которого электрический ток совершал определенную работу.

Есть и другой вариант данной формулы:

Формула 6

\(P=I\times{U}\)

В данной формуле I является силой тока, U является электрическим напряжением на участке цепи. Единицей измерения является ватт.

Формула для закона преломления света

Луч падающий и преломленный, плюс перпендикуляр к границе раздела двух сред, находятся в единой плоскости, причем отношение синуса угла падения к синусу угла преломления будет значением постоянной для таких сред — относительный показатель преломления \(n_{21}\).

Формула закона преломления света:

\(n_{21}=\frac{\sin\alpha}{\sin\beta}\)

Формулы по физике за 9 класс

В программе 9 класса школьникам необходимы формулы для вычисления следующих тем: проекция вектора перемещения; скорость равномерного движения; уравнение движения (то есть зависимость координаты от времени) в случае равномерного движения; движение тела по окружности; закон всемирного тяготения; импульс тела; связь между частотой и периодом колебания; скорость волны; электрическая емкость конденсатора; энергия связи (формула Альберта Эйнштейна).

Формула проекции вектора перемещения

Проекция вектора перемещения на ось будет равняться разности между начальной и конечной координатами тела по определенной оси.

Источник: presentacii.ru

Формула скорости равномерного движения

Скорость равномерного прямолинейного движения — постоянная векторная величина, равная отношению перемещения объекта ко времени, за которое произошло перемещение.

Расчет скорости равномерного прямолинейного движения производят по следующей формуле:

\(\overrightarrow{V}=\frac{\overrightarrow{S}}{t}\)

В данной формуле \(\overrightarrow{V}\) является скоростью тела, \(\overrightarrow{S}\) является путем, который объект проходит, а t — время, за которое тело проходит конкретный путь.

Обратите внимание, что вектор скорость всегда направляется в сторону движения. Единицей измерения скорости равномерного прямолинейного движения являются метры в секунду или километры в час.

Выражение движения (при зависимости координаты от времени) при равномерном движении

Формула 7

Уравнение облагает следующим видом: \(x=x_{0}+vt\). {-11}\). Единицы измерения — Ньютоны.

Формула импульса тела

Импульс тела — векторная физическая величина, равная произведению массы тела на скорость тела.

Формула импульса:

\(\overrightarrow{p}=m\times\overrightarrow{V}\)

В формуле \(\overrightarrow{p}\) является импульсом тела, m является массой тела, \(\overrightarrow{V}\) является скоростью движения. Единица измерения — кг на мс.

Формула связи между периодом и частотой колебаний

Период — время единичного полного колебания. Частота — количество полных колебаний за единицу времени (берется одна секунда). Период и частота свободных колебаний нитяного маятника зависит от длины нити маятника.

Между частотой и периодом колебаний есть обратно-пропорциональная зависимость: чем больше будет период колебаний, тем меньше значение частоты, и наоборот.

Формула:

\(T=\frac{1}{v}\)

\(v=\frac{1}{t}\)

В данных формулах T является периодом колебаний, а v является частотой колебаний. Единица измерения частоты — герц, периода — секунда.

Формула скорости волны

Скорость волны — скорость распространения колебаний в условиях упругой среды.

Рассчитывается по формуле: 

\(V=\lambda\times{v}\)

В данной формуле V является скоростью волны, \(\lambda\) является длиной волны, а v является частотой волны. Измеряется в метрах в секунду.

Формула электрической емкости конденсатора

Конденсатор — это два проводника, которые находятся на малом расстоянии друг от друга, разделяются слоем диэлектрика. Электроемкость — это физическая величина, которая характеризует умение проводника накапливать некоторое количество электрического заряда. 

Электроемкость конденсатора зависит от нескольких параметров. Среди них размер проводников, форма проводников, расстояния между проводниками, электрические свойства диэлектрика. Электрическая емкость конденсатора никак не зависит от размера заряда, напряжения и материала проводников. {2}}\)

В данной формуле \(\triangle{m}\) является дефектом массы ядра, а c является скоростью света в вакууме.

 Все формулы за 7-9 классы в виде таблицы: 

Источник: fgoskomplekt.ru

Насколько полезной была для вас статья?

У этой статьи пока нет оценок.

19.4 Электроэнергетика — Физика

Раздел Цели обучения

К концу этого раздела вы сможете делать следующее:

  • Давать определение электрической мощности и описывать уравнение электрической мощности
  • Расчет электрической мощности в цепях резисторов, соединенных последовательно, параллельно и комплексно

Поддержка учителей

Поддержка учителей

Цели обучения в этом разделе помогут вашим учащимся освоить следующие стандарты:

  • (5) Научные понятия. Учащийся знает природу сил в физическом мире. Ожидается, что студент:
    • (Ж) проектировать, конструировать и рассчитывать с точки зрения сквозного тока, разности потенциалов, сопротивления и мощности, используемой элементами электрической цепи, соединенными как последовательно, так и параллельно.

Кроме того, руководство по физике для средней школы рассматривает содержание этого раздела лабораторной работы под названием «Работа, энергия и мощность в цепях», а также следующие стандарты:

  • (6) Научные концепции. Учащийся знает, что изменения происходят в физической системе, и применяет законы сохранения энергии и импульса. Ожидается, что студент:
    • (С) рассчитать механическую энергию, мощность, генерируемую внутри, приложенный к ней импульс и импульс физической системы.

Основные термины раздела

электроэнергия

Энергия у многих ассоциируется с электричеством. Каждый день мы используем электроэнергию для работы наших современных приборов. Линии электропередачи являются наглядными примерами электроснабжения. Мы также используем электроэнергию, чтобы заводить автомобили, компьютеры или освещать дома. Мощность — это скорость передачи энергии любого типа; электрическая мощность – это скорость, с которой электрическая энергия передается в цепи. В этом разделе мы узнаем не только, что это значит, но и какие факторы определяют электрическую мощность.

Для начала давайте подумаем об лампочках, которые часто характеризуются номинальной мощностью в ваттах. Сравним лампочку мощностью 25 Вт с лампочкой мощностью 60 Вт (см. рис. 19.20). Хотя оба работают при одинаковом напряжении, лампа мощностью 60 Вт излучает больше света, чем лампа мощностью 25 Вт. Это говорит нам о том, что выходная мощность электрической цепи определяется не напряжением, а чем-то иным.

Лампы накаливания, такие как две, показанные на рис. 19.20, по существу представляют собой резисторы, которые нагреваются, когда через них проходит ток, и нагреваются настолько, что излучают видимый и невидимый свет. Таким образом, две лампочки на фотографии можно рассматривать как два разных резистора. В простой цепи, такой как лампочка с приложенным к ней напряжением, сопротивление определяет ток по закону Ома, поэтому мы можем видеть, что ток, как и напряжение, должен определять мощность.

Рисунок 19.20 Слева лампочка на 25 Вт, а справа лампочка на 60 Вт. Почему их выходная мощность различна, несмотря на то, что они работают на одном и том же напряжении?

Формулу мощности можно найти с помощью размерного анализа. Рассмотрим единицы мощности. В системе СИ мощность указывается в ваттах (Вт), что представляет собой энергию в единицу времени, или Дж/с

Вт=Джс.Вт=Джс.

19,47

Вспомним теперь, что напряжение — это потенциальная энергия на единицу заряда, а это означает, что единицы измерения напряжения составляют Дж/Кл

В=ДК.В=ДК.

19,48

Мы можем переписать это уравнение как J=V×CJ=V×C и подставить его в уравнение для ватт, чтобы получить

W=Js=V×Cs=V×Cs.W=Js=V× Cs=V×Cs.

Но кулон в секунду (Кл/с) — это электрический ток, который мы можем видеть из определения электрического тока, I=ΔQΔtI=ΔQΔt, где ΔΔ Q — заряд в кулонах, а ΔΔ t — время в секундах. Таким образом, приведенное выше уравнение говорит нам, что электрическая мощность равна напряжению, умноженному на ток, или

П=IV.П=IV.

Это уравнение дает электрическую мощность, потребляемую цепью с падением напряжения В и током I .

Например, рассмотрим схему на рис. 19.21. По закону Ома ток, протекающий через цепь, равен

I=VR=12V100Ω=0,12A.I=VR=12V100Ω=0,12A.

19,49

Таким образом, мощность, потребляемая схемой, равна

P=VI=(12 В)(0,12 А)=1,4 Вт. P=VI=(12 В)(0,12 А)=1,4 Вт.

19,50

Куда уходит эта сила? В этой схеме мощность идет в основном на нагрев резистора в этой цепи.

Рисунок 19.21 Простая схема, потребляющая электроэнергию.

При расчете мощности в цепи на рис. 19.21 мы использовали сопротивление и закон Ома для определения силы тока. Закон Ома дает ток: I=V/RI=V/R, который мы можем подставить в уравнение для электрической мощности, чтобы получить

P=IV=(VR)V=V2RP. P=IV=(VR)V= В2Р.

Мощность определяется только напряжением и сопротивлением.

Мы также можем использовать закон Ома, чтобы исключить напряжение из уравнения для электрической мощности и получить выражение для мощности только через ток и сопротивление. Если мы запишем закон Ома как V=IRV=IR и используя это, чтобы исключить V в уравнении P=IVP=IV, мы получаем

P=IV=I(IR)=I2R.P=IV=I(IR)=I2R.

Мощность определяется только током и сопротивлением.

Таким образом, комбинируя закон Ома с уравнением P=IVP=IV для электрической мощности, мы получаем еще два выражения для мощности: одно через напряжение и сопротивление, а другое через ток и сопротивление. Обратите внимание, что в выражения для электрической мощности входят только сопротивление (а не емкость или что-то еще), ток и напряжение. Это означает, что физической характеристикой цепи, которая определяет, сколько мощности она рассеивает, является ее сопротивление. Любые конденсаторы в цепи не рассеивают электроэнергию — напротив, конденсаторы либо накапливают электроэнергию, либо отдают ее обратно в цепь.

Чтобы понять, как связаны между собой напряжение, сопротивление, ток и мощность, рассмотрите рисунок 19.22, на котором показано колесо формул . Величины в центральной четверти окружности равны количествам в соответствующей внешней четверти окружности. Например, чтобы выразить потенциал V через мощность и ток, мы видим из круга формул, что V=P/IV=P/I.

Рисунок 19.22 Колесо формул показывает, как связаны вольты, сопротивление, ток и мощность. Количества во внутренних четвертях кругов равны количествам в соответствующих внешних четвертях кругов.

Рабочий пример

Найдите сопротивление лампочки

Типичная старая лампочка накаливания имела мощность 60 Вт. Если предположить, что на лампочку подается напряжение 120 В, какова сила тока через лампочку?

Стратегия

Нам известны напряжение и выходная мощность простой цепи, содержащей лампочку, поэтому мы можем использовать уравнение P=IVP=IV, чтобы найти ток I , протекающий через лампочку.

Решение

Решение P=IVP=IV для тока и вставка заданных значений напряжения и мощности дает

P=IVI=PV=60W120V=0,50A.P=IVI=PV=60W120V=0,50A.

19,51

Таким образом, через лампочку проходит полампера, когда на нее подается напряжение 120 В.

Обсуждение

Это значительный ток. Напомним, что бытовая электроэнергия является переменным, а не постоянным током, поэтому 120 В, подаваемые из бытовых розеток, представляют собой переменную, а не постоянную мощность. 120 В — это фактически усредненная по времени мощность, выдаваемая такими розетками. Таким образом, средний ток, проходящий через лампочку за период времени, превышающий несколько секунд, составляет 0,50 А.

Рабочий пример

Грелки для ботинок

Чтобы согреть ботинки в холодные дни, вы решили вшить в стельку ботинок схему с резисторами. Вам нужно 10 Вт тепла от резисторов в каждой стельке, и вы хотите питать их от двух 9-вольтовых батарей (соединенных последовательно). Какое общее сопротивление вы должны оказывать на каждую стельку?

Стратегия

Мы знаем желаемую мощность и напряжение (18 В, потому что у нас есть две 9-вольтовые батареи, соединенные последовательно), поэтому мы можем использовать уравнение P=V2/RP=V2/R, чтобы найти необходимое сопротивление.

Решение

Решая P=V2/RP=V2/R для сопротивления и подставляя данные напряжения и мощности, мы получаем

P=V2RR=V2P=(18V)210W=32Ω.P=V2RR=V2P=(18V )210 Вт=32 Ом.

19,52

Таким образом, общее сопротивление в каждой стельке должно быть 32 Ом.Ом.

Обсуждение

Давайте посмотрим, какой ток будет проходить по этой цепи. У нас есть 18 В, приложенные к сопротивлению 32 Ом, поэтому закон Ома дает

I=VR=18V32Ω=0,56A.I=VR=18V32Ω=0,56A.

19,53

Все аккумуляторы имеют маркировку, на которой указано, сколько заряда они могут обеспечить (в пересчете на ток, умноженный на время). Типичная щелочная батарея 9 В может обеспечить заряд 565 мА⋅чмА⋅ч. (таким образом, две 9-вольтовые батареи обеспечивают 1130 мА⋅чмА⋅ч ), поэтому эта система отопления будет работать в течение времени

t=1130×10−3A⋅ч0,56A=2,0ч.t=1130×10−3A⋅ч0. 56А=2,0ч.

19,54

Рабочий пример

Питание через ветвь цепи

Сопротивление каждого резистора в схеме ниже составляет 30 Ом. Какая мощность рассеивается на средней ветви цепи?

Стратегия

Средняя ветвь схемы содержит последовательно соединенные резисторы R3 и R5R3 и R5. Напряжение на этой ветви составляет 12 В. Сначала мы найдем эквивалентное сопротивление в этой ветви, а затем с помощью P=V2/RP=V2/R найдем мощность, рассеиваемую в ветви.

Решение

Эквивалентное сопротивление равно Rmiddle=R3+R5=30Ω+30Ω=60ΩRmiddle=R3+R5=30Ω+30Ω=60Ω . Мощность, рассеиваемая средней ветвью цепи, составляет

Pmiddle=V2Rmiddle=(12В)260Ом=2,4Вт. Pmiddle=V2Rmiddle=(12В)260Ом=2,4Вт.

19,55

Обсуждение

Давайте посмотрим, сохраняется ли энергия в этой цепи, сравнив мощность, рассеиваемую в цепи, с мощностью, выдаваемой батареей. Во-первых, эквивалентное сопротивление левой ветви составляет

Rleft=11/R1+1/R2+R4=11/30Ω+1/30Ω+30Ω=45Ω.Rleft=11/R1+1/R2+R4=11/ 30 Ом + 1/30 Ом + 30 Ом = 45 Ом.

19,56

Мощность через левую ветвь

Pleft=V2Rleft=(12В)245Ом=3,2Вт.Pleft=V2Rleft=(12В)245Ом=3,2Вт.

19,57

Правая ветвь содержит только R6R6, поэтому эквивалентное сопротивление Rright=R6=30ΩRright=R6=30Ω . Мощность через правую ветвь составляет

Pright=V2Rright=(12В)230Ом=4,8Вт.Pright=V2Rright=(12В)230Ом=4,8Вт.

19,58

Общая мощность, рассеиваемая цепью, представляет собой сумму мощностей, рассеиваемых в каждой ветви.

P=Pleft+Pmiddle+Pright=2,4W+3,2W+4,8W=10,4WP=Pleft+Pmiddle+Pright=2,4W+3,2W+4,8W=10,4W

19,59

Мощность, обеспечиваемая аккумулятором

П=IV. П=IV.

19,60

где I — общий ток, протекающий через батарею. Поэтому мы должны сложить токи, проходящие через каждую ветвь, чтобы получить I . Ответвления вносят токи

0,40А.

19,61

Суммарный ток равен

I=Iлевый+Iсредний+Iправый=0,2667A+0,20A+0,40A=0,87A.I=Iлевый+Iсредний+Iправый=0,2667A+0,20A+0,40A=0,87A.

19,62

и мощность, обеспечиваемая аккумулятором, равна

P=IV=(0,87A)(12В)=10,4Вт.P=IV=(0,87А)(12В)=10,4Вт.

19,63

Это та же мощность, которая рассеивается на резисторах цепи, что показывает, что в этой цепи сохраняется энергия.

Практические задачи

16.

Какова формула мощности, рассеиваемой на резисторе?

  1. Формула мощности, рассеиваемой на резисторе: P=IV. P=IV.
  2. Формула для мощности, рассеиваемой на резисторе: P=VI.P=VI.
  3. Формула для мощности, рассеиваемой на резисторе: P = IV .
  4. Формула для мощности, рассеиваемой на резисторе: P = I 2 В .

17.

Какова формула для мощности, рассеиваемой резистором, с учетом его сопротивления и напряжения на нем?

  1. Формула для мощности, рассеиваемой на резисторе: P=RV2P=RV2
  2. Формула мощности, рассеиваемой на резисторе: P=V2RP=V2R
  3. Формула мощности, рассеиваемой на резисторе: P=V2RP=V2R
  4. Формула для мощности, рассеиваемой на резисторе: P=I2RP=I2R

Проверьте свое понимание

18.

Какие элементы схемы рассеивают мощность?

  1. конденсаторы
  2. катушки индуктивности
  3. идеальные переключатели
  4. резисторы

19.

Объясните словами уравнение для мощности, рассеиваемой на данном сопротивлении.

  1. Электрическая мощность пропорциональна току через резистор, умноженному на квадрат напряжения на резисторе.

  2. Электрическая мощность пропорциональна квадрату тока через резистор, умноженному на напряжение на резисторе.

  3. Электрическая мощность пропорциональна току через резистор, деленному на напряжение на резисторе.

  4. Электрическая мощность пропорциональна току через резистор, умноженному на напряжение на резисторе.

Сила (физика): определение, формула, единицы измерения, как найти (с примерами)

Бодибилдер и пятиклассник могли бы поднять все книги с полки на лестничный пролет, но это вряд ли выполнить задание за одинаковое время. Бодибилдер, вероятно, будет быстрее, потому что у него выше 9 баллов.0341 номинальная мощность , чем у пятиклассника.

Точно так же гоночный автомобиль с высокой мощностью лошадиных сил сможет проехать дальше намного быстрее, чем, ну, в общем, лошадь.

TL;DR (слишком длинно, не читал)

Мощность — это мера того, сколько работы выполняется за интервал времени.

Небольшое примечание о лошадиных силах: этот термин предназначен для сравнения мощности парового двигателя с мощностью лошади, так как двигатель мощностью 700 лошадиных сил может выполнять примерно в 700 раз больше работы, чем одна лошадь. Это восходит к тому времени, когда паровые двигатели были новыми, и один из самых выдающихся изобретателей, работавших над повышением их эффективности, Джеймс Уатт придумал этот термин, чтобы убедить среднего человека в их ценности.

Формулы для мощности

Существует два способа расчета мощности в зависимости от доступной информации. Кроме того, есть две единицы мощности, которые одинаково действительны.

1. Мощность в пересчете на работу и время:

P=\frac{W}{t}

Где работа ​ Вт ​ измеряется в ньютон-метрах (Нм), а время ​ t ​измеряется в секундах (с).

2. Мощность по силе и скорости:

P=Fv

Где сила ​ F ​ в ньютонах (Н), а скорость ​ v ​ в метрах в секунду (м/с).

Эти уравнения не являются случайно эквивалентными. Второе уравнение может быть получено из первого следующим образом:

Обратите внимание, что работа ​ это то же самое, что сила, умноженная на перемещение:

W=Fd

Подставьте это в первое уравнение мощности:

Затем, поскольку перемещение в любую единицу времени равно скорости ​ (v = d/t), перепишите термины в конце как ​ v ​, чтобы получить уравнение второй степени.

Единицы мощности

Единица мощности в системе СИ p обычно представлена ​​как Вт (Вт) ​, названная в честь того же Джеймса Уатта, который разработал двигатели и сравнил их с лошадьми. На бирках лампочек и других бытовых приборов обычно указывается эта единица.

Однако вторая формула мощности приводит к другой единице измерения. Сила, умноженная на скорость, измеряется в ньютон-метрах в секунду (Нм/с). Затем, поскольку единица энергии джоуль также определяется как один ньютон-метр (Нм), первая часть этого может быть вместо этого переписана как джоуль, что приводит к второй единице мощности в системе СИ:0405 Джоулей в секунду (Дж/с). ​

  • Мощность может измеряться в ваттах (Вт) или джоулях в секунду (Дж/с).

Как стать могущественным

Принимая во внимание определение силы и два способа ее получения, можно найти несколько способов ​ увеличить силу чего-либо та же работа выполняется быстрее (уменьшение t ​ или увеличение t ​). Мощный автомобиль силен и быстрые, а слабый ни то, ни другое. Чем легче и быстрее можно выполнить работу , тем мощнее сущность, выполняющая работу.

  • Как повысить мощность: сделать больше за более короткий период времени.

Это также означает, что очень мощная машина, скажем, мускулистый бодибилдер, все еще может ​ не хватать мощности ​. Человек, который может поднять очень тяжелый груз, но только очень медленно, менее силен, чем тот, кто может поднять его быстро.

Точно так же очень быстрая машина или человек, который мало что делает, кто-то быстро вертится на месте, но никуда не двигается, на самом деле не обладает силой.

Пример расчета мощности

1. Усэйн Болт выработал около 25 Вт энергии в своем рекордном спринте на 100 м, который длился 9,58 секунды. Сколько работы он сделал?

Поскольку ​ P ​ и ​ t ​ даны, а ​ W ​ неизвестно, используйте первое уравнение:

P=\frac{W}{t}\implies 25=\frac{ Вт}{9. 58}\implies W=239.5\text{ Нм}

2. С какой средней силой он отталкивался от земли во время бега?

Так как ​ работа ​ в Нм уже известна, как и ​ перемещение ​ в метрах, деление на длину пробега даст ​ силу ​ (иными словами, ​ работа ​равно силе, умноженной на перемещение: W = F × d):

\frac{239,5}{100}=2,395\text{ N}

6 секунд, чтобы подняться по 3-метровой лестнице?

В этой задаче заданы перемещение и время, что позволяет быстро рассчитать скорость:

v=\frac{d}{t}=\frac{3}{6}=0,5\text{ м/с}

Во втором уравнении мощности есть скорость, но также и сила. Человек, взбегающий по лестнице, пытается противостоять силе тяжести. Итак, силу в этом случае можно найти, используя их массу и ускорение свободного падения, которое на Земле всегда равно 9,8 м/с 2 .

F_{грав}=мг=48\умножить на 9.8=470,4\text{ N}

Теперь сила и скорость вписываются во вторую формулу для мощности:

=Fv=470,4\times 0,5 = 235,2\text{ Дж/с}

Обратите внимание на решение оставить единицы измерения здесь как Дж / с, а не Вт является произвольным. Столь же приемлемым ответом является 235,2 Вт.

4. Одна лошадиная сила в единицах СИ составляет около 746 Вт, что основано на нагрузке, которую здоровая лошадь могла бы нести в течение одной минуты. Какую работу за это время выполнила лошадь?

Единственный шаг перед подстановкой значений мощности и времени в первое уравнение — убедиться, что время указано в правильных единицах СИ (секунды), переписав одну минуту как 60 секунд. Тогда:

P=\frac{W}{t}\implies 746=\frac{W}{60}\implies W=44 670\text{ Нм}

Киловатты и электричество

Многие электроэнергетические компании взимают плату с клиентов исходя из их киловатт-часов ​ использования. Чтобы понять значение этой общей единицы электрической мощности, начните с разбивки единиц.

Префикс кило означает 1000, поэтому киловатт (кВт) равен 1000 Вт. Таким образом, ​ киловатт-час (кВтч) — это количество киловатт, израсходованных за один час.

Чтобы подсчитать киловатт-часы, умножьте количество киловатт на использованные часы. Таким образом, если кто-то использует 100-ваттную лампочку в течение 10 часов, он израсходует в общей сложности 1000 ватт-часов или 1 кВтч электроэнергии.

Киловатт-час Примеры задач

1. Электроэнергетическая компания взимает 0,12 доллара за киловатт-час. Очень мощный пылесос мощностью 3000 Вт используется в течение 30 минут. Во сколько домовладельцам обходится это количество энергии?

3000 Вт = 3 кВт

30 минут = 0,5 часа

3\text{ кВт}\times 0,5\text{ ч}= 1,5\text{ кВтч}\text{ и }1,5\text{ кВтч}\times 0,12\text{доллар/кВтч} = \$0,18

2. Та же коммунальная служба начисляет домохозяйству 10 долларов за каждые 4 кВтч электроэнергии, возвращаемой в сеть. Солнце дает около 1000 Вт энергии на квадратный метр. Если солнечная батарея площадью два квадратных метра в доме собирает энергию в течение 8 часов, сколько денег она приносит?

Учитывая информацию в задаче, солнечная батарея должна быть в состоянии собрать 2000 Вт от Солнца, или 2 кВт.

Решить задачу по физике онлайн бесплатно: бесплатно физика онлайн решение задач

бесплатно физика онлайн решение задач

Вы искали бесплатно физика онлайн решение задач? На нашем сайте вы можете получить ответ на любой математический вопрос здесь. Подробное решение с описанием и пояснениями поможет вам разобраться даже с самой сложной задачей и бесплатно физика решение задач онлайн, не исключение. Мы поможем вам подготовиться к домашним работам, контрольным, олимпиадам, а так же к поступлению в вуз. И какой бы пример, какой бы запрос по математике вы не ввели — у нас уже есть решение. Например, «бесплатно физика онлайн решение задач».

Применение различных математических задач, калькуляторов, уравнений и функций широко распространено в нашей жизни. Они используются во многих расчетах, строительстве сооружений и даже спорте. Математику человек использовал еще в древности и с тех пор их применение только возрастает. Однако сейчас наука не стоит на месте и мы можем наслаждаться плодами ее деятельности, такими, например, как онлайн-калькулятор, который может решить задачи, такие, как бесплатно физика онлайн решение задач,бесплатно физика решение задач онлайн,задачи онлайн по физике,задачи по физике онлайн,задачи по физике онлайн решать,задачи по физике онлайн решение,задачи по физике онлайн решение бесплатно,задачи по физике решать онлайн,задачи по физике решение онлайн,задачи по физике решение онлайн бесплатно,задачник онлайн по физике,калькулятор для решения задач по физике,калькулятор задач по физике,калькулятор по физике для решения задач,калькулятор по физике для решения задач онлайн,калькулятор по физике решение задач,калькулятор решение задач по физике,калькулятор решения задач по физике,онлайн задачи по физике,онлайн задачи по физике решать,онлайн задачи по физике решить,онлайн калькулятор для решения задач по физике,онлайн калькулятор по физике решение задач,онлайн калькулятор решение задач по физике,онлайн решатель задач по физике,онлайн решать задачи по физике,онлайн решение задач по физике,онлайн решение задач по физике бесплатно,онлайн решение задач по физике бесплатно и без регистрации,онлайн решение задач по физике с помощью программы,онлайн решение задач физика бесплатно,онлайн решение задачи по физике,онлайн решение уравнений по физике,онлайн решение физика,онлайн решение физики,онлайн решение физических задач,онлайн решения задач по физике,онлайн решить задачу по физике,онлайн физика бесплатно решение задач,онлайн физика решение,помощь по физике онлайн бесплатно,решатель задач по физике,решатель задач по физике онлайн,решать задачи онлайн по физике,решать задачи по физике онлайн,решать онлайн задачи по физике,решение задач онлайн бесплатно физика,решение задач онлайн по физике,решение задач онлайн по физике бесплатно,решение задач онлайн физика,решение задач по физике калькулятор,решение задач по физике онлайн,решение задач по физике онлайн база готовых решений бесплатно,решение задач по физике онлайн бесплатно,решение задач по физике онлайн бесплатно и без регистрации,решение задач по физике онлайн калькулятор,решение задач физика онлайн бесплатно,решение задачи онлайн по физике,решение задачи по физике онлайн,решение задачи по физике онлайн бесплатно,решение онлайн задач бесплатно физика,решение онлайн задачи по физике,решение онлайн физика,решение онлайн физики,решение онлайн физических задач,решение уравнений онлайн по физике,решение уравнений по физике онлайн,решение физика онлайн,решение физики онлайн,решение физических задач онлайн,решения задач онлайн по физике,решения задач по физике онлайн,решения онлайн задач по физике,решить бесплатно задачи по физике онлайн,решить бесплатно онлайн задачи по физике,решить задачи по физике бесплатно онлайн,решить задачи по физике онлайн бесплатно,решить задачу онлайн по физике,решить задачу онлайн по физике бесплатно,решить задачу по физике онлайн,решить задачу по физике онлайн бесплатно,решить задачу по физике онлайн бесплатно без регистрации,розв язати задачу з фізики,физика онлайн решение,физика онлайн решение задач,физика онлайн решение задач бесплатно,физика решение задач бесплатно онлайн,физика решение задач онлайн,физика решение задач онлайн бесплатно,физика решение онлайн,физики решение. На этой странице вы найдёте калькулятор, который поможет решить любой вопрос, в том числе и бесплатно физика онлайн решение задач. Просто введите задачу в окошко и нажмите «решить» здесь (например, задачи онлайн по физике).

Решить задачу бесплатно физика онлайн решение задач вы можете на нашем сайте https://pocketteacher.ru. Бесплатный онлайн решатель позволит решить онлайн задачу любой сложности за считанные секунды. Все, что вам необходимо сделать — это просто ввести свои данные в решателе. Так же вы можете посмотреть видео инструкцию и узнать, как правильно ввести вашу задачу на нашем сайте. А если у вас остались вопросы, то вы можете задать их в чате снизу слева на странице калькулятора.

Решение задач по физике по фото

На носу экзамены или контрольная работа — приложения помогут вам найти решение трудных задач по физике при помощи фото. Нет необходимости искать варианты ответов вручную. За вас всё сделает умных программный алгоритм. Попробуйте использоваться описанные ниже приложения.

Содержание

  1. Поиск ответов по физике через Google Lens
  2. Решение задач по фото через Яндекс Картинки
  3. «Физика — формулы» — удобное мобильное приложение
  4. «Бетафизика» — поможет решить любые сложные задачи
  5. Видео-инструкция

Поиск ответов по физике через Google Lens

Если задача существует в учебнике, значит её решение можно будет найти в поисковых системах. Но вручную сделать это довольно трудно. Используйте мобильно приложение Google Lens.

Это простой и удобный вариант обычной камеры на смартфоне. Но она работает через свою поисковую систему Гугл. Всё, что попадёт в камеру приложение попытается найти в Интернете. Изображение лучше всего делать прямо из учебника печатным текстом. В таком случае больше вероятности, что пользователь получит в ответ ожидаемый результат.

Приложение от Гугл способно различать через камеру мельчайшие подробности и учитывать их при поиске. Попробуйте с этим приложением решить свою задачу, найдя готовый ответ в Интернете. Кроме этого Google Lens умеет различать и другие предметы на фото, такие как: насекомые, архитектурные объекты, текст на любом языке и переводит его, узнаёт товары на фото и помогает их найти. После решения задач по физике вам пригодится эта программа для поиска в сети любых предметов.

Это интересно: поиск одежды по фото онлайн.

Решение задач по фото через Яндекс Картинки

Не менее эффективным инструментом для поиска решений для сложных задач по физике являются Яндекс.Картинки. Этот раздел для поиска по фото можно найти в Яндекс Браузере или мобильном приложении Яндекс. При запуске приложения в поиске необходимо выбрать «Картинки», после чего предоставить приложению фотографию вашей задачи. Картинке отобразится в поисковой строке, а результаты по-прежнему будут появляться ниже, как при стандартном поиске.

В верхней части у поисковой строке находятся кнопки с категориями картинок. Из них вы сможете выбрать подходящую категорию, чтобы поиск был более точным по вашему запросу. А справа в окне находится кнопка «Фильтры». С их помощью можно корректировать результаты поиска и выбирать по необходимости. Используйте сразу несколько разных приложений и фото, чтобы найти подходящий ответ на задачу по физике. Попробуйте применять разные поисковые системы, включая те, на которых нет возможности искать по картинке.

Читайте также: поиск по фото с телефона в Яндекс.

«Физика — формулы» — удобное мобильное приложение

Полезным инструментом для решения задач по физике является приложение «Физика — формулы». Аналогичное приложение, которое считывает по фото, есть в App Store для устройств с IOS. Это целый справочник по науке с большим количеством формул, полезных таблиц и теорий.

Если вы готовитесь к ЕГЭ, то программа не даст вам завалить экзамен. Им можно будет пользоваться в любом месте, ведь справочник будет всегда в вашем телефоне. Удобный и простой интерфейс: в верхних вкладках выбирайте то, что вам нужно.

В «Физика — формулы» вы найдёте:

  • Более 200 непонятных для ученика терминов из физики;
  • Набор полезных формул;
  • Несколько десятков таблиц, с которыми решение задач будет более простым;
  • Более 50 специализированных калькуляторов, которые необходимы для расчётов;
  • Умный поиск по всем материалам сразу в программе.

Приложение поможет любому ученику закрепить полученные ранее знания и подготовиться к экзамену в любых учебных заведениях. Узнайте новые формулы по физике, о которых вы могли и не знать или не могли найти решение. Если вы услышали новый термин или слово на уроке, то быстро сможете узнать его значение из приложения «Физика — формулы». Незаменимый помощник при решении задач любой сложности. Попробуйте применить этот инструмент.

«Бетафизика» — поможет решить любые сложные задачи

Не просто решить задачу по физике, если её условия слишком сложны для понимания. Можно поступить другим способом: сначала узнать решение, а затем разобраться в самой задаче. А если вы не знаете, где его взять, то подсказываем — в Google Play. В данном приложении есть строка поиска, по которой можно найти всё, что угодно по предмету по фото. Введите название формулы в поисковой строке или часть условия задания. А приложение попытается вам помочь.

Если поисковая строка не даст результатов, нажмите вверху на панели иконку, которая похожа на крошечный фотоаппарат. На экране появится окно с выделением, которое нужно сфокусировать на условии. Сделайте снимок экрана и попытайтесь найти подходящий вариант решения. Приложение проводит поиск по своей базе данных и незамедлительно отображает материал по запросу. Полное решение с описанием действий приложение может показать лишь тем пользователям, которые оформили платную подписку.

Бесплатно можно только найти решение. Программа будет полезна всем учащимся средних школ. Ориентирована на общеобразовательные учреждения. Встроены языки: русский, украинский, казахский, румынский, белорусский, молдавский и другие.

Видео-инструкция

Рассмотрены все полезные программы по решению непростых задач по физике при помощи фото. В видео интересные лайфхаки для тех, кому приходится ломать голову над уроками.

Поиск в нашей библиотеке домашних заданий

Вопросы и ответы по физике — Сомнения, задачи, решения домашних заданий по физике

Все уровни13 PlusA LevelGCSEIBKS3Scottish HighersUniversity

Часть А) В жидкостях происходит конвекция. В этом процессе более энергичные частицы перемещаются из более теплых областей в более холодные и уносят с собой свою энергию.

Конвекционный ток возникает потому, что …

В БАК протоны разгоняются почти до скорости света. Они удерживаются в круговом ускорителе сверхпроводящими дипольными магнитами 1232. Насколько силен магический …

Бесконечная квадратная яма — это область с постоянной потенциальной энергией (внутри ямы), окруженная областью бесконечной потенциальной энергии (снаружи ямы). Поэтому нельзя за …

Горизонтальное положение x (измеренное от оптической оси) точки на экране на расстоянии D от двойной щели с разделением щелей a связано с …

Температура – ​​это макроскопический эффект средней кинетической энергии субатомных частиц, составляющих материал. Поэтому используются кельвины, потому что это единственная истинная шкала, которая …

Когда большинство студентов начинают свои курсы физики, первая форма энергии, о которой они узнают, — это кинетическая энергия, которая связана со скоростью, которую имеет объект. И наоборот, потенциал …

Просто приравняйте центростремительную силу и силу тяжести, действующую на МКС и найдите v.

P. S. Погуглите радиус и массу Земли, расстояние до МКС…

Электродвижущая сила или ЭДС ячейки в цепи, для начала, вовсе не сила: она имеет единицу измерения вольт. Хорошее определение это …

Этот вопрос относится к поведению газов при сжатии. В IB мы моделируем такие ситуации, используя уравнение идеального газа, которое утверждает, что:

pV = nRT,

где:

p — …

В физике есть два типа чисел: скаляры и векторы. Скаляры — это просто обычные числа, к которым мы привыкли. Векторы также имеют связанное с ними направление. …

Типовой трансформатор состоит из железного сердечника с намотанными вокруг него двумя катушками — первичной и вторичной. Любой провод с током создает вокруг себя магнитное поле, но …

Это распространенный вопрос IB — довольно сложный, когда вы сталкиваетесь с ним впервые!

Модель электрона в ящике использует идею о том, что энергия электрона и энергия стоячей волны в ящике аналогичны. Итак, используя свои знания…

Чтобы ответить на этот вопрос, важно помнить, что атом имеет энергию в двух формах: кинетическую (энергия, которая позволяет атому двигаться) и потенциальную (энергия …

).

Когда я впервые столкнулся с этой проблемой, я был немного сбит с толку из-за приведенной выше аналогии с автомобилем. Так что думать об этом можно так: ваше тело и машина…

Если вас попросят указать единицы измерения какой-либо сложной величины в более простых единицах, попробуйте придумать известное вам физическое уравнение, связывающее эту сложную величину (например, энергию, …

Ответ: 25. Объяснение: В двоичной системе каждая цифра может быть одним из двух значений: 0 и 1. Это две возможности («би» на латыни означает «дважды»). В десятичной системе …

  1. всегда рисуйте диаграмму свободного тела. Это позволит уточнить силы, действующие на объект

    .
  2. Определить, какой закон Ньютона действует: 1-й или 2-й, и использовать …

При вычислении эквивалентности единиц наиболее простым подходом является использование формул (часто в буклете данных), которые помогут получить правильный ответ. Начнем с Джоулей; …

Разделить векторы вертикальной и горизонтальной скорости, рассчитать вертикальное ускорение (под действием силы тяжести), рассчитать время до удара объекта о землю, использовать время для расчета горизонтального расстояния. Используйте уравнения «большой тройки».

Онлайн Калькулятор Вентиляторов | Как рассчитать массовый расход, CFM вентилятора?

Создатель: Нааз Фатима

Отзыв: Раджашекхар Валипишетти

Последнее обновление: 10 апр. 2023 г.


Калькулятор вентилятора представляет собой удобный и гибкий инструмент, который поможет вам рассчитать массовый расход и CFM с учетом соответствующих входных данных. Все, что вам нужно сделать, это предоставить выходную мощность, давление, чтобы узнать CFM за считанные секунды.

Калькулятор вентилятора: Хотите рассчитать воздухообмен, необходимый для здания? Если это так, вы можете использовать наш удобный инструмент, например, вентилятор или калькулятор CFM, и легко рассчитать кубические футы в минуту, введя необходимые данные, как в полях ввода инструмента. Мы рассмотрели различные типы вентиляторов, что означает CFM, как найти массовый расход и познакомим вас с уравнением массового расхода и т. д. в одном месте.

Вентилятор представляет собой вращающийся круг, разделенный на лопасти таким образом, что он перемещает воздух с одной стороны круга на другую. Вентилятор создает охлаждающий эффект за счет циркуляции воздуха. Наличие большого количества воздуха на вашем лице на самом деле заставляет вас чувствовать себя прохладнее, так как температура падает.

Тем не менее, температура не падает, и это только то, как мы воспринимаем изменения. Этот эффект аналогичен охлаждению ветром и зависит от нескольких факторов, таких как законы охлаждения, теплопроводность, удельная теплоемкость и потери тепла.

Вентиляторы могут быть разных размеров и форм. Вы можете наблюдать их в своей повседневной жизни, например, вентиляторы для охлаждения компьютера или потолочные вентиляторы, вентиляторы, используемые в больших системах вентиляции для охлаждения помещения и т. д. уродливые, шумные, но мощные и с оптимизированной производительностью. Они обычно используются в горячих местах и ​​холодных помещениях. Поскольку они скрыты, они не служат никакой эстетической цели.

Потолочные вентиляторы — Эти вентиляторы разработаны бесшумно в отличие от их обычной функции циркуляции воздуха и не мешают тем, кто сидит под ними. Они не будут отвлекать внимание от комнаты, в которой находятся.

Вентиляторы для ванных — относятся к категории вентиляции и встречаются редко.

Наружные вентиляторы — похожи на стационарные вентиляторы и могут использоваться, когда вам нужен прохладный ветерок. Они обеспечивают баланс между массовым расходом и уровнем шума.

Вентиляторы с питанием от USB или батареек — обычно используются в компьютерных системах для их охлаждения.

Спортивные болельщики — Не в последнюю очередь это спортивные болельщики, они не так эффективны в циркуляции воздуха и производят много шума.

Physicscalc.Com содержит такие понятия, как трение, ускорение под действием силы тяжести, давление воды, гравитация и многие другие, а также их соответствующие калькуляторы под одной крышей.

Теперь, когда вы знакомы с типами вентиляторов, давайте подробно обсудим технические характеристики, на которые нужно обращать внимание при выборе вентилятора. Они подробно описаны и выглядят следующим образом:

Размер — характеризуется диаметром окружности, покрывающей вентилятор.

RPM (оборотов в минуту) — RPM — это не что иное, как скорость вращения вентилятора.

Форма лопасти — В зависимости от конструкции лопасти вентилятора вы можете оптимизировать ее для воздушного потока или давления воздуха.

Мощность — это не что иное, как мера мощности, потребляемой/вырабатываемой вентилятором и известная как выходная/вводимая мощность. 1 вообще.

Массовый расход определяется как количество материала, проходящего через блок вентилятора за раз. Математическое определение массового расхода дается выражением dm/dt, т.е. производной массы по времени. Единицей измерения скорости потока является кубический фут в минуту или сокращенно CFM. Вам может показаться странным, глядя на единицу измерения, и вам может быть интересно, что кубические футы — это единица измерения объема, а не массы или веса. Однако, если мы знаем плотность вещества, входящего в вентилятор, мы можем легко преобразовать ее из объема в массу.

Для этого вам необходимо знать другую информацию, например, о среде, в которой работает вентилятор, и т. д. Именно тогда на сцену выходит SFCM, а SFCM означает стандартный кубический фут в минуту и ​​является мерой кубического фута в минуту для газа с фиксированной плотностью.

1. Что такое ОВЛХ?

       

CFM означает кубические футы в минуту и ​​является мерой расхода воздуха.

Онлайн решение задач по физике бесплатно: Решение задач по физике. База готовых решений. Решение задачи онлайн.

Бесплатные материалы и Марафон Весь ЕГЭ за 3 дня!

Оглавление:

  • Полезные материалы
  • Онлайн-марафон «Весь ЕГЭ по физике за 3 дня»
  • План марафона
  • Подробно о том, что будет на Марафоне
  • Откуда задачи?
  • Как занимаемся?
  • org/ListItem»> Кто ведет?
  • Что получите?

Что повторить перед ЕГЭ по физике. Полезные материалы и онлайн-марафон

До ЕГЭ по физике 4 дня. Мы собрали для вас полезные материалы, которые помогут освежить и структурировать ваши знания.

ЕГЭ по физике будет совсем не страшен, если у тебя есть наш «Стартовый набор ЕГЭ на сотку»! Там и все формулы на ЕГЭ по физике, и классификатор задач, и полезнейшие ролики по механике, электродинамике, термодинамике!

А завтра, 3 июня, у нас стартует онлайн-марафон, на котором мы повторим весь курс физики на ЕГЭ! На него действует обратная распродажа. Т.е. скидка уменьшается и курс немного дорожает каждый день.

Подробнее про марафон здесь, а ниже в письме полезнейшие бесплатные материалы! Удачи на ЕГЭ!

ПОДРОБНЕЕ ПРО МАРАФОН!

к оглавлению ▴

Полезные материалы

1. Шпаргалка с формулами. Всё, что тебе нужно вызубрить, находится здесь.

2. Таблица ключей для решения задач ЕГЭ по физике. То есть, все задачи ЕГЭ мы разобрали на типажи и описали, какие законы и формулы применять, если, например, шар летит вертикально вниз, а брусок движется по наклонной плоскости. Скачать можно здесь.

3. Видеоразбор пробного ЕГЭ. Посмотри здесь, как нужно решать вторую часть ЕГЭ (ну и немножко первую).

4. Ты не очень разбираешься в термодинамических процессах? Иди сюда!

5. Ты часто ошибаешься в решении задач по механике? У нас ты сможешь разобраться в этой теме значительно лучше, посмотрев видео.

6. Тебя пугает электродинамика? Законы Ома тебе не знакомы? Правило левой руки путаешь с правилом правой руки? Заходи сюда.

7. Качественные задачи для тебя самые трудные? Не вопрос — мы в этом тоже поможем!

8. Ты вообще не знаешь с чего начать? Начни с начала!

к оглавлению ▴

Онлайн-марафон «Весь ЕГЭ по физике за 3 дня»

Сдаешь ЕГЭ по физике? Приходи на наш Супермарафон!

3 дня занятий по 3 часа. С 3 по 5 июня.

С 10:00 до 13:00 по московскому времени.

Каждый день – домашнее задание. На следующий день – разбор и новые задачи.

Повторим весь курс физики!

Все темы – механика, термодинамика, электричество и магнетизм, оптика, квантовая и ядерная физика.

Все задачи – Часть 1 и Часть 2.

Записаться на онлайн-марафон

к оглавлению ▴

План марафона

День 1. Механика и Термодинамика. Решение задач первой и второй части. Разбор первого задания. Контрольная на дом.

День 2. Электродинамика. Решение задач первой и второй части. Разбор контрольной работы. Проверочная работа на дом.

День 3. Оптика и квантовая физика. Решение задач первой и второй части. Разбор проверочной работы.

Заказать!

к оглавлению ▴

Подробно о том, что будет на Марафоне

Повторим все формулы и необходимую теорию.

Покажем, как из кубиков-заготовок собирается решения задач, которые вы никогда не решали!

Поговорим о «ключах» к решению задач по физике.

Что делать, если ты не знаешь, с чего начать решение задачи. Как использовать то или иное условие. И еще – когда не вполне уверен в своем решении. Как проверить – не нарушил ли какой-нибудь физический закон? Все ли условия задачи понял правильно?

Например, если мы в задаче видим слово «гладкая», то это практически всегда означает применение закона сохранения энергии. А если видим фразу «цилиндр с жёсткими стенками», то, придётся использовать законы и свойства изохорного процесса. Приходите на марафон — мы вспомним все сложности и подводные камни ЕГЭ!

И главное. Будем решать задачи. Решим не менее 50 реальных задач ЕГЭ.

Будем учиться правильному оформлению решений Части 2.

Разберем все непонятные моменты.

к оглавлению ▴

Откуда задачи?

Реальные задачи ЕГЭ. Банк заданий ФИПИ.

Выбираем наиболее часто встречающиеся на реальном ЕГЭ.

Как занимаемся?

Работаем на нашей онлайн-платформе.

Она специально разработана программистами ЕГЭ-Студии для качественного онлайн-обучения.

Это не Zoom, где вы видите только голову преподавателя. Это трансляции из нашей ЕГЭ-Студии. Полный эффект присутствия на реальном уроке!

Каждый день мы будем начинать с основных моментов теории.

Решаем задачи – от простых к сложным.

Будут ДЗ и контрольные.

Мы создадим специальную группу в WhatsApp, где каждый из вас сможет задать вопрос по любой задаче ЕГЭ по физике.

Как всегда, будет видеозапись занятий. Если что-то непонятно — можно будет позже посмотреть еще раз и задать вопросы.

Вы можете задавать вопросы или предлагать свое решение задачи! Можно общаться, как в настоящем школьном классе.

Заказать марафон!

к оглавлению ▴

Кто ведет?

Вадим Александрович Муранов, победитель всероссийского конкурса «Учитель года», преподаватель физики с 25-летним опытом работы, автор и ведущий Онлайн-курса подготовки к ЕГЭ в ЕГЭ-Студии.

Что получите?

+ 10 баллов на ЕГЭ по физике. Знания и уверенность на экзамене.

Не пропусти! Запишись на наш Марафон!

Спасибо за то, что пользуйтесь нашими статьями. Информация на странице «Бесплатные материалы и Марафон Весь ЕГЭ за 3 дня!» подготовлена нашими авторами специально, чтобы помочь вам в освоении предмета и подготовке к ЕГЭ и ОГЭ. Чтобы успешно сдать необходимые и поступить в ВУЗ или колледж нужно использовать все инструменты: учеба, контрольные, олимпиады, онлайн-лекции, видеоуроки, сборники заданий. Также вы можете воспользоваться другими материалами из данного раздела.

Публикация обновлена: 08.02.2023

Аделина — Санкт-Петербург,Санкт-петербург : Выпускница ГФМЛ №30, студентка 2 курса немецкого университета, репетитор по физике

  • Linkedin

  • Facebook

  • Messenger

  • WhatsApp

  • Электронная почта

Аделина

  • 5 (2 отзыва)

1е занятие бесплатно

  • ОГЭ по математике
  • Физика
  • ОГЭ по физике

Места проведения курсов

    • Онлайн

О преподавателе

Опыт работы репетитором — 3 года. Занималась с детьми 7-9 классов, веду подготовку к олимпиадам (10 класс). Была куратором группы проекта от CERN Beamline4Schools.Работа «Исследование Черенковского излучения» вошла в ТОП-20 по миру.

О занятиях

  • Все уровни
  • Английский язык
  • Немецкий язык
  • Русский язык

В 2019 году закончила Физико-математический лицей №30 и поступила на Физический факультет СПбГУ. Проучившись один курс и закрыв две сессии, поступила на естественно-научный факультет немецкого университета FAU Erlangen-Nürnberg по направлению физика, где продолжаю обучение в настоящий момент на втором курсе.
Занятия структурированы следующим образом :
1. Повторение пройденного материала (решение задач, разбор вопросов по теории)
2. Объяснение нового материала (в виде презентации)
3. Решение задач на новую тему и/или помощь с выполнением домашнего задания.
Также провожу занятия по подготовке к ОГЭ по физике и математике в 9 классе. Для изучающих немецкий язык возможен разбор и изучение лексики физических терминов на немецком языке.
Занятия провожу исключительно дистанционно (Zoom или Skype), поскольку живу в Германии.

Рекомендации

2

  • Хорошая учительница по физике, знает материал и хорошо его объясняет, не душит

  • Аделина помогала мне справиться с физикой в школе, где профильными предметами являлись математика и как раз физика. Аделина мне очень понятно объясняла материал, если что-то было не усвоено на уроках, а также помогала подготовиться к контрольным работам и зачетам. В результате, в начале года я думала, что мне придётся уйти из школы, а уже в середине года у меня было по физике 4 и 5, за что безумно благодарна))) Я рекомендую Аделину, потому что это человек, который безумно любит этот сложный, но очень интересный предмет, и который умеет всё чётко разложить по полочкам!

  • Посмотреть больше рекомендаций

Тарифы

Похожие профили по подготовке к ОГЭ по математике

  • Посмотреть всех преподавателей по подготовке к ОГЭ по математике

SuperprofИндивидуальные занятияИндивидуальные занятия по подготовке к ОГЭ по математикеиндивидуальные занятия по физикеиндивидуальные занятия по подготовке к ОГЭ по физике

Посмотреть всех преподавателей

Поделиться

  • Копировать

    Ссылка скопирована!

  • Messenger

  • Linkedin

  • Facebook

  • WhatsApp

  • Электронная почта

  • SMS

уровни

Все уровни

Все языки на курсе

Английский язык

Немецкий язык

Русский язык

Отзывы

Все наши отзывы собраны командой Superprof и достоверны на 100%. Они соответствуют реальному опыту учеников.

Рекомендации

Эти рекомендации получены от друзей, коллег и бывших учеников преподавателя

Бесплатное занятие

Этот первый предлагаемый курс позволяет вам узнать своего учителя, чтобы он наилучшим образом соответствовал вашим потребностям.

Отправить отчёт по данному объявлению

Опишите проблему, связанную с этим объявлением

Информация, которая нарушает права пользователейПовторПредмет не соответсвует категории Текст нечитаем Другое

‎Домашнее задание по физике в App Store

Описание

Мы помогаем решать домашние задания, рисуем ответы и шаг за шагом показываем, как их получить. Для занятий по физике на основе триггеров.

Вам нужна помощь в выполнении домашних заданий? У вас проблемы с началом работы? Вы устали от попыток следовать онлайн-видео с инструкциями? Почему нельзя просто поставить задачу в приложение и получить ответ? Мы можем.

В этой версии:
— Краткое руководство
— Калькулятор векторов
— Калькулятор результирующих векторов
— Калькулятор одномерной кинематики
— Калькулятор двумерной кинематики
— Калькулятор законов Ньютона
— Калькулятор импульса
— Калькулятор импульса
— Калькулятор упругого столкновения
Калькулятор упругого столкновения
— Калькулятор крутящего момента
— Калькулятор чистого крутящего момента

С приложениями на
— Диаграммы свободного тела
— Объект на плоской плоскости
— Объект на наклонной плоскости
— Объект относительно вертикальной стены
— Натяжение объекта от 1 троса
— Натяжение объекта от 2 тросов
— Импульс
— Импульс
— При упругих столкновениях — Сохранение импульса
— Упругие столкновения — Сохранение кинетической энергии и импульса
— Объект в свободном падении
— Выстрел снаряда прямо вверх
— Снаряд в движении под углом
— Крутящий момент
— Чистый крутящий момент (луч)
— Чистый крутящий момент (составное колесо)

И мы покажем вам, как мы это сделали.

Кроме того, если ваш учитель бросает вам крученый мяч, мы также предоставляем вам возможность смоделировать фиксированный сценарий по вашему выбору в версии для iPad. При повторении сценария вы можете изменять определенные переменные, включая угол наклона, приложенную силу, угол приложенной силы, массу и коэффициент статического трения. Функции моделирования генерируют пошаговые отчеты для каждого ответа, чтобы вы могли проверить свою работу.

Бесплатная пробная версия не по времени, а по использованию. В частности, каждый раз, когда вы моделируете сценарий, запускаете симуляцию или решаете уравнение кинематики, вы будете использовать одну из двух бесплатных попыток.

После того, как вы завершили бесплатную пробную версию, вы можете получить доступ к приложению с помощью одного из вариантов подписки ниже. Некоторые модули используют более одного раза для решения конечной проблемы, включая Импульс и Столкновения.

Опции: Пользователи могут по своему выбору выбрать следующие проходы.

1) Semester Pass — 5 месяцев полного доступа.

Мелкий шрифт:
— подписка не продлевается и позволяет вам получить доступ ко всем функциям приложения во время активной подписки.
— Оплата будет снята с учетной записи iTunes при подтверждении покупки.
— Срок действия подписки истекает в конце срока. Он НЕ будет автоматически обновляться.
— Отмена текущей подписки не допускается в течение активного периода подписки.
— Любая неиспользованная часть бесплатного пробного периода, если она предлагается, будет аннулирована, когда пользователь приобретет подписку.
— Вы можете управлять своей подпиской, перейдя в настройки App Store

Условия использования: https://v2appdevelopment.com/physics/eula

Политика конфиденциальности: https://v2appdevelopment.com/physics/privacy

Версия 1.6.20

Вот чего ожидать от последней версии:
— обновленный веб-сайт

Рейтинги и обзоры

122 Оценки

Домашнее задание по физике

Отличная работа над диаграммой свободного тела. Я использовал это, чтобы сделать мою летнюю домашнюю физику. Она написана не совсем так, как пишет мой профессор, но все же работает. Мне очень нравится, как можно менять массу и углы.

Спасибо за отзыв. Мы всегда открыты для предложений и хотим сделать функцию «Показать работу» максимально полезной.

Очень полезно

Больше всего я использовал Кинематику и Снаряды, потому что я присоединился в середине летнего класса. Я бы поставил 5, но он мог бы течь немного мягче.

Спасибо за отзыв. Мы рады, что вам нравятся Кинематика и Снаряды. Мы определенно работаем над тем, чтобы сделать его более плавным и ценим ваши отзывы. Пожалуйста, напишите нам по электронной почте, если вы видите что-то еще, что вы хотели бы улучшить? Просто нажмите «Отправить отзыв по электронной почте» в меню.

Мошенничество

Вы должны заплатить, чтобы использовать это. Не теряйте время

Привет. Нам жаль, что вы не верите, что это стоит своих денег! Есть ли что-то еще о вашем опыте, что мы должны знать?

Разработчик, V2 App, LLC, указал, что политика конфиденциальности приложения может включать обработку данных, как описано ниже. Для получения дополнительной информации см. политику конфиденциальности разработчика.

Данные, не связанные с вами

Могут быть собраны следующие данные, но они не связаны с вашей личностью:

  • Идентификаторы
  • Данные об использовании
  • Диагностика

Методы обеспечения конфиденциальности могут различаться, например, в зависимости от используемых вами функций или вашего возраста. Узнать больше

Информация

Продавец
В2 Апп, ООО

Размер
9 megabytes»> 14,9 МБ

Категория
Образование

Возрастной рейтинг
4+

Авторское право
© 2022 V2 App, LLC

Цена
Бесплатно

  • Сайт разработчика
  • Тех. поддержка
  • политика конфиденциальности

Еще от этого разработчика

Вам также может понравиться

LivePhysics

Ваш браузер не поддерживает JavaScript! Некоторые функции не будут работать должным образом…
Астрофизика

Звук песни Земли
Космический корабль НАСА записал жутко звучащее радиоизлучение, исходящее с нашей планеты. По иронии судьбы, эти прекрасные «песни Земли» могут быть ответственны за распространение смертоносного электро в…

Обнаружена редкая система-прародитель сверхновой типа 1a
Многопрофильная команда Palomar Traient Factory (PTF) выпустила непревзойденное прямое наблюдение системы-прародителя сверхновой типа 1a. Астроном собрал доказательства, свидетельствующие о том, что…

Темная материя
Мы смотрим на таинственную темную материю — и шоколадный пирог. Персеиды метеорный поток
Идет метеорный поток Пид. Однако есть на что посмотреть, кроме метео, когда ливень достигает пика с 11 по 13 августа. Самые яркие планеты Солнечной системы выстраиваются посередине…

Классическая механика

Движение снаряда – Выстрел на уровне земли
Мяч бьют с начальной скоростью 25 м/с под углом 45 градусов с горизонталью. Определить время полета, горизонтальное смещение и высоту пика футбольного мяча…..

Движение снаряда – шар для пула покидает стол с начальной горизонтальной скоростью
Шар для пула покидает стол высотой 0,20 м с начальной горизонтальной скоростью 2,4 м/с. Предсказать время, необходимое для того, чтобы шар для пула упал на землю, и расстояние по горизонтали между выступом…

Сила удара по мячу – Импульс
Бейсбольный мяч (m = 0,14 кг), брошенный питчером, достигает биты с скорость 32,0 м/с в горизонтальном направлении. а) Рассчитайте импульс, который нужно приложить к мячу, чтобы он покинул…

Средняя сила удара между авиалайнером и птицей
Оцените среднюю силу удара между авиалайнером, летящим со скоростью 260 м/с, и птицей весом 0,3 кг и длиной 20 см? Изменение скорости птицы оценивается как 260 РС Thes…

Найти угловой момент Земли
Найти угловой момент Земли, используя расстояние Земля-Солнце и массу Земли? Земля — ​​Солнце расстояние 149,6×109 м Масса Земли 5,9742×1024 кгМомент импульса ЗемлиДля орбиты, момент импульса…

Найти скорость спутника, находящегося на геостационарной орбите.
Спутник связи массой 270 кг выведен на геостационарную орбиту на высоте 35 780 км над релейной поверхностью земли. Какова скорость спутника на орбите? Геостационарный о…

Классическая механика

Импульс
Импульс в классической механике Если объект движется в какой-либо системе отсчета, то он имеет импульс в этой системе отсчета. Количество импульса, которым обладает объект, зависит от двух физических величин: th…

Второй закон Ньютона
Второй закон Ньютона: Закон движения Скорость изменения импульса тела равна равнодействующей силы, действующей на тело, и направлена ​​в ту же сторону. Второй закон Ньютона в первоначальном виде…

Кинетика точки в фиксированной системе координат
Определение Положение, скорость и ускорение определяются следующим образом: Для единицы вектора в направлении пер…

Динамика точки в фиксированной системе координат
Сила, (угловой)импульс и энергия Второй закон Ньютона связывает силу, действующую на объект, и результирующее ускорение объекта, где импульс определяется выражением :: Третий закон Ньютона определяется выражением. ..

Динамика точек в движущейся системе координат
Кажущиеся силы Суммарная сила в подвижной системе координат может быть найдена путем вычитания кажущихся сил из сил, действующих в системе отсчета: . Различные кажущиеся силы дают…

Динамика коллекций точек масс
Центр масс Скорость относительно центра масс определяется выражением Координаты центра масс определяются выражением: В двухчастичной системе координаты центра масс…

Классическая механика

Преобразование единиц измерения
Пол обсуждает передачу единиц посредством сокращения, проиллюстрированную простой задачей средней скорости с участием Fast Freda.0005

Задача о велосипедах и пчелах
Пол показывает простое решение классической задачи о движении пчелы, которая летает туда-сюда между приближающимися велосипедами. Barry Biker.MfDv4FMDlpI

Гравитация: четыре фундаментальные силы физики
Хэнк продолжает нашу серию статей о четырех фундаментальных силах физики описанием гравитации — взаимодействия, посредством которого физические тела притягиваются с силой, пропорциональной силе их действия.

Усэйн Болт против гравитации
Кто быстрее на 10 мете — самый быстрый спринтер в мире или гравитация 9YUtFpLpGfk

Инструменты классической механики

Движение снаряда
Инструменты физики для движения снаряда Используйте этот простой инструмент для решения физической задачи, связанной с движением снаряда. Вы можете использовать этот инструмент для любого снаряда, выпущенного на уровне земли под углом тета-де…

Движение снаряда – Снаряд, выпущенный над уровнем земли
Используйте этот простой инструмент для решения физической задачи, связанной с движением снаряда. Вы можете использовать этот инструмент для любого снаряда, выпущенного над уровнем земли с начальной скоростью в горизонтальном направлении. Просто…

Решите физическую задачу, связанную со столкновением авиалайнера и птицы.
Физические инструменты для классической механики. Используйте этот простой инструмент для решения физической задачи, связанной со столкновением авиалайнера и птицы. Используйте этот простой инструмент для решения физической задачи, связанной со столкновением…

Решите задачу, связанную с падением объекта из состояния покоя
Когда объект падает из состояния покоя, его гравитационная потенциальная энергия преобразуется в кинетическую энергию. Назовите массу m, падающую вертикально под действием силы тяжести. Потенциальная энергия…

Решить задачу, связанную с силой удара падающего объекта
Когда объект падает из состояния покоя, его гравитационная потенциальная энергия преобразуется в кинетическую энергию. Придумайте массу m, которая падает вертикально под действием силы тяжести. Объект падает…

Демонстрация

Кот Шредингера
Кот Шредингера — это мысленный эксперимент, который иногда называют парадоксом, разработанный австрийским физиком Эрвином Шредингером. Это иллюстрирует то, что он видел как проблему копенгагенской интерпретации…

Физика пены
Какая связь между жирафами, винными пробками и картой Univee?yLTrD9LYQTs

Как работает бумеранг?
Бумеранг может совершить свой уникальный полет туда и обратно, используя три фундаментальных принципа физики: подъемную силу, относительную скорость и гироскопическую прецессию. Йо-йо в космосе
Астронавт НАСА Дон Петтит использует свое свободное время, чтобы практиковать свои навыки йо-йо в условиях микрогравитации. ni4j5K4Lz3o

Сенсорные экраны и квантовое туннелирование
Магнетизм

Электромагнетизм – электростатическая сила: четыре фундаментальные силы физики
Хэнк достигает четвертого и последнего из четырех фундаментальных взаимодействий в физике: электромагнетизма. В этой части он рассказывает нам об электростатической силе, которая создает заряд в объекте…

Ток и магниты
Профессор Роджер Боули использует плавкую проволоку, магниты и электрический ток для быстрой демонстрации UV0wtX9AXq4

Электромагнетизм – магнитное взаимодействие: четыре фундаментальные физические силы
магнитная сила, второй из двух способов проявления электромагнетизма в univee.cy6kba3A8vY

Напряжение в зависимости от расстояния в однородном электрическом поле
Грег Джейкобс — AP Physics Summer Ititute: Voltage vs Distance in a Uniform Electric Field4Jnb_biFbd8

Криогенные эксперименты с пассивными и активными электронными компонентами
В этом эпизоде ​​Шахриар исследует теорию и экспериментальные результаты воздействия экстремально низких температур на пассивные и активные компоненты. Жидкий азот используется в прозрачном стекле D…

Бутылочный радиоприемник
Кристаллическая радиотехнология существует уже много лет. Этот «бутылочный радиоприемник» похож на кристаллический радиоприемник, не требует источника питания, работает на энергии радиоволн и принимает сигнал от длинного…

Жидкости и термодинамика

Твердый азот, вакуумное охлаждение и сухой лед
Что произойдет, если уменьшить давление вокруг жидкости? Это кипит. Вода закипает при комнатной температуре, когда давление достаточно низкое. Что интересно, это снижает температуру тела…

Воздействие давления воды
Этот ролик иллюстрирует воздействие давления воды на шар, наполненный воздухом. Такое же сжатие происходит с воздухом в человеческом ухе, поэтому мы должны выравнивать его во время фридайвинга….

Теплопроводность
Теплопроводность. Деревянно-металлический пьедестал стоит в горячей воде. Поскольку металл лучше проводит тепло, чем дерево, кубик льда на металлическом пьедестале тает быстрее, чем кубик льда на деревянном пьедестале. ..

Парамагнетизм кислорода
Газообразный кислород переводится в жидкую форму, а затем выливается между полюсами сильный магнит, поэтому мы можем наблюдать его парамагнитные свойства. Мы пропускаем газообразный O2 через медную катушку, которая затем…

Линзы и вихри
Астронавт НАСА Дон Петтит использует невесомость. на борту МКС для проведения диффузии и экспериментов с чистой водой. Распространяется в рамках сотрудничества НАСА и…

Антигравитационный трюк с водой
Легко держать воду в перевернутом контейнере, используя только карточку. Однако только тот, кто обладает исключительным мастерством, может удержать чтобы вода не выпадала при извлечении карты. Вы умеете…

FORTRAN

Fortran Online Resources
Университет Мичигана-Дирборна, компьютерные и информационные науки Этот сайт содержит историю языка Фортран и множество ссылок на полезные сайты. http://www.engin.umd.umich….

Переменные и имена переменных
Основы вычислений на Фортране Переменные и имена переменных Целое число Целое число. Это может…

Операции и выражения
Operatio и Expressio FORTRAN предоставляет пять основных арифметических операций: Дополнение + Вычитание — Умножение * Дивизион

Учебники FORTRAN – Ввод и вывод
Ввод и вывод FORTRAN обеспечивает большую мощность и гибкость при чтении данных и выводе результатов на печать. А для опытного программиста эта мощь и гибкость…

Написание и выполнение программы на FORTRAN
Операторы STOP и END ОСТАНАВЛИВАТЬСЯ Этот оператор может быть написан всякий раз, когда необходимо остановить выполнение операторов в программе. Обычно имеется STOP a…

FORTRAN Оператор DO
Оператор DO Этот оператор позволяет многократно выполнять часть программы с автоматическим изменением значения целочисленной переменной между…

Математические инструменты

Вычисление линейной регрессии и построение диаграммы рассеяния и линии наилучшего соответствия
Что такое простая линейная регрессия? Простая линейная регрессия — это способ описания отношения между двумя переменными с помощью уравнения прямой линии, называется. ..

Интеллектуальная функция построения графиков Google
Google запустил функцию построения графиков прямо в поиске, чтобы помочь учащимся и математикам полюбить функцию построения графиков простым и легким способом. В дополнение к вычислению чего-то простого, например, деление доли…

Google Smart Calculator
Хотите составить список чисел, преобразовать мили в километры или оценить какое-то другое математическое выражение? Вместо использования листа бумаги ваш калькулятор или компьютерная математическая программа…

Побитовая операция
Конвертер десятичных чисел в шестнадцатеричные и шестнадцатеричных в десятичныеОнлайн побитовый калькулятор В компьютерном программировании…

Инструмент преобразования кода Морзе
Код Моэ — это метод передачи телеграфной информации, использование стандартизированных последовательностей коротких и длинных элементов для представления буквы, цифры, знаки препинания и пр…

Преобразование римских цифр с помощью римского калькулятора и римских тестов
Римские цифры — это система счисления, возникшая в Древнем Риме, адаптировано из этрусских цифр. Система, использовавшаяся в классической древности был слегка изменен в середине…

Современная физика

Мичио Каку объясняет теорию струн
Физик-теоретик Митио Каку объясняет основы теории струнkYAdwS5MFjQ

Сильное взаимодействие: четыре фундаментальные физические силы
Первая часть из четырех частей серии фундаментальные силы (или взаимодействия) физики начинаются с сильного взаимодействия или сильного взаимодействия, которое в малом масштабе удерживает кварки вместе, образуя прото…

Что такое антивещество?
Ученый Фермилаборатории Дон Линкольн описывает антивещество и его свойства. Он также объяснил, почему антиматерия, хотя и существует, в настоящее время не представляет угрозы для нашего существования! en2S1tBl1_s

Бозон Хиггса: Как вы ее ищете?
Ученый Фермилаборатории Дон Линкольн описывает концепцию поиска бозона Хиггса. Несколько крупных экспериментальных групп идут по горячим следам этой неуловимой субатомной частицы. ..

Что такое двойственность волны и частицы?
Дуальность волновых частиц и почему квантовая механика более странная, чем все, к чему мы привыкли в нашей повседневной жизни! Q_h5IoPJXZw_riIY-v2Ym8

Нейтроны и нанонаука
Пучки нейтронов можно использовать для важных открытий в мире нанонауки. Здесь мы изучаем три инструмента (и линии луча) на объекте ISIS в Оксфордшире, где Нейтро подвергают воздействию…

Оптика

Наблюдатель, смотрящий через бассейн в точке C, видит световой луч
Наблюдатель, смотрящий через плавательный бассейн бассейн в точке C видит луч света, идущий из точки A на противоположной стороне бассейна внизу. Луч выходит из воды в точке на расстоянии 1,7 метра от противоположной…

Свет с интенсивностью 50 Вт/м2 падает на оконное стекло
Свет с интенсивностью 50 Вт/м2 падает на оконное стекло (n = 1,5) с лучами, приблизительно перпендикулярными поверхности, как показано на рисунке ниже. Найдите бесконечности I1, I2 и I3.

Свет переходит из воздуха в другую среду
Свет переходит из воздуха в другую среду в точке A. Сколько времени требуется лучу света, чтобы пройти из A в B? Используя геометрию, мы можем найти преломление…

Луч света падает на стеклянную пластину под углом 40 градусов
Луч света падает на стеклянную пластину под углом 40°, как показано на рисунке. а) Найдите углы А, В и Г? b) Будет ли какой-либо свет падать на стеклянную пластину со стороны…

Фокусное расстояние и сила собирающей линзы
Ниже на диаграмме показан эскиз файла с маркировкой поверхности в соответствии с их кривизной, если смотреть с левой стороны. , предположим, что это сторона падающего света. Стекло изготовлено из стекл…

Конвергентная линза с различными радиусами
Некоторый тонкий лист из стекла имеет поверхности с радиусами R1 = +10 см и R2 = +8 см. а) Набросать файл? б) Найдите фокусное расстояние? c) Расстояние до объекта на расстоянии 15 см?…

Инструменты для оптики

Закон Снеллса
Используйте этот простой инструмент для решения физических задач, связанных с простыми задачами отражения световых лучей. Вы можете использовать этот инструмент для прохождения лучей света из одной среды в другую. Просто введите известные вам значения…

Найти критический угол
Используйте этот простой инструмент для решения физической задачи, связанной с условием критического угла для отражения. Вы можете использовать этот инструмент для прохождения светового луча из одной среды в другую, чтобы найти критический угол…

Найдите критический угол и условие для полного отражения
Используйте этот простой инструмент для решения физической задачи, связанной с условием критического угла для полного отражения или коэффициента пропускания. Вы можете использовать этот инструмент для перехода светового луча из одной среды в другую с…

Фокусное расстояние для тонкой линзы
Инструменты физики для оптики — Найдите фокусное расстояние тонкого файла с помощью радиусов R1 и R2 Используйте этот простой инструмент для решения физической задачи, связанной с тонким осадком. Вы можете использовать этот инструмент для определения фокусного расстояния для тонких. ..

Инструмент системы с двумя линзами — расстояние до изображения и увеличение
Инструменты физики для оптики — определение расстояния до изображения и увеличения для системы с двумя линзами Используйте этот простой инструмент для решения физической задачи, связанной с двумя файловыми системами. Вы можете использовать этот инструмент, чтобы найти положение…

Физика элементарных частиц

Эксперимент NOvA Neutrino устанавливает первый блок детектора
Промежуток времени нейтринного эксперимента NOvA Fermilab, в ходе которого устанавливаются 28 блоков детекторов в Эш-Ривер, штат Миннесота. Каждый блок имеет размеры 51 x 51 x 7 футов и при сборке будет весить от 500 метрических до .gFpK00WJl9….

Эволюция Национальной ускорительной лаборатории SLAC
За свою 50-летнюю историю Национальная ускорительная лаборатория SLAC превратилась из новаторского исследовательского центра физики элементарных частиц в одну из передовых многоцелевых лабораторий в мире. ..

Гаргамель и Neutral Currents
Пыль (и пиво) собирает под деревом в ЦЕРНе — это Гаргамель. Этот эксперимент сыграл ключевую роль в исследованиях слабого взаимодействия, получивших Нобелевскую премию. Теперь он выставлен на всеобщее обозрениеbasLNz6frO8…

Резерфордское рассеяние – демонстрация в классе
Резерфордское рассеяние: простой способ демонстрации обратного рассеяния в классе.5ovtgA6GbxU

Бозон Хиггса: как обнаружить частицу частица? Что мы подразумеваем под «открытием»?0184

Плотность обычных веществ
Божество определяется как масса, деленная на объем, или масса на единицу объема. Божество в килограммах на кубический метр можно получить, умножив табличные значения на 1000. Божества большинства чистых…

Вязкость жидкостей и газов
Вязкость измеряется в системе СИ Паскаль-секунды (Па·с), которая называется Пуазейлем. Чаще используется дина-сек/см2, которую называют Пуазом. Один Па с равен 10 пуазам. Баланс используется в таблице, потому что. ..

Сжимаемость жидкостей
Сжимаемость – это относительное изменение объема на единицу увеличения давления. При увеличении давления на каждую атмосферу объем воды уменьшится на 46,4 частей на миллион. Сжимаемость…

Физические свойства некоторых типичных жидкостей
Свойство Аргон Бензол Твердый Жидкость Твердый Жидкость Божество (кг/м3) 1636 1407 100…

Угловая скорость Земли
Средняя угловая скорость Wav — это средняя скорость изменения углового смещения. Угловая скорость Земли равна котанту (или почти котанту). Поэтому мы можем рассчитать средний угол…

Момент инерции однородных объектов
Объект Ось вращения Момент инерции Твердый диск Центральная ось диска…

Теории

Магнетизм животных: как животные перемещаются
Хэнк рассказывает нам о новом исследовании вопроса о том, как животные перемещаются с места на место — хотя проблема все еще не решена, у нас есть несколько гипотез, и все они связаны с чем-то вроде.

В физике формула g: Ускорение свободного падения — урок. Физика, 9 класс.

Сила тяготения — материалы для подготовки к ЕГЭ по Физике

 

Автор — профессиональный репетитор, автор учебных пособий для подготовки к ЕГЭ Игорь Вячеславович Яковлев

Темы кодификатора ЕГЭ: силы в механике, закон всемирного тяготения, сила тяжести, ускорение свободного падения, вес тела, невесомость, искусственные спутники Земли.

Любые два тела притягиваются друг к другу — по той лишь одной причине, что они имеют массу. Эта сила притяжения называется силой тяготения или гравитационной силой.

Закон всемирного тяготения.

 

Гравитационное взаимодействие любых двух тел во Вселенной подчиняется достаточно простому закону.

Закон всемирного тяготения. Две материальные точки массами и притягиваются друг к другу с силой, прямо пропорциональной их массам и обратно пропорциональной квадрату расстояния между ними:

(1)

Коэффициент пропорциональности называется гравитационной постоянной. Это фундаментальная константа, и её численное значение было определено на основе эксперимента Генри Кавендиша:

Порядок величины гравитационной постоянной объясняет, почему мы не замечаем взаимного притяжения окружающих нас предметов: гравитационные силы оказываются слишком малыми при небольших массах тел. Мы наблюдаем лишь притяжение предметов к Земле, масса которой примерно кг.

Формула (1), будучи справедливой для материальных точек, перестаёт быть верной, если размерами тел пренебречь нельзя. Имеются, однако, два важных для практики исключения.

1. Формула (1) справедлива, если тела являются однородными шарами. Тогда — расстояние между их центрами. Сила притяжения направлена вдоль прямой, соединяющей центры шаров.

2. Формула (1) справедлива, если одно из тел — однородный шар, а другое — материальная точка, находящаяся вне шара. Тогда сстояние от точки до центра шара. Сила притяжения направлена вдоль прямой, соединяющей точку с центром шара.

Второй случай особенно важен, так как позволяет применять формулу (1) для силы притяжения тела (например, искусственного спутника) к планете.

Сила тяжести.

 

Предположим, что тело находится вблизи некоторой планеты. Сила тяжести — это сила гравитационного притяжения, действующая на тело со стороны планеты. В подавляющем большинстве случаев сила тяжести — это сила притяжения к Земле.

Пусть тело массы лежит на поверхности Земли. На тело действует сила тяжести , где — ускорение свободного падения вблизи поверхности Земли. С другой стороны, считая Землю однородным шаром, можно выразить силу тяжести по закону всемирного тяготения:

,

где — масса Земли, км — радиус Земли. Отсюда получаем формулу для ускорения свободного падения на поверхности Земли:

. (2)

Эта же формула, разумеется, позволяет найти ускорение свободного падения на поверхности любой планеты массы и радиуса .

Если тело находится на высоте над поверхностью планеты, то для силы тяжести получаем:

.

Здесь — ускорение свободного падения на высоте :

.

В последнем равенстве мы воспользовались соотношением

которое следует из формулы (2).

Вес тела. Невесомость.

 

Рассмотрим тело, находящееся в поле силы тяжести. Предположим, что есть опора или подвес, препятствующие свободному падению тела. Вес тела — это сила, с которой тело действует на опору или подвес. Подчеркнём, что вес приложен не к телу, а к опоре (подвесу).

Рис. 1. Сила тяжести, реакция опоры и вес тела

 

На рис. 1 изображено тело на опоре. Со стороны Земли на тело действует сила тяжести (в случае однородного тела простой формы сила тяжести приложена в центре симметрии тела). Со стороны опоры на тело действует сила упругости (так называемая реакция опоры). На опору со стороны тела действует сила — вес тела. По третьему закону Ньютона силы и равны по модулю и противоположны по направлению.

Предположим, что тело покоится. Тогда равнодействующая сил, приложенных к телу, равна нулю. Имеем:

С учётом равенства получаем . Стало быть, если тело покоится, то его вес равен по модулю силе тяжести.

Задача. Тело массы вместе с опорой движется с ускорением , направленным вертикально вверх. Найти вес тела.

Решение. Направим ось вертикально вверх (рис. 2).

Рис. 2. Вес тела больше силы тяжести.

 

Запишем второй закон Ньютона:

Перейдём к проекциям на ось :

.

Отсюда . Следовательно, вес тела

.

Как видим, вес тела больше силы тяжести. Такое состояние называется перегрузкой.

Задача. Тело массы вместе с опорой движется с ускорением , направленным вертикально вниз. Найти вес тела.

Решение. Направим ось вертикально вниз (рис. 3).

Рис. 3. Вес тела меньше силы тяжести.

 

Схема решения та же. Начинаем со второго закона Ньютона:

Переходим к проекциям на ось :

.

Отсюда c. Следовательно, вес тела

.

В данном случае вес тела меньше силы тяжести. При (свободное падение тела с опорой) вес тела обращается в нуль. Это — состояние
невесомости, при котором тело вообще не давит на опору.

Искусственные спутники.

 

Для того, чтобы искусственный спутник мог совершать орбитальное движение вокруг планеты, ему нужно сообщить определённую скорость. Найдём скорость кругового движения спутника на высоте над поверхностью планеты. Масса планеты , её радиус (рис. 4)

Рис. 4. Спутник на круговой орбите.

 

Спутник будет двигаться под действием единственной силы — силы всемирного тяготения, направленной к центру планеты. Туда же направлено и ускорение спутника — центростремительное ускорение

.

Обозначив через массу спутника, запишем второй закон Ньютона в проекции на ось, направленной к центру планеты: , или

.

Отсюда получаем выражение для скорости:

.

Первая космическая скорость — это максимальная скорость кругового движения спутника, отвечающая высоте . Для первой космической скорости имеем

,

или, с учётом формулы ( 2),

.

Для Земли приближённо имеем:

км/с.

 

Благодарим за то, что пользуйтесь нашими публикациями. Информация на странице «Сила тяготения.» подготовлена нашими редакторами специально, чтобы помочь вам в освоении предмета и подготовке к ЕГЭ и ОГЭ. Чтобы успешно сдать необходимые и поступить в высшее учебное заведение или колледж нужно использовать все инструменты: учеба, контрольные, олимпиады, онлайн-лекции, видеоуроки, сборники заданий. Также вы можете воспользоваться другими материалами из данного раздела.

Публикация обновлена: 08.03.2023

Открытая Физика. Свободное падение тел

Свободное падение тел

Свободным падением тел называют падение тел на Землю в отсутствие сопротивления воздуха (в пустоте). В конце XVI века знаменитый итальянский ученый Г. Галилей опытным путем с доступной для того времени точностью установил, что в отсутствие сопротивления воздуха все тела падают на Землю равноускоренно, и что в данной точке Земли ускорение всех тел при падении одно и то же. До этого в течение почти двух тысяч лет, начиная с Аристотеля, в науке было принято считать, что тяжелые тела падают на Землю быстрее легких.

Ускорение, с которым падают на Землю тела, называется ускорением свободного падения. Вектор ускорения свободного падения обозначается символом g→, он направлен по вертикали вниз. В различных точках земного шара в зависимости от географической широты и высоты над уровнем моря числовое значение g оказывается неодинаковым, изменяясь примерно от 9,83 м/с2 на полюсах до 9,78 м/с2 на экваторе. На широте Москвы g = 9,81523 м/с2. Обычно, если в расчетах не требуется высокая точность, то числовое значение g у поверхности Земли принимают равным 9,8 м/с2 или даже 10 м/с2.

Простым примером свободного падения является падение тела с некоторой высоты h без начальной скорости. Свободное падение является прямолинейным движением с постоянным ускорением. Если направить координатную ось OY вертикально вверх, совместив начало координат с поверхностью Земли, то для анализа свободного падения без начальной скорости можно использовать формулу (*) §1.4, положив υ0 = 0, y0 = h, a = –g. Обратим внимание на то, что если тело при падении оказалось в точке с координатой y < h, то перемещение s тела равно s = y – h < 0. Эта величина отрицательна, так как тело при падении перемещалось навстречу выбранному положительному направлению оси OY. В результате получим: υ = –gt.

Скорость отрицательна, так как вектор скорости направлен вниз. y=h-gt22.

Время падения tп тела на Землю найдется из условия y = 0: tп=2hg.

Скорость тела в любой точке составляет: υ=2g(h-y).

В частности, при y = 0 скорость υп падения тела на Землю равна υп=2gh.

Пользуясь этими формулами, можно вычислить время падения тела с данной высоты, скорость падения тела в любой момент после начала падения и в любой точке его траектории и т. д.

Аналогичным образом решается задача о движении тела, брошенного вертикально вверх с некоторой начальной скоростью υ0. Если ось OY по-прежнему направлена вертикально вверх, а ее начало совмещено с точкой бросания, то в формулах равноускоренного прямолинейного движения следует положить: y0 = 0, υ0 > 0, a = –g. Это дает: υ = υ0 – gt.

Через время υ0 / g скорость тела υ обращается в нуль, т.  е. тело достигает высшей точки подъема. Зависимость координаты y от времени t выражается формулой y=υ0t-gt22.

Тело возвращается на землю (y = 0) через время 0 / g, следовательно, время подъема и время падения одинаковы. Во время падения на землю скорость тела равна –υ0, т. е. тело падает на землю с такой же по модулю скоростью, с какой оно было брошено вверх.

Максимальная высота подъема h=ymax=υ022g.

Графики скоростей для различных режимов движения тела с ускорением a = –g

На рис. 1.5.1 представлены графики скоростей для трех случаев движения тела с ускорением a = –g. График I соответствует случаю свободного падения тела без начальной скорости с некоторой высоты h. Падение происходило в течение времени tп = 1 с. Из формул для свободного падения легко получить: h = 5 м (все числа в этих примерах округлены, ускорение свободного падения принято равным 10 м/с2).

График II – случай движения тела, брошенного вертикально вверх с начальной скоростью υ0 = 10 м/с. Максимальная высота подъема h = 5 м. Тело возвращается на землю через время t = 2 с.

График III – продолжение графика I. Свободно падающее тело при ударе о землю отскакивает (мячик), и его скорость за очень короткое время меняет знак на противоположный. Дальнейшее движение тела не отличается от случая II.

Задача о свободном падении тел тесно связана с задачей о движении тела, брошенного под некоторым углом к горизонту. Для кинематического описания движения тела удобно одну из осей системы координат (ось OY) направить вертикально вверх, а другую (ось OX) – расположить горизонтально. Тогда движение тела по криволинейной траектории можно представить как сумму двух движений, протекающих независимо друг от друга – движения с ускорением свободного падения вдоль оси OY и равномерного прямолинейного движения вдоль оси OX. На рис. 1.5.2 изображен вектор начальной скорости υ→0 тела и его проекции на координатные оси.

Движение тела, брошенного под углом α к горизонту. Разложение вектора υ→0 начальной скорости тела по координатным осям

Таким образом, для движения вдоль оси OX имеем следующие условия: x0 = 0, υ0x = υ0 cos α, ax = 0, а для движения вдоль оси OY y0 = 0, υ0y = υ0 sin α, ay = –g.

Приведем здесь некоторые формулы, описывающие движение тела, брошенного под углом α к горизонту.

Время полета: t=2υ0sinαg.

Дальность полета: L=υ02sin2αg;  L=Lmax=υ02g  при  α=45ˆ.

Максимальная высота подъема: h=υ02 sin2 α2g.

Движение тела, брошенного под углом к горизонту

Движение тела, брошенного под углом к горизонту, происходит по параболической траектории. В реальных условиях такое движение может быть в значительной степени искажено из-за сопротивления воздуха, которое может во много раз уменьшить дальность полета тела.





учет резерва отпусков здесь
time-off.ru

Смотрите также: Математика, Английский язык, Химия, Биология, Физика, География, Астрономия.
А также: библиотека ЭОРов и образовательный онлайн-сервис с тысячами интерактивных работ «Облако знаний».

Ускорение под действием гравитации Учебное пособие

Инструменты для творчества скоро появятся, чтобы вдохновить!

Присоединяйтесь к списку рассылки, чтобы узнать, когда мы запустимся.

Физика

Общая физика

Движение в одном измерении

Ускорение под действием силы тяжести Учебное пособие

Кристалл

HS-PS2-1

Все тела на Земле сталкиваются с постоянной силой тяжести и ускорением, обусловленным гравитацией.

Содержание

Что вы ожидаете, когда одновременно роняете перо и кирпич? Ответ заключается в том, что все они упадут одновременно; хотя некоторые объекты, например перья, кажутся медленнее из-за сопротивления воздуха. Для того, чтобы увидеть истинную природу гравитации, влияющей на перья, нужно удалить из помещения весь воздух. Причина этого в том, что в вакууме оба будут сталкиваться с одинаковым ускорением из-за гравитации!

Источник

УСКОРЕНИЕ И ТЯЖЕСТИ

  • Значение ускорение в основном любой процесс, в котором скорость (которая является мерой скорости и направления движения тела) изменяется со временем.

  • Математически представленный как dv/dt (изменение скорости/периода времени), это векторная величина , которая может быть положительной, отрицательной или нулевой в зависимости от скорости и ее направления .

  • Значение гравитации , с другой стороны, это сила, которая притягивает объект к центру земли.

  • F= мг — сила тяжести, действующая на тело,

, где f — сила, действующая на тело,

m — его масса,

и g — ускорение свободного падения.

УСКОРЕНИЕ СИЛЫ ТЯЖЕСТИ

  • Чем дальше объект опускается вперед к низу, тем больше гравитация заставляет его падать все быстрее и быстрее.
  • На практике скорость объекта увеличивается на 9,8 м/с2 (значение ускорения из-за силы тяжести на Земле).
  • Следовательно, через 1 секунду после начала падения скорость тела составляет 9,8 м/с.
  • Его скорость будет продолжать увеличиваться со временем из-за ускорения гравитации .
  • Единица СИ для ускорения свободного падения такая же, как и для ускорения, м/с2 .

Источник

Универсальный закон всемирного тяготения гласит, что f = GmM/(r+h)2

Где f обозначает силу между двумя телами

G (6,6710-11 Нм2/кг2) универсальная гравитационная постоянная

м масса объекта0002 r — радиус Земли, а

h = расстояние между телом и поверхностью земли.

Поскольку высота будет намного меньше по сравнению с радиусом Земли

f = GmM/r2

Сравнивая оба уравнения, мы получаем

g = GM/r2 , формула для ускорение силы тяжести.

Это позволяет нам понять следующее:

  • Гравитация ускоряет все тела с одинаковой скоростью, независимо от их массы.
  • Его ценность на Земле определяется массой планеты, а не массой предмета.
  • Ускорение — это процесс, при котором скорость тела изменяется со временем.
  • Гравитация — это сила, которая притягивает объект к центру Земли.
  • Величина ускорения силы тяжести на земле составляет 9,8 м/с2.
  • g = GM/r2 — уравнение, используемое для расчета ускорения свободного падения.

Часто задаваемые вопросы

1. От чего не зависит ускорение силы тяжести?

Ускорение свободного падения не зависит от массы тела.

2. Какова единица ускорения свободного падения?

Единица СИ для ускорения свободного падения такая же, как и для ускорения, м/с2.

3. Как рассчитать ускорение свободного падения?

Мы можем рассчитать ускорение свободного падения с помощью выражения g = GM/r2.

4. Каково уравнение для ускорения свободного падения?

g = GM/r2 — уравнение, используемое для ускорения под действием силы тяжести

Мы надеемся, что вам понравился этот урок, и вы узнали что-то интересное о Ускорение под действием силы тяжести! Присоединяйтесь к нашему сообществу Discord, чтобы получить ответы на любые вопросы и пообщаться с другими учениками, такими же, как и вы! Обещаем, учиться будет намного веселее!😎

ССЫЛКА

  1. Ускорение под действием силы тяжести: https://www.ck12.org/physics/acceleration-due-to-gravity/lesson/Acceleration-Due-to-Gravity-MS-PS/?referrer=concept_details. По состоянию на 8 апреля 2022 г.
  2. Ускорение за счет гравитации: https://www.vedantu.com/iit-jee/acceleration-due-to-gravity, по состоянию на 8 апреля 2022 г.

Гравитационное ускорение: значение и формула| StudySmarter

Стоя в \(24\) милях над Землей, австрийский смельчак Феликс Баумгартнер собирался совершить то, о чем люди едва ли могли даже мечтать: прыжок в космос. Гравитационное притяжение Земли заставляет объекты непрерывно ускоряться с приблизительно постоянной скоростью по мере их падения. Зная это, 14 октября 2012 года Феликс наклонился вперед и позволил гравитации вытащить себя из безопасности космического корабля, в котором он находился.0003

Рис. 1 — Феликс Баумгартнер собирается начать свое космическое погружение. Как только он наклонится вперед, пути назад уже не будет!

Обычно его замедляет сопротивление воздуха. Но Феликс находился так высоко над Землей, что сопротивление воздуха оказывало слишком малое влияние, и поэтому он находился в полном свободном падении. Прежде чем раскрыть свой парашют, Феликс преодолел звуковой барьер, а также установил многочисленные мировые рекорды. В этой статье мы обсудим, что заставило Феликса достичь такой скорости — гравитационное ускорение: его значение, формула, единицы измерения и расчет, — а также рассмотрим некоторые примеры гравитационного ускорения.

Значение гравитационного ускорения

Говорят, что объект, испытывающий только гравитационное ускорение, находится в состоянии свободного падения .

Гравитационное ускорение — это ускорение, которое испытывает объект, когда гравитация является единственной силой, действующей на него.

Независимо от массы или состава все тела в вакууме ускоряются с одинаковой скоростью. Это означает, что если бы не было трения о воздух, любые два тела, падающие с одной и той же высоты, всегда одновременно достигали бы пола. Но насколько велико это ускорение? Ну, это зависит от величины силы, с которой нас притягивает Земля.

Величина силы, с которой Земля воздействует на нас в фиксированном месте на поверхности, определяется совместным действием гравитации и центробежной силы, вызванной вращением Земли. Но на обычных высотах мы можем пренебречь вкладами последних, так как они пренебрежимо малы по сравнению с гравитационной силой. Поэтому мы сосредоточимся только на гравитационной силе.

Силу гравитации у поверхности Земли можно считать примерно постоянной. Это потому, что он слишком мало меняется для нормальных высот, которые слишком малы по сравнению с радиусом Земли. Вот почему мы часто говорим, что объекты на Земле падают с постоянным ускорением. 92}\) является общепринятым стандартным значением. Области, где это значение значительно отличается, известны как g аномалии равити.

Формула гравитационного ускорения

Согласно закону всемирного тяготения Ньютона между любыми двумя массами существует гравитационное притяжение, направленное на приведение двух масс друг к другу. 2}\\$$ 92\,kg}}\) , а \(r\) — расстояние между центрами масс тел. Как видим, сила тяжести прямо пропорциональна произведению масс и обратно пропорциональна квадрату расстояния между их центрами масс. Когда мы говорим о такой планете, как Земля, притягивающей обычный объект, мы часто ссылаемся на гравитационную силу как на вес этого объекта.

Вес объекта — это гравитационная сила, с которой астрономический объект действует на него.

Вы могли заметить, что мы часто рассчитываем величину веса \(W,\) объекта на Земле по формуле:

$$W= mg,$$

где \(m \) масса объекта и \(g\) обычно называют ускорением силы тяжести на Земле. Но откуда берется это значение?

Мы знаем, что вес тела есть не что иное, как гравитационная сила, действующая на него со стороны Земли. Итак, давайте сравним эти силы:

\begin{aligned} W&=m\textcolor{#00b692}} \\ \end{aligned}

Если мы идентифицируем \( g\) как \( \frac{GM_\text{E}}{r_\text{E}} \), мы получаем сокращение для вычисления гравитационная сила, действующая на объект — его вес — просто как \(w=mg\). Это настолько полезно, что мы определяем физическую величину, относящуюся именно к ней: напряженность гравитационного поля.

Напряженность гравитационного поля астрономического объекта в точке определяется как вектор с величиной

$$ |\vec{g}| = \frac{|\vec{F}_g|}{m}$$

Направление этого вектора указывает на центр масс объекта.

А теперь вам может быть интересно, почему мы называем это «ускорением Земли»? Если вес является единственной силой, действующей на наш объект, закон секунды Ньютауна говорит нам, что

\begin{aligned} ma &= F\\ma &= w\\ ma &= mg\\ a &= g.\end {выровнено}

ускорение объекта равно величине напряженности гравитационного поля, независимо от массы объекта! Вот почему мы рассчитываем ускорение свободного падения или гравитационное ускорение Земли как 92},$$

, так как числовое значение одинаковое, это просто концептуальная разница.

Обратите внимание, что гравитационное ускорение Земли зависит только от массы и радиуса Земли (поскольку мы рассматриваем объект как находящийся на поверхности Земли). Однако здесь есть оговорка. Земля не идеально сферическая! Его радиус меняется в зависимости от того, где мы находимся. Из-за формы Земли значение гравитационного ускорения на полюсах отличается от на экваторе. В то время как сила тяжести на экваторе составляет около \(92.}\end{align*}$$

Как мы видим, когда расстояние настолько велико, что становится значительным по сравнению с радиусом Земли, ускорение силы тяжести уже нельзя считать постоянным, поскольку оно уменьшается заметно.

Примеры гравитационного ускорения

В приведенном выше примере мы видели, что по мере увеличения высоты значение гравитации уменьшается. Когда мы смотрим на график ниже, мы видим, как именно он меняется. Обратите внимание, что это не линейная зависимость. Это ожидается из нашего уравнения, поскольку гравитация обратно пропорциональна квадрат дистанции.

Рис. 3 — Это график гравитационного ускорения в зависимости от высоты. С увеличением высоты значение силы тяжести уменьшается. 2}\) Sun \(274.1\) Mercury \(3.703\) Venus \(8.872\) Mars \(3.72\) Jupiter \ (25,9 \) Уран \ (9,01 \)

Acceleration -Acceleration -Key Takeaewease

7 7003. . . 7. . Acceleration

Acceleration -Key Takeaewease

777777779 гг. сила, действующая на него. 92}.\)
  • С увеличением высоты гравитация уменьшается. Этот эффект заметен для высот, которыми нельзя пренебречь по сравнению с радиусом Земли.
  • Объект, испытывающий только гравитационное ускорение, называется свободным падением .
  • Все объекты падают с одинаковой скоростью в свободном падении.
  • Когда вес является единственной силой, действующей на объект, его ускорение равно величине напряженности гравитационного поля, но в \( \mathrm{\frac{m}{s}}.
  • Олимпиадные задания по физике за 8 класс: Всероссийская олимпиада по физике, задания

    Физика 8. Программа подготовки к олимпиадам

    Объём представленного здесь материала весьма велик и является верхним пределом, к которому хочется стремиться. С каждым конкретным учеником некоторые разделы данной программы обычно удаётся проработать лишь частично.

    Программа согласована с программой олимпиад. В первом полугодии мы занимаемся механикой (Тема Х) и тепловыми явлениями (Темы 1 — 6). Именно эти темы вас ждут на школьном и муниципальном этапах Всероссийской олимпиады (осень), на региональном этапе олимпиады Максвелла (январь), на заключительном этапе Московской олимпиады школьников по физике (февраль).

    С января по март мы занимаемся электричеством (Темы 7 — 10), готовясь к заключительным этапам «Росатома» и «Курчатова». Там уже встречаются задачи на расчёт электрических цепей, вычисление сопротивлений и мощность тока. На очень высоком уровне электричество понадобится тем восьмиклассникам, кто пройдёт на заключительный этап олимпиады Максвелла.

    В апреле и мае у нас оптика. Олимпиадный сезон позади, готовимся к 9-му классу.

    В каком объёме заниматься каждой темой — я решаю с каждым учеником индивидуально.

    Ссылки, даваемые в теории на уроки, ведут на лекции Павла Виктора.

    Тема 1. Атомы и молекулы

    Теория:

    • Основные положения МКТ
    • Газы, жидкости и твёрдые тела
    • Уроки 9, 11 — 14

    Задачи:

    • Атомы и молекулы. 1
    • Атомы и молекулы. 2

    Тема 2. Количество теплоты

    Теория:

    • Внутренняя энергия
    • Количество теплоты
    • ЗФТШ
    • Уроки 98 — 111

    Задачи:

    • Количество теплоты. 1
    • Количество теплоты. 2
    • Количество теплоты. 3

    Тема 3. Уравнение теплового баланса

    Теория:

    • Количество теплоты
    • ЗФТШ
    • Уроки 112 — 118

    Задачи:

    • Теплообмен. 1
    • Теплообмен. 2
    • Теплообмен. 3
    • Теплообмен. 4
    • Теплообмен. 5

    Тема 4. Фазовые переходы

    Теория:

    • Фазовые переходы
    • ЗФТШ
    • Уроки 119 — 129

    Задачи:

    • Фазовые переходы. 1
    • Фазовые переходы. 2
    • Фазовые переходы. 3
    • Фазовые переходы. 4
    • Фазовые переходы. 5
    • Фазовые переходы. 6
    • Фазовые переходы. 7
    • Фазовые переходы. 8

    Тема 5. Тепловые двигатели

    Теория: Уроки 130 — 134.

    Задачи:

    • КПД теплового двигателя

    Тема 6. Теплопроводность

    Задачи:

    • Теплопроводность. 1
    • Теплопроводность. 2
    • Теплопроводность. 3
    • Теплопроводность. 4
    • Теплопроводность. 5

    Тема 7. Электрические цепи

    Теория:

    • Постоянный электрический ток
    • Закон Ома
    • Соединения проводников
    • ЗФТШ
    • Уроки 135 — 152

    Задачи:

    • Электрические цепи. 1
    • Электрические цепи. 2
    • Электрические цепи. 3
    • Электрические цепи. 4
    • Электрические цепи. 5

    Тема 8. Вычисление сопротивлений

    Теория:

    • Соединения проводников
    • ЗФТШ
    • Уроки 153 — 162

    Задачи:

    • Вычисление сопротивлений. 1
    • Вычисление сопротивлений. 2
    • Вычисление сопротивлений. 3
    • Вычисление сопротивлений. 4
    • Вычисление сопротивлений. 5

    Тема 9.

    Работа и мощность тока

    Теория:

    • Работа и мощность тока
    • ЗФТШ
    • Уроки 163 — 169

    Задачи:

    • Мощность тока. 1
    • Мощность тока. 2
    • Мощность тока. 3
    • Мощность тока. 4
    • Электронагреватель. 1
    • Электронагреватель. 2
    • Электронагреватель. 3
    • Электронагреватель. 4

    Тема 10. Нелинейные элементы

    Задачи:

    • Вольт-амперная характеристика
    • Нелинейные элементы. 1
    • Нелинейные элементы. 2
    • Нелинейные элементы. 3
    • Идеальный диод
    • Неидеальный диод. 1
    • Неидеальный диод. 2

    Тема 11. Отражение и преломление света

    Теория:

    • Световые лучи
    • Отражение света
    • Преломление света
    • ЗФТШ

    Задачи:

    • Световые лучи. 1
    • Световые лучи. 2
    • Плоское зеркало. 1
    • Плоское зеркало. 2
    • Закон преломления. 1
    • Закон преломления. 2
    • Закон преломления. 3
    • Закон преломления. 4
    • Полное отражение. 1
    • Полное отражение. 2

    Тема 12. Тонкие линзы

    Теория:

    • Линзы. Ход лучей
    • Тонкие линзы. Ход лучей
    • Тонкие линзы. Построение изображений
    • ЗФТШ

    Задачи:

    • Формула линзы. 1
    • Формула линзы. 2
    • Формула линзы. 3
    • Формула линзы. 4
    • Продольное увеличение. 1
    • Продольное увеличение. 2

    Тема X. Механика и прочее

    Задачи:

    • Величины и процессы. 1
    • Величины и процессы. 2
    • Путь, скорость, время. 1
    • Путь, скорость, время. 2
    • Путь, скорость, время. 3
    • Средняя скорость. 1
    • Средняя скорость. 2
    • Графики движения. 1
    • Графики движения. 2
    • Путь как площадь. 1
    • Путь как площадь. 2
    • Сложение скоростей
    • Круговое движение
    • Туда-сюда
    • Движение по реке (СО воды)
    • Плотность. 1
    • Плотность. 2
    • Средняя плотность
    • Поверхностная и линейная плотность
    • Скорость заполнения
    • Метеорология и пробки
    • Силы
    • Вес тела
    • Соединение пружин
    • Давление
    • Давление жидкости
    • Сообщающиеся сосуды. 1
    • Сообщающиеся сосуды. 2
    • Эврика!
    • Сила Архимеда. 1
    • Сила Архимеда. 2
    • Сила Архимеда. 3
    • Плавание тел. 1
    • Плавание тел. 2
    • Плавание тел. 3
    • Комбинированные задачи по гидростатике. 1
    • Комбинированные задачи по гидростатике. 2
    • Комбинированные задачи по гидростатике. 3
    • Работа и энергия. 1
    • Работа и энергия. 2
    • Равновесие рычага. 1
    • Равновесие рычага. 2
    • Блоки
    • Равновесие тел. 1
    • Равновесие тел. 2
    • Равновесие тел. 3
    • Равновесие тел. 4
    • Задачи в буквах
    • Линейная зависимость
    • Неравенства в физике
    • Лампочки
    • Комбинаторика и графы
    • Малые шевеления
    • Анализ графиков
    • Неравенство Коши в физических задачах

    Архивы задач 2009-2018 — Всероссийская олимпиада по физике. Олимпиада Максвелла.

    • > Главная
    • > Новости и события
    • > Всероссийская олимпиада
      • > Этапы Всероссийской олимпиады
      • > Ближайшие этапы
      • > Результаты
    • > Центры подготовки
    • > Архивы задач
      • > Архивы задач 2019-2028
      • > Архивы задач 2009-2018
      • > Архивы задач 2000-2008
    • > Сборники и сайты олимпиадных задач
    • > Документы

    Сборники олимпиадных задач под редакцией М. Ю.Замятнина для 7, 8 и 9 класса можно приобрести в магазине МФТИ (1 этаж) и в интернет магазинах Физматкнига и Карандаш

     (Решения для международных олимпиад — скорее исключение, чем правило. В большинстве случаев есть только условия.)

     

    Год

    Муниципальный

    этап

    Региональный этап

     

      Финал России      

     

    Азиатская

    APhO

    Международная  

    IPhO

    Естественнонаучная

    IJSO

    2018

     

    LIII

     

     

     

    LII

     

     

     Тюмень

     

     

    7-8 класс, Сочи

     

     

     

     

     

    Вьетнам

     

     

     

     

     

    Португалия

     

     

     

     Ботсвана

     

    2017

     

     

    LII

     

     

    LI

     

     

    Казань

     

     

    7-8 класс, Сочи

     
     

     

     

     Россия, Якутск

     

     

       Индонезия

     

     

     Голландия

     

     

    2016

     

     

      

     LI

     

     

     

      L

     

     

    Сочи

     

     

    7-8 класс, Сочи

     

     

     

     Гонконг

     

     

     Швейцария

     

     

     Индонезия

     

     

    2015

     

     

     

     L

     

     

     

     XLIX

     

     

    Екатеринбург

     

     

    Китай

     

     

    Индия

     

     

    Южная Корея

     

    2014

     

     

     

     XLIX

     

     

     

     XLVIII

     

     

    Санкт-Петербург

     

     

    Сингапур

     

     

     Казахстан

      

     

    Аргентина

     

    2013

     

     

     XLVIII

     

     

     XLVII

     

     

    Владивосток

     

     

    Индонезия

     

     

    Дания

     

     

    Индия

     

    2012

     

     

     XLVI

     

     

    XLVI

     

     

    Саранск

     

     

    Индия

     

     

    Эстония

     

     

    Иран

     

    2011

     

     

     XLVI

     

     

     XLV

     

     

    Оренбург

     

     

    Израиль

     

     

    Таиланд

     

      

     ЮАР

     

    2010

     

     

     XLV

     

     

     XLIV

     

     

    Белгород

     

     

    Тайвань

     

     

    Хорватия

     

     

     Нигерия

     

    2009

     

     

     XLIV

     

     

     XLIII

     

     

     Жуковский

     

     

      Таиланд

     

     

     Мексика

     Муниципальный этап 2010

     

     Азербайджан

     

    Скачать задания

    Муниципальных этапов

    2009-2016

    одним архивом

    Скачать задания

    Региональных этапов

    2009-2016

    одним архивом

    Скачать задания

    Финалов России

    2009-2016

    одним архивом

    Скачать задания

    APhO

    2009-2016

    одним архивом

    Скачать  задания

    IPhO

    2009-2016

    одним архивом

    Скачать задания

    IJSO

    2009-2016

    одним архивом

    Архив за 2000-2008 годы

     

    Всероссийская олимпиада

     

    Всероссийская олимпиада и олимпиада Максвелла по физике

     

    Этапы Всероссийской олимпиады школьников по физике и олимпиады Максвелла

    Документы

     

    Доккументы ЦПМК

     

    от ЦПМК по физике

    Сайт создан для информационной поддержки центральной предметно-методической комиссии олимпиады школьников по физике

    Задачи олимпиады по математике и естественным наукам


    Задачи олимпиады по естественным наукам

    Предварительная олимпиада по физике
    Учащиеся должны ответить на 25 вопросов с несколькими вариантами ответов. 5% лучших могут быть приглашены на участие в USAPhO в формате бесплатных ответов. Лучшие участники могут быть приглашены в лагерь олимпиады по физике в США, где может быть сделан окончательный выбор.
    Нажмите на ссылку ниже, чтобы ознакомиться с прошлыми экзаменами и решениями.

    • F=ma Прошлые экзамены​

    Задачи олимпиады по математике

    AMC 8
     «AMC 8 — это 40-минутный экзамен с несколькими вариантами ответов из 25 вопросов по математике в средней школе, предназначенный для развития и совершенствования навыков решения задач». (https://www.maa.org/math-competitions/american-mathematics-contest-8-amc-8)
    Обладатели высоких результатов могут получить признание и сертификат достижения.
    Нажмите на ссылку ниже, чтобы ознакомиться с прошлыми экзаменами и решениями.

    • ​AMC 8 Проблемы и решения  

    ​AMC 10
     «AMC 10 — это 75-минутный экзамен с несколькими вариантами ответов по математике для средней школы, состоящий из 25 вопросов и содержащий задачи, которые можно понять и решить с помощью основ математического анализа. Калькуляторы запрещены»
    (https://www.maa.org/math-competitions/amc-10-american-mathematics-contest-10)
     
    2,5% лучших по количеству баллов могут быть приглашены для сдачи AIME. Тест доступен только для тех, кто находится в 10-м классе или ниже.
    Нажмите на ссылку ниже, чтобы ознакомиться с прошлыми экзаменами и решениями.

    • AMC 10 Проблемы и решения  

     
    AMC 12
    AMC 12 похож на AMC10, но включает более сложные вопросы и открыт для учащихся 11-го и 12-го классов. Лучшие 5% по количеству очков могут быть приглашены для участия в AIME.
    Нажмите на ссылку ниже, чтобы ознакомиться с прошлыми экзаменами и решениями.

    • AMC 12 Проблемы и решения

     
    AIME
     «Учащиеся, показавшие исключительные результаты по AMC 10/12, приглашаются продолжить участие в серии экзаменов AMC, кульминацией которых станет Международная математическая олимпиада ( ИМО). Первым в этой серии является Американский пригласительный экзамен по математике (AIME), за которым следуют Математическая олимпиада США и Юношеская математическая олимпиада (USAMO и USAJMO). Лучшие участники из USA(J)MO приглашаются на программу математической олимпиады летом после соревнований. Затем участники программы математической олимпиады имеют право быть отобранными следующим летом в состав команды из шести человек, которая будет представлять Соединенные Штаты Америки в ИМО».
    (https://www.maa.org/math-competitions/invitational-competitions)
    Нажмите на ссылку ниже, чтобы ознакомиться с прошлыми экзаменами и решениями.

    • AIME Проблемы и решения

     
    Другие соревнования
     Ссылка ниже содержит хранилище и дискуссионную доску различных математических олимпиадных задач из многих стран за многие годы.

    • Сборники соревнований со всего мира

     
    MATHCOUNTS
     «Национальная программа, которая предоставляет учащимся 6–8 классов возможность соревноваться в живых личных соревнованиях со своими сверстниками и вместе с ними».
     «Серия соревнований включает 4 уровня соревнований: школа, отделение, штат и национальный уровень. Каждый уровень соревнований состоит из 4 раундов: спринт, мишень, командный раунд и раунд обратного отсчета. В целом раунды рассчитаны примерно на 3 часа».
     ( ( https://www.mathcounts.org/programs/mathcounts-competition-series)
     
    Высокоэффективные участники приглашаются на государственные и национальные мероприятия, часть платы за проезд и проживание предоставляется организаторами Mathcounts. Некоторые денежные вознаграждения могут существовать
     
    Ссылка ниже содержит задачи прошлого года.

    • MATHCOUNTS Архив задач недели предстоящий конкурс.

      • Школьный справочник MATHCOUNTS

      Академия математического образования им. Гаусса

      Занятия по ускорению физики и решению задач. Классы включают младшую физику, физику с отличием, AP Physics 1/2, AP Physics C и олимпиаду по физике. Занятия на олимпиаде по физике готовят учащихся к участию в квалификационных экзаменах US Physics Bowl и олимпиады по физике в США.

      Почему вы должны присоединиться к обучающей программе олимпиады по физике Гаусса?

       Олимпиада по физике предлагает четыре основных преимущества:

      • Продемонстрируйте свою любовь к науке . Мы сосредоточены на научных исследованиях, решении проблем и лучшем понимании нашего физического мира. Изучая физику на раннем этапе, учащиеся имеют больше возможностей для участия в научных исследованиях в старших классах и участия в конкурсных летних программах, основанных на исследованиях.
      • Продемонстрируйте свои таланты на соревнованиях по физике.   Мы готовим вас к участию в олимпиаде по физике, турнире Physics Bowl и других соревнованиях, связанных с физикой.
      • Достигните успехов в учебе по программе Physics First . Мы уникальным образом интегрировали учебную программу AP Physics с экзаменом олимпиады по физике, чтобы добиться наилучшего взаимодействия в обучении и усилиях.
      • Подготовьтесь к успешной учебе в колледже . Большинство колледжей, в том числе школы Лиги плюща, будут принимать экзамен AP Physics C (4 или выше для большинства колледжей и 5 для элитных школ) в качестве кредитов колледжа.

      Олимпиада по физике и отбор сборной команды США по физике

      Физика — один из самых сложных предметов в старшей школе. Наш подход к решению задач по физике готовит учащихся не только к успеху на занятиях по физике, но и к поступлению в колледж с демонстрацией энтузиазма, приверженности делу и результатов посредством внеклассных занятий на соревнованиях по физике.

      Американская ассоциация учителей физики (AAPT) отвечает за набор, отбор и подготовку команд олимпиады по физике в США каждый год для участия в Международной олимпиаде по физике.

    I в физике формула: Еще больше физических формул и свойств — Физика — Теория, тесты, формулы и задачи

    Еще больше физических формул и свойств — Физика — Теория, тесты, формулы и задачи

    • Главная —
    • Формулы и прочее —
    • Физика: Еще больше формул

    На этой странице представлен исчерпывающий список формул по физике и важнейших физических свойств для успешной подготовки к ЦТ или ЕГЭ. Список составлен в формате «вопрос-ответ» на основе многолетнего опыта, и является самым полным на этом сайте. Успешное изучение всех формул по физике и физических свойств из этого файла позволит абитуриентам, не просто очень уверенно чувствовать себя на ЦТ или ЕГЭ, но и с легкостью, чуть ли не автоматически, решить большую часть экзаменационных заданий. Знание всех этих формул позволит Вам набрать очень солидный балл на экзамене, даже если у Вас нет феноменальных способностей в физике. А если Вы хотите набрать максимальный балл на ЦТ или ЕГЭ, то выучив эти формулы, Вы с легкостью и очень быстро прорешаете основную часть теста, и у Вас останется много времени на решение самых сложных задач теста, в которых Вам, к слову, также понадобится знание этих формул.

     

    Изучать еще больше формул по физике и физических свойств онлайн:

    • Назад
    • Вперёд

     

    Как успешно подготовиться к ЦТ по физике и математике?

    Для того чтобы успешно подготовиться к ЦТ по физике и математике, среди прочего, необходимо выполнить три важнейших условия:

    1. Изучить все темы и выполнить все тесты и задания приведенные в учебных материалах на этом сайте. Для этого нужно всего ничего, а именно: посвящать подготовке к ЦТ по физике и математике, изучению теории и решению задач по три-четыре часа каждый день. Дело в том, что ЦТ это экзамен, где мало просто знать физику или математику, нужно еще уметь быстро и без сбоев решать большое количество задач по разным темам и различной сложности. Последнему научиться можно только решив тысячи задач.
    2. Выучить все формулы и законы в физике, и формулы и методы в математике. На самом деле, выполнить это тоже очень просто, необходимых формул по физике всего около 200 штук, а по математике даже чуть меньше. В каждом из этих предметов есть около десятка стандартных методов решения задач базового уровня сложности, которые тоже вполне можно выучить, и таким образом, совершенно на автомате и без затруднений решить в нужный момент большую часть ЦТ. После этого Вам останется подумать только над самыми сложными задачами.
    3. Посетить все три этапа репетиционного тестирования по физике и математике. Каждый РТ можно посещать по два раза, чтобы прорешать оба варианта. Опять же на ЦТ, кроме умения быстро и качественно решать задачи, и знания формул и методов необходимо также уметь правильно спланировать время, распределить силы, а главное правильно заполнить бланк ответов, не перепутав ни номера ответов и задач, ни собственную фамилию. Также в ходе РТ важно привыкнуть к стилю постановки вопросов в задачах, который на ЦТ может показаться неподготовленному человеку очень непривычным.

    Успешное, старательное и ответственное выполнение этих трех пунктов, а также ответственная проработка итоговых тренировочных тестов, позволит Вам показать на ЦТ отличный результат, максимальный из того, на что Вы способны.

     

    Нашли ошибку?

    Если Вы, как Вам кажется, нашли ошибку в учебных материалах, то напишите, пожалуйста, о ней на электронную почту (адрес электронной почты здесь). В письме укажите предмет (физика или математика), название либо номер темы или теста, номер задачи, или место в тексте (страницу) где по Вашему мнению есть ошибка. Также опишите в чем заключается предположительная ошибка. Ваше письмо не останется незамеченным, ошибка либо будет исправлена, либо Вам разъяснят почему это не ошибка.

    Механика. Формулы по физике — Физика для всех

    w3.org/1999/xhtml» cellspacing=»0″>

    Кинематика

    ОбозначениеИзмеряется вОписание
    Sмпройденный путь
    vм/сскорость
    tсвремя
    xмкоордината
    aм/с2ускорение
    ωс-1угловая скорость
    Tспериод
    νГцчастота
    εс-2угловое ускорение
    Rмрадиус

    Скорость и ускорение:

     

    Равномерное движение: ν = const

     

    Равнопеременное движение: 

     

    Криволинейное движение:

     

    Вращательное движение:

     

    Динамика и статика

    ОбозначениеИзмеряется вОписание
    FНсила
    Pкг*м/симпульс
    aм/с2ускорение
    mкгмасса
    vм/сскорость
    pНвес тела
    gм/с2ускорение свободного падения
    EДжэнергия
    AДжработа
    NВт мощность
    tсвремя
    Iкг*м2момент инерции
    Lкг*м2момент импульса
    MН*ммомент силы
    ωс-1угловая скорость

    Первый закон Ньютона:
    При ∑ F = 0 => v = const

    Второй закон Ньютона:

     

    Третий закон Ньютона:

     

    Основной закон динамики для неинерциальных систем отчета.
    ma=ma0+Fинерц ,где а- ускорение в неинерциальной а0— в инерциальной системе отчета.

    Скорость центра масс:

     

    Закон всемирного тяготения:

     

    Вес тела:

     

    Сила трения:

     

    Закон Гука:

     

    Закон Гука: σ = Eε, где Е- модуль Юнга.

     

    Динамика и статика вращательного движения:

     

    системаосьI
    точка по окружностиось симметрииmR2
    стержень через середину1/12 mR2
    стержень через конец1/3 mR2
    шарчерез центр шара2/5 mR2
    сферачерез центр сферы2/3 mR2
    кольцо или тонкостенный цилиндрось симметрииmR2
    диск сплошной цилиндрось симметрии1/2 mR2

    Условие равновесия тел  ∑ M = 0

    Закон сохранения импульса:

     

    Потенциальная и кинетическая энергия. Мощность:

     

    Закон сохранения энергии:

     

    ‎App Store: Формулы физики Lite

    Описание

    Благодаря успеху приложения Maths Formulas, приложение Physics Formulas было разработано и выпущено, чтобы помочь пользователям быстро обращаться к любым физическим формулам для учебы и работы. Это приложение отображает самые популярные формулы в семи категориях:
    — Механика
    — Электричество
    — Теплофизика
    — Периодическое движение
    — Оптика
    — Атомная физика
    — Константы

    Это приложение имеет все функции, чтобы помочь пользователям удобно использовать приложение
    — Инструменты: пользователи могут вводить данные, и приложение рассчитает некоторые популярные физические задачи.
    — Поддержка нескольких языков: лучше всего читать на родном языке, а также на английском, чтобы расширить свои языковые навыки. Это приложение поддерживает английский, вьетнамский, китайский (традиционный/упрощенный), турецкий, испанский, немецкий, французский, португальский, русский, индонезийский, персидский, итальянский, хинди и арабский язык. Скоро появятся новые языки.
    — Избранная папка: сохраняйте часто используемые формулы в избранной папке для быстрого доступа к ним.
    — Обмен: коснитесь и поделитесь формулой с друзьями через сообщение, электронную почту или Facebook.
    — Поиск: пользователи могут вводить ключевые слова в верхней части экрана, чтобы быстро найти формулу.
    — Добавьте свои собственные формулы или заметки в раздел «Избранное».
    — Добавьте свои собственные инструменты в раздел «Инструменты» (неограниченное количество формул и переменных для полной версии).

    Это важное приложение для всех, особенно для студентов, инженеров и ученых.

    Версия 4.0

    — Настройте интерфейс приложения для всех размеров телефонов.
    — Улучшить некоторые второстепенные функции.

    Рейтинги и обзоры

    51 Рейтинг

    Отличное приложение с большим количеством U

    Цвет действительно дополняет пользовательский интерфейс и помогает с распознаванием и упрощает определение местоположения/перемещение в приложении.
    Вам определенно следует добавить подкатегории для таких формул, как электричество (интегралы, работа и т. д.), а также отношения между формулами. Это сделало бы это приложение удивительным.

    Очень удобный доступ к фундаментальным формулам

    Хотел бы я иметь это, когда учился на инженерном факультете

    Бесполезный

    Сбои на iPhone 5s ios 9. 3.3

    Разработчик Sinh Chuong Nguyen не предоставил Apple подробностей о своей политике конфиденциальности и обработке данных. Для получения дополнительной информации см. политику конфиденциальности разработчика.

    Сведения не предоставлены

    Разработчик должен будет предоставить сведения о конфиденциальности при отправке следующего обновления приложения.

    Информация

    Продавец
    Шинь Чуонг Нгуен

    Размер
    18,4 МБ

    Категория
    Образование

    Возрастной рейтинг
    4+

    Авторское право
    © NSC Co.

    Цена
    Бесплатно

    • Тех. поддержка
    • Политика конфиденциальности

    Еще от этого разработчика

    Вам также может понравиться

    ‎Физическая формула в App Store

    Все физические формулы и уравнения собраны в одном приложении.
    Вы также можете добавлять свои собственные заметки, которые будут полезны для последней проверки!

    Все формулы и уравнения по физике собраны в одном приложении.
    Вы также можете добавлять свои собственные заметки, которые будут полезны для последней проверки!

    В этом приложении мы попытались объединить все физические формулы и уравнения, необходимые для решения числовых.
    Он охватывает все аспекты механики, теплофизики, электростатики и электрического тока, магнетизма, лучевой оптики, волновой оптики и современной физики.

    Это приложение чрезвычайно полезно для студентов, обучающихся в классах 11 и 12 или на первом курсе, а также для тех, кто готовится к конкурсным экзаменам, таким как JEE main, JEE Advance, BITSAT, MHTCET, EAMCET, KCET, UPTU (UPSEE), WBJEE, VITEEE, NEET PMT, CBSE PMT, AIIMS, AFMC, CPMT и все другие инженерные и медицинские вступительные экзамены.

    Это приложение также очень полезно для учителей, преподающих физику.

    В дополнение к предварительно заполненным формулам,
    — Вы можете создавать свои собственные главы или разделы.
    — Вы можете добавлять фотографии формулы или фотографии напрямую из галереи телефона или камеры.
    — Вы также можете добавлять текстовые заметки к своим главам.
    — Эта функция поможет вам добавлять свои собственные заметки, и вы сможете получить к ней доступ в любое время и в любом месте, не открывая книги.

    Особенности приложения
    — Простой интерфейс: легко переходите к любой теме.
    Красиво оформленный для планшетов
    — Популярные формулы физики и уравнения расположены наиболее удобным способом.

    Формула в физике периода: Физика: формула периода колебаний

    формула расчета, как найти для физического маятника

    Содержание:

    • Что такое колебательный процесс 
    • Определение периода колебаний, формула
      • Как найти период для физического маятника
    • Примеры решений

    Содержание

    • Что такое колебательный процесс 
    • Определение периода колебаний, формула
      • Как найти период для физического маятника
    • Примеры решений

    Что такое колебательный процесс 

    Колебания — это движения или процессы, которые повторяются с определенным интервалом времени.

    Систему, совершающую колебания, называют колебательной системой или осциллятором.

    Исходя из физической природы, колебательные процессы бывают механического, электромагнитного и других видов.

    Осторожно! Если преподаватель обнаружит плагиат в работе, не избежать крупных проблем (вплоть до отчисления). Если нет возможности написать самому, закажите тут.

    Свободные или собственные колебания — колебания, которые наблюдают в системе, предоставленной себе после выведения из равновесного состояния.

    Вынужденными колебаниями называют колебания, происходящие под действием внешней силы, изменяющейся периодически.

    При механических колебаниях, которые относят к категории вынужденных:

    \(F=F_{0}\cos \cot\)

    Гармоническими колебаниями называют колебания, определяемые физической величиной, которая изменяется, согласно закону синуса или косинуса.

    Разные периодические процессы, повторяющиеся в течение равных временных интервалов, могут быть записаны в виде суммы или суперпозиции гармонических колебаний.

    Определение периода колебаний, формула

    Колебательный процесс можно представить в виде уравнения. Тогда гармоническое колебание значения х будет представлено следующей формулой:

    \(x(t)=A\times \cos \left(\omega _{0}t+\phi _{0} \right)\)

    Где \(x(t)\) является отклонением колеблющейся физической величины от равновесного значения;

    А представляет собой амплитуду гармонических колебаний;

    \(\omega _{0}\) равно циклической или круговой частоте колебаний;

    \(\phi _{0}\) является начальной фазой колебаний, характерной для момента времени t=0, что можно определить с помощью выбора начала отсчета времени;

    \(cp(t)=(co_{0}t+cp_{0})\) описывает фазу колебаний в момент времени t, определяется в радианах, соответствует значению колеблющейся величины в данное время.

    В случае, когда имеется какая-либо материальная точка с массой m, характеристика х будет соответствовать смещению тела из равновесного положения. Следует заметить, что амплитуда и частота гармонических колебаний обладают постоянными значениями. Исходя из того, что cos меняет значение в интервале от +1 до -1, параметр х будет изменяться от +А до –А. Так как:

    \(\cos \left(\alpha +2\pi \right)=\cos \alpha,\)

    то х остается без изменений при фазе колебаний, получающей приращение в $$2\pi$$

    Период колебаний Т представляет собой минимальный временной интервал, в течение которого колебательная система возвращается в то состояние, в котором она находилась в начальный момент времени, определенный произвольно.

    В этом случае фаза будет увеличена на \(2\pi:\)

    \(\omega _{0}(t+T)+\phi _{0}=\left(\omega _{0}t+\phi _{0} \right)+2\pi\)

    Из данного равенства можно вычислить период колебаний:

    \(T=\frac{2\pi }{\omega _{0}}\)

    Частота колебаний v является величиной, которая обратна периоду колебаний. Это количество полных колебаний, выполняемых за единицу времени:

    \(v=\frac{\omega _{0}}{2\pi}\)

     

    На графике изображены гармонические колебания, где а — зависимость смещения х от времени /, б — зависимость скорости vx от времени С, в — зависимость ускорения ах от времени t. {2}x(t)=0\)

    Таким образом, получается, что формула циклической частоты пружинного маятника имеет следующий вид:

    \(\omega =\sqrt{\frac{mgl}{J}}\)

    В таком случае для расчета периода колебаний математического маятника будет использоваться формула:

    \(T =\frac{2\pi }{\omega }=2\pi \sqrt{\frac{J}{mgl}}\)

    Исходя из расчетов, можно сделать следующие выводы:

    1. Период пружинного маятника \(T =2\pi \sqrt{\frac{m}{k}}\)
    2. Период математического маятника \(T =2\pi \sqrt{\frac{L}{g}}\)
    3. Период крутильного маятника \(T =2\pi \sqrt{\frac{I}{K}}\)

    В приведенных формулах:

    • T — период физического маятника;
    • J — момент силы маятника относительно оси вращения;
    • l — расстояние от оси вращения до центра масс;
    • m — масса маятника;
    • g=9.8 — ускорение свободного падения.

    Примеры решений

    Задача № 1

    Шариком, привязанным к нити, совершено 60 колебаний в течение 2 минут. Необходимо определить, каковы период и частота колебаний шарика.

    Решение

    \(T =\frac{t}{N}=\frac{120}{60}=2\)

    \(V=\frac{1}{T}=\frac{1}{2}=0.5\)

    Ответ: период колебаний маятника равен 2 секундам, а частота составляет 0,5 Гц.

    Задача № 2

    Согласно изображенного графика зависимости координаты от времени, необходимо рассчитать характеристики колебательного движения тела.

     

    Решение

    А = 20

    Т = 0,8

    \(V=\frac{1}{T}=\frac{1}{0,8}=1,25\)

    \(x(t)=A\sin 2\pi Vt=0.2\sin 2\pi \times 1.25t=0.2\sin 2.5\pi t\)

    Ответ: амплитуда колебаний маятника составляет 0,2 метра, период колебаний соответствует 0,8 с, частота колебаний равна 1,25 Гц, уравнение координаты будет записано в следующем виде: \(x(t)=0.2\sin 2.5\pi t\)

    Задача № 3

    Необходимо определить, какой длиной обладает математический маятник, который совершает гармонические колебания при частоте 0,5 Гц на поверхности Луны. {2}}\approx 0.16\)

    Ответ: длина математического маятника примерно составляет 0,16 метра.

    Насколько полезной была для вас статья?

    У этой статьи пока нет оценок.

    Выделите текст и нажмите одновременно клавиши «Ctrl» и «Enter»

    Поиск по содержимому

    Период колебаний – формула определения, расчет

    4.8

    Средняя оценка: 4.8

    Всего получено оценок: 123.

    4.8

    Средняя оценка: 4.8

    Всего получено оценок: 123.

    Важнейшим параметром, требуемым при расчетах колебательных и волновых процессов, является период колебаний. Он входит во многие формулы, и является одним из базовых. Рассмотрим это понятие.

    Колебательный процесс

    Одними из самых частых процессов в Природе являются колебательные. Как правило, любой колебательный процесс состоит в том, что некоторый параметр рассматриваемой системы изменяет свое значение, периодически отклоняясь то в одну, то в другую сторону от некоторого положения равновесия.

    Рис. 1. Колебательные процессы в природе.

    Колебания маятника

    Простейший пример колебательного процесса – маятник, легкая нить с грузом на конце. Отклоним его от равновесия в крайнее положение, а потом отпустим (чтобы уменьшить влияние трения, отклонение должно быть намного меньше длины нити).

    Груз, начнет движение к противоположной крайней точке. Здесь его скорость упадет до нуля, и он качнется в обратную сторону до начального положения. (Реальный маятник имеет потери на трение, и немного не дойдет до начальной точки, но этим небольшим отклонением можно пренебречь).

    Рис. 2. Колебания маятника.

    Полное движение, которое начинается от начальной точки и продолжается до ближайшего возвращение в нее, называется колебанием.

    Период колебаний

    Если сравнить несколько последовательных колебаний, то можно заметить, что они очень похожи. При этом каждое колебание длится одно и то же время.

    Время за которое происходит одно колебание, называется периодом колебаний. {-6}$ нс (период рентгеновского излучения) и до 250 млн. лет (время обращения Солнечной Системы вокруг центра нашей галактики).

    Что мы узнали?

    Одно колебание маятника (или другого колеблющегося объекта) – это движение от точки максимального отклонения и до возвращения в эту точку. Время, за которое совершается одно колебание, называется периодом колебаний.

    Тест по теме

    Доска почёта

    Чтобы попасть сюда — пройдите тест.

    • Егор Князев

      5/5

    Оценка доклада

    4.8

    Средняя оценка: 4.8

    Всего получено оценок: 123.


    А какая ваша оценка?

    Как рассчитать период движения в физике

    Мир природы полон примеров периодического движения, от орбит планет вокруг Солнца до электромагнитных колебаний фотонов и биения нашего собственного сердца.

    Все эти колебания связаны с завершением цикла, будь то возвращение тела на орбиту в исходную точку, возвращение вибрирующей пружины в точку равновесия или расширение и сокращение сердцебиения. Время, за которое колебательная система совершает полный цикл, называется ее период ​.

    Период системы — это мера времени, и в физике он обычно обозначается заглавной буквой ​ T ​. Период измеряется в единицах времени, соответствующих данной системе, но чаще всего используются секунды. Секунда — это единица времени, первоначально основанная на вращении Земли вокруг своей оси и на ее орбите вокруг Солнца, хотя современное определение основано на колебаниях атома цезия-133, а не на каком-либо астрономическом явлении.

    Периоды некоторых систем интуитивно понятны, например, вращение Земли, равное дню, или (по определению) 86 400 секунд. Вы можете рассчитать периоды некоторых других систем, таких как колеблющаяся пружина, используя характеристики системы, такие как масса и жесткость пружины.

    Когда дело доходит до колебаний света, все становится немного сложнее, потому что фотоны движутся в поперечном пространстве, пока они вибрируют, поэтому длина волны является более полезной величиной, чем период.

    Период обратно пропорционален частоте

    Период – это время, необходимое колебательной системе для завершения цикла, тогда как ​ частота (​ f ​) ​ – это количество циклов, которое система может совершить завершить в заданный период времени. Например, Земля вращается один раз в сутки, поэтому период равен 1 дню, а частота также равна 1 обороту в сутки. Если вы установите стандарт времени на годы, период составит 1/365 года, а частота — 365 циклов в год. Период и частота являются обратными величинами:

    T = \frac{1}{f}

    В расчетах, связанных с атомными и электромагнитными явлениями, частота в физике обычно измеряется в циклах в секунду, также называемых Герцами (Гц), с −1 или 1/сек. . При рассмотрении вращающихся тел в макроскопическом мире число оборотов в минуту (об/мин) также является общепринятой единицей. Период может измеряться в секундах, минутах или любом другом подходящем периоде времени.

    Период простого гармонического осциллятора

    Самым основным типом периодического движения является движение простого гармонического осциллятора, который определяется как движение, которое всегда испытывает ускорение, пропорциональное его расстоянию от положения равновесия и направленное к равновесию позиция. В отсутствие сил трения и маятник, и груз, прикрепленный к пружине, могут быть простыми гармоническими осцилляторами.

    Колебания массы на пружине или маятнике можно сравнить с движением тела, вращающегося с равномерным движением по круговой траектории с радиусом ​ r ​. Если угловая скорость тела, движущегося по окружности, равна ω, его угловое смещение ( θ ) от начальной точки в любой момент времени t равно θ ​ = ωt ​, и компоненты его положения x и y равны x = r ​ cos(​ ωt ​) и ​ y ​ = ​ r ​ sin(​ ωt ​).

    Многие осцилляторы движутся только в одном измерении, и если они движутся горизонтально, то движутся в направлении ​ x ​. Если амплитуда, которая дальше всего отходит от положения равновесия, равна A , то положение в любой момент времени t равно x = A cos( ωt ​). Здесь ω известно как угловая частота и связано с частотой колебаний ( f ​) уравнением ​ ω ​ = 2π​ f ​. Поскольку ​ f ​ = 1/​ T ​, период колебаний можно записать так:

    T = \frac{2π}{ω}

    Пружины и маятники: уравнения периода по закону Гука на массу пружины действует восстанавливающая сила ​ F ​ = −​ k x ​, где ​ k ​ – характеристика пружины, известная как жесткость пружины, и ​ x — смещение. Знак минус указывает, что сила всегда направлена ​​против направления перемещения. Согласно второму закону Ньютона эта сила также равна массе тела ( м ), умноженной на его ускорение ( a ), поэтому ma = − kx .

    Для объекта, колеблющегося с угловой частотой ω , ускорение равно − 2 cos ωt или, упрощенно, − ω 2 x ​. Теперь вы можете написать м ( − ω 2 x ​) = − kx , удалить x и получить √ 1 ω 90 к ​/​ м ​). Тогда период колебаний массы на пружине равен:

    T = 2π\sqrt{\frac{m}{k}}

    Аналогичные рассуждения можно применить к простому маятнику, на котором вся масса центрируется на конце строки. Если длина строки равна L ​, уравнение периода в физике для малоуглового маятника (т.е. такого, в котором максимальное угловое смещение от положения равновесия мало), которое оказывается не зависящим от массы, равно

    T = 2π\sqrt{ \frac{L}{g}}

    где ​ g ​ — ускорение свободного падения.

    Период и длина волны волны

    Подобно простому осциллятору, волна имеет точку равновесия и максимальную амплитуду по обе стороны от точки равновесия. Однако, поскольку волна распространяется через среду или пространство, колебания растягиваются вдоль направления движения. Длина волны определяется как поперечное расстояние между любыми двумя одинаковыми точками цикла колебаний, обычно точками максимальной амплитуды по одну сторону от положения равновесия.

    Период волны — это время, за которое одна полная длина волны проходит контрольную точку, тогда как частота волны — это количество длин волн, которые проходят контрольную точку за заданный период времени. Когда период времени равен одной секунде, частота может быть выражена в циклах в секунду (Герцах), а период выражен в секундах.

    Период волны зависит от скорости ее движения и длины волны (​ λ ​). Волна проходит расстояние в одну длину волны за один период, поэтому формула скорости волны равна 9.0009 v ​ = ​ λ ​/​ T ​, где ​ v ​ — скорость. Выразив период через другие величины, вы получите:

    T = \frac{λ}{v}

    Например, если волны на озере разделены 10 футами и движутся со скоростью 5 футов в секунду, период каждой волны 10/5 = 2 секунды.

    Использование формулы скорости волны

    Все электромагнитное излучение, одним из видов которого является видимый свет, распространяется с постоянной скоростью, обозначаемой буквой ​ c ​, через вакуум. Вы можете написать формулу скорости волны, используя это значение, и поступая так, как обычно делают физики, заменив период волны на ее частоту. Формула принимает вид:

    c = \frac{λ}{T} = f × λ

    Поскольку c является константой, это уравнение позволяет вычислить длину волны света, если известна его частота и наоборот. наоборот Частота всегда выражается в герцах, а поскольку длина волны света чрезвычайно мала, физики измеряют ее в ангстремах (Å), где один ангстрем равен 10 −10 метров.

    Период маятника: значение, формула и частота

    Когда что-то свободно свисает с потолка, и вы толкаете его, оно начинает раскачиваться взад и вперед. Но как быстро он будет качаться и почему? Это то, на что мы действительно можем ответить, и есть довольно простая формула, чтобы понять это. Эти вопросы связаны со свойством, называемым периодом маятника.

    Значение периода маятника

    Чтобы понять, что такое период маятника, нам нужно знать значение двух вещей: периода и маятника.

    Маятник представляет собой систему, состоящую из объекта определенной массы, подвешенного на стержне или шнуре к неподвижной оси. Висячий объект называется боб .

    Маятник будет качаться вперед и назад, и максимальное значение, которое принимает угол θ шнура с вертикалью, называется амплитудой . Эта ситуация на самом деле довольно сложная, и в этой статье мы будем говорить только о простой версии маятника.

    Простой маятник — это маятник, в котором стержень или шнур не имеют массы, а ось не имеет трения.

    На рисунке ниже изображен простой маятник.

    Простой маятник, адаптированный из изображения Chetvorno (Public Domain).

    В этой статье всякий раз, когда мы говорим о маятнике, мы имеем в виду простой маятник с небольшой амплитудой. Теперь, когда мы понимаем, что мы подразумеваем под маятником, нам нужна еще одна информация, а именно, что мы подразумеваем под периодом.

    период маятника — это продолжительность одного полного колебания боба.

    Например, продолжительность времени между двумя последовательными ситуациями, в которых груз маятника находится полностью вправо, составляет один период маятника.

    Влияние длины на период маятника

    Длина шнура маятника влияет на период маятника, которому он принадлежит. Это утверждение довольно убедительно, если мы просто посмотрим на несколько повседневных примеров.

    Некоторые елочные игрушки являются хорошими примерами маятника. Эти маленькие украшения имеют небольшую длину шнура в пару сантиметров и небольшие периоды менее полсекунды (быстро раскачиваются).

    Качели для детской площадки являются примером маятника с длиной шнура в несколько метров. Период этих колебаний часто превышает 3 секунды.

    Набор колебаний, из которых левое будет иметь более короткий период, чем правое, Wikimedia Commons (Public Domain).

    Таким образом, чем длиннее шнур, тем больше период маятника.

    Другие факторы, влияющие на период маятника

    Есть два других фактора, влияющих на период маятника: ускорение свободного падения и амплитуда маятника. Поскольку мы говорим только о маятниках с малыми амплитудами, единственный другой фактор, который мы должны учитывать, — это гравитационное ускорение. С очень малым гравитационным ускорением мы можем представить, как вещи разыгрываются в замедленном темпе. Таким образом, мы ожидаем, что чем больше ускорение свободного падения, тем быстрее качается маятник и тем меньше период маятника.

    Но подождите, почему масса шарика не влияет на период маятника? Это очень похоже на тот факт, что масса объекта не влияет на скорость его падения: если масса удваивается, сила тяжести, действующая на него, также удваивается, но ускорение остается прежним: а=F/m. То же самое испытывает груз нашего маятника: сила, действующая на груз 1, который в два раза массивнее, чем на груз 2, в два раза больше, но и сам груз в два раза тяжелее, чем груз 2. Таким образом, груз 1 в два раза так же трудно переместить, как и боб 2, поэтому ускорение обоих бобов будет одинаковым (опять же на a=F/m). Следовательно, период маятника не зависит от массы груза.

    Вы можете экспериментально проверить это, подойдя к качелям на игровой площадке и измерив период качания, когда на них кто-то находится и когда на них никого нет. Два измеренных периода окажутся одинаковыми: масса шарика не влияет на период колебания.

    Формула периода маятника

    Если длина шнура маятника и g ускорение свободного падения, то формула периода T маятника:

    T=2πlg

    Мы видим, что были правы в своих предсказаниях. Большая длина шнура маятника и меньшее гравитационное ускорение вызывают больший период маятника, а масса груза вообще не влияет на период маятника.

    Это хорошее короткое упражнение, чтобы проверить правильность единиц измерения в этом уравнении.

    Схема простого маятника с малой амплитудой с указанием соответствующих величин, Арьян ван Дензен — StudySmarter.

    Немного посчитав, мы можем вывести формулу для периода маятника. Нам нужно измерять углы в радианах, чтобы для малых углов мы имели примерно sin( θ ) = θ . Единственные результирующие силы, действующие на груз массой м , — это горизонтальные силы, а единственная горизонтальная сила, которую мы можем найти, — это горизонтальная часть натяжения шнура.

    Общее натяжение шнура примерно равно вертикальной составляющей натяжения, поскольку амплитуда маятника мала. Эта вертикальная составляющая равна направленной вниз силе на груз (поскольку на груз нет чистой вертикальной силы), что составляет его вес мг .

    Тогда горизонтальная часть напряжения равна — мг sin( θ ) (со знаком минус, потому что ускорение направлено в сторону, противоположную его положению, которое мы принимаем за положительное). Это примерно — мг θ из-за малой амплитуды маятника. Итак, ускорение боба составляет а=F/m=-gθ.

    Ускорение также измеряется как вторая производная по времени от его горизонтального положения, что примерно равно a=d2xdt2=d2(lθ)dt2. Butlis постоянная, так что уравнение теперь имеет вид ld2θdt2=a=-gθ, где мы должны найти угол θ как функция времени t . Решение этого уравнения (как вы можете проверить) есть θ=Asinglt, где A — амплитуда маятника. Мы видим, что θ равно A через каждые 2πl единиц времени, поэтому период маятника равен T=2πlg. Этот вывод ясно показывает, откуда берутся все факторы, влияющие на период маятника.

    Делаем вывод, что на Земле единственным фактором, влияющим на период маятника, является длина шнура маятника.

    Вычисление периода маятника

    Предположим, мы можем рассматривать детские качели как простой маятник. Каков период качелей, седло которых расположено на 4 м ниже оси вращения, если мы позволим им раскачиваться только плавно, т. е. с небольшой амплитудой?

    Мы знаем, что г = 10 м/с 2 и что l=4м. Затем период T этого маятника рассчитывается как:

    T=2πlg=2π4m10m/s2=4s.

    Это действительно то, что мы знаем из собственного опыта.

    Предположим, мы можем рассматривать сережку как простой маятник. Если кто-то идет, он лишь немного толкает сережку, вызывая небольшую амплитуду. Каков срок службы такой серьги, если длина шнура 1 см?

    Период этого маятника рассчитывается следующим образом:

    T=2πlg=2π0,01м10м/с2=0,2с.

    Это также то, что мы знаем из опыта: маленький маятник качается очень быстро.

    Частота маятника

    Частота (часто обозначаемый как f ) системы всегда является обратным периоду этой системы.

    Следовательно, частота маятника определяется как:

    f=12πgl.

    Помните, что стандартной единицей измерения частоты является герц (Гц), который является обратным значением секунды.

    Как найти объем формула в физике: «По какой формуле можно найти объем?» — Яндекс Кью

    как найти значение, расчёт примеров для 7 класса, площадь фигуры

    Математика

    12.11.21

    12 мин.

    Форма объекта и его линейные размеры определяют один из важных с геометрической точки зрения параметров — объём. Формула для прямоугольника, позволяющая его вычислить, довольно проста и основана на знании стороны и высоты. Но при решении не всегда известны значения рёбер фигуры, поэтому приходится использовать свойства параллелограмма, а именно — способы нахождения площади.

    Оглавление:

    • Общие сведения
    • Вычисление объёма
    • Решение задач
    • Использование калькулятора

    Общие сведения

    По своей сути объём является количественной характеристикой пространства, которое занимает тело или вещество. Простыми словами, этот параметр показывает вместимость. В качестве единицы измерения, согласно СИ, принят кубический метр. За обозначение же объёма взята латинская буква V.

    У тел, имеющих простую форму, характеристики находятся путём перемножения площади на высоту. Например, для куба он равен a3, прямоугольной призмы — h * b * a, пирамиды — (S * b * h) / 3. В эллипсоидных фигурах при расчётах используется радиус. Так, для конуса объём равен (p * R2 * h) / 3, сферы — (4 * p * R3) / 3, тора — 2 * p2 * R1 * R22.

    Объём плоских фигур, таких как треугольник, круг, квадрат, прямоугольник, равен нулю. Но если их стороны или окружности имеют связанные с ними попарно параллельные линии, то они уже являются объёмными фигурами. Например, прямоугольник в пространстве называют параллелепипедом. Таким образом, чтобы найти объём прямоугольника, необходимо, чтобы он представлял собой прямоугольный параллелепипед, иначе действие будет бессмысленным.

    Определить объём — значит, знать две основные величины фигуры:

    • площадь — двумерная характеристика, определяющая размер фигуры;
    • высоту — это длина перпендикуляра, опущенного из вершины на основание в трёхмерном пространстве.

    Так как площадь измеряется в метрах квадратных, а высота просто в метрах, то перемножение площади и высоты как раз и даст единицу измерения объёма — метр кубический.

    Чтобы узнать площадь прямоугольника, используют его свойства. Изучают их обычно в седьмом классе средней школы. К основным из них относят правило равенства противоположных сторон и углов, деление диагонали пополам в точке пересечения и их равенство.

    Именно одинаковые диагонали являются отличительным свойством прямоугольника от параллелограмма. Фактически же диагональ делит фигуру на два прямоугольных треугольника. Это свойство довольно часто используют при проведении расчётов площади или объёма.

    Вычисление объёма

    Существует теорема сообщающая, что объём параллелепипеда, то есть тела основанием которой является параллелограмм, можно рассчитать, умножив площадь основания на высоту фигуры. Записывается это формула как V = h * S, при этом S является произведением сторон прямоугольника.

    Исходя из этого вычислить объём прямоугольника (параллелепипеда) можно по формуле: V = a * b * h, где: a, b — рёбра фигуры; h — высота тела.

    Другими словами, параметр находится как произведение трёх измерений фигуры. Для доказательства нужно рассмотреть два возможных случая.

    1. Пусть имеется фигура, которая состоит из трёх измерений: a, b, c. Первые два являются основанием, к которому пристроена третье. Основание можно представить, как совокупность прямоугольников с площадью S = a * b, состоящую их квадратных единиц. На каждом из квадратов размещается кубическая единица. В итоге получается слой, состоящий из S единиц в кубе. Учитывая, что высота слоя это одна линейная единица, а высота всей фигуры состоит из энного количества таких единиц, то внутри тела можно поместить энное число слоёв. А значит, объём тела равен произведению этих кубических единиц, то есть V = a * b * c.
    2. Имеется прямоугольный параллелепипед. В его основании лежит прямоугольник с вершинами A, A1, B, B1. Соответственно плоскости ABCD и A1B1C1D1 будут боковыми гранями. В середине фигуры можно построить перпендикулярную плоскость MNPQ являющуюся сечением. Она будет равновеликим прямому параллелепипеду с основанием MNPQ и высотой (боковым ребром) BC. По признаку перпендикулярности, плоскости с двугранными углами являются прямыми. Отсюда можно утверждать, что MNPQ — прямоугольное тело, а значит и параллелепипед прямоугольный. Значит, его объём можно найти как произведение MN * MQ * BC. Исходя из того, что MQ перпендикулярно BC, площадь основания можно рассчитать как MQ * BC. А так как MN высота, то объём параллелепипеда можно вычислить, умножив площадь его основания на высоту.​

    Действительно, можно понять и без доказательств, что формула объёма верна. Если представить замкнутую фигуру с любым основанием, образованную в пространстве прямыми линиями, то в её середину можно будет поместить столько оснований, сколько позволит высота тела. Используя законы умножения, чтобы узнать, сколько же поместится в середине фигуры таких площадей, нужно основание умножить на высоту. То есть и получается доказываемая формула: V = S * h.

    Решение задач

    На самом деле вычисление объёма не только выполняют на уроках математики. Это знание востребовано в довольно многих специальностях и науках. Например, при строительстве, в архитектуре, инженерии, физике, химии. Поэтому знание нахождения параметра может пригодиться не только в школе. Теорию обязательно необходимо закреплять на практике. Вот некоторые задачи, которые помогут усвоить рассматриваемый материал:

    1. Пусть есть параллелепипед с прямыми сторонами. Его рёбра у основания равняются 19 и 20 сантиметрам. Размер же боковой грани составляет 10 сантиметров. Вычислить объём фигуры. Эта задача на одну формулу, все данные для подстановки в неё известны. Так, V = a * b * c = 19 * 20 * 10 = 3 800 см3 = 0,0038 м³.
    2. Пусть имеется параллелепипед с основанием 1 см на 1,2 см и высотой 0,8 см. Из него был удалено другое прямоугольное тело с размерами 0,3 x 0,55 x 0,5. Найти объём получившейся фигуры. Так как искомый параметр новой фигуры равен разнице изначального и удалённого объёмов, то зная формулу найти ответ не составит труда: V = 0,8 * 1 * 1,2 — 0,3 * 0,5 * 0,55 = 0,877 см3.

    3. Дан прямоугольный параллелепипед с вершинами ABCD и A1B1C1D1. Сравнить объём образованного в середине пирамиды AA1BD тела со значением фигуры. Для удобства решения стороны AB, AD, AA соответственно можно обозначить как x, y, z. Тогда объём прямоугольного тела будет равен Vп = Sп * AA1 = x * y * z. Если начертить условие на рисунке, то можно отметить, что площадь пирамиды вполовину меньше площади основания прямоугольника. То есть, Sabd = 0,5 * Sabd. Тогда V = Sabd * AA1 / 3 = x * y * z / 3 * 2 = x * y* z / 6. Значит, объём вписанной пирамиды меньше в шесть раз чем у фигуры.
    4. В гальванической ванне помещается три тысячи литров раствора. Высота наполнения ёмкости при этом достигает 75 сантиметров. В ванную поместили заготовку, после чего уровень поднялся на два сантиметра. Найти объём заготовки в метрах кубических. Итак, в одном кубическом метре содержится тысяча литров. Поэтому изначально в ёмкости было 3 м³ раствора. Значит, изначально в ванне раствор занимал: 3 = S * 75. Отсюда s = 3/75 = 1/25 см2. Объём детали составляет: V = S * 2 = (1/25) * 2 = 2 / 25 = 0,08 м³.

    Использование калькулятора

    Конечно же, на обычном калькуляторе объём прямоугольника не подсчитаешь. Разве что известны три его грани и формула нахождения параметра. Тогда нужно будет просто перемножить три числа. В других же случаях, когда нужно решить сложную задачу, связанную с громоздкими вычислениями, можно использовать математические сайты, имеющие название онлайн-калькуляторы.

    Это интернет-сервисы, предлагающие своим пользователям бесплатно воспользоваться услугами по вычислению объёма геометрических фигур и выполнить другие математические операции. Для того чтобы воспользоваться сайтами-решателями, нужно иметь любой гаджет с возможностью подключения его к интернету и установленным на него веб-обозревателем.

    После загрузки онлайн-калькулятора все действия пользователя сводятся к заполнению специальной формы в которую вносится условие задания. Конечно же, такое решение не может называться самостоятельным, но для проверки полученного результата или выявления ошибок в расчёте подходит идеально. Кроме, непосредственно автоматического вычисления объёма большинство сайтов содержат на своих страницах теоретический материал, а также примеры решений типовых заданий. Так что при обучении учащихся их использование на первых порах вполне оправданно.

    Расчет массы и объема тела по его плотности – конспект урока – Корпорация Российский учебник (издательство Дрофа – Вентана)

    Внимание! Администрация сайта rosuchebnik.ru не несет ответственности за содержание методических разработок, а также за соответствие разработки ФГОС.

    Методическая разработка урока физики для 7 класса по теме «Расчет массы и объема тела по его плотности.

    Класс: 7

    Цели урока: повторить тему «Плотность тела», научиться определять объем тела и его массу по известной плотности, научиться решать задачи с применением этих формул.

    Задачи урока: применять полученные знания в новой ситуации; развивать интерес к предмету путём выполнения разных заданий: практических и теоретических.

    Дидактические материалы и оборудование к уроку: презентация, плакаты «Международная система единиц», «Приставки для образования кратных и дольных единиц», индивидуальные карточки с проверочными заданиями; весы с разновесами, мензурка, линейка, мыло, картофелина, камень.

    I. Организационный момент

    II. Актуализация знаний

    1. Фронтальный опрос по теоретическому материалу

    2. Заполнить таблицу (устно)

      v  

      

      t  

      

      ρ  

      V  

     

      кг  

     

      м  

       

    3. Назовите число, составленное вами из номеров верных формул.

    1. v = S/t;
    2. v = St;
    3. S = vt;
    4. ρ = mv;
    5. ρ = m/V;
    6. m = ρ/V.

    3. Выразите в СИ.

    4,8 т; 502 г; 175 г; 35 см; 2341 мм; 584 л; 3846 см3; 0,00567 г/см3

    III Проверка д/з

    Проверочная работа по теме «Масса. Плотность»

    Вариант 1

    1) При нагревании тело расширяется. При этом плотность вещества, из которого оно сделано, …

    а) не меняется б) увеличивается в) уменьшается г) увеличивается, потом уменьшается

    2) Из некоторого вещества изготовили тело массой 42 г в форме параллелепипеда. Параллелепипед имеет следующие размеры: высота 1 см, ширина 2 см, длина 3 см. Определите его плотность.

    3) После того как на одну чашу весов поместили тело, а на другую – набор гирь (20 г, 500 мг, 200 мг, 100 мг, 10 мг, 20 мг, 20 мг и 50 мг), весы пришли в равновесие. Чему равна масса тела?

    Вариант 2

    1) При охлаждении тело сжимается. При этом плотность вещества, из которого оно сделано, …

    а) не меняется б) увеличивается в) уменьшается г) увеличивается, потом уменьшается

    2) Канистра имеет размеры: высота 3 дм, ширина 1 дм, длина 5 дм. Масса пустой канистры 1 кг, заполненной – 13 кг. Чему равна плотность жидкости.

    3) После того как на одну чашу весов поместили тело, а на другую – набор гирь (50 г, 500 мг, 200 мг, 100 мг, 10 мг, 10 мг, 20 мг и 50 мг), весы пришли в равновесие. Чему равна масса тела?

    IV. Решение практических задач

    1) Перед вами лежит кусочек камня. Какие измерения и расчеты необходимо выполнить для определения его плотности? С помощью какого прибора удобно измерить объем тела неправильной формы? Оформите задачу с решением.

    2) Теперь вы видите кусок мыла. Какие измерения и расчеты необходимо выполнить для определения плотности этого тела? С помощью какого прибора удобно измерить объем мыла? Оформите задачу с решением.

    Постановка проблемы

    3) Недавно в одной из телепередач был показан сюжет о крестьянском хозяйстве, в котором вырастили замечательную картошку. Из всех городов приезжают закупать картофель, но только картофелины берут определенного объема. Примерно вот такую картофелину я вам принесла на урок. Давайте, определим ее объем. Вот только в мензурку, да и в отливной стакан она не войдет. Ваши предложения?

    Это и есть тема нашего урока: Расчет массы и объема тела по его плотности.

    Оформите эту задачу в тетрадях: Как найти объем тела, если известна масса тела и его плотность?

    Рассмотрим примеры решения задач.

    1) Перед вами в мензурке находятся вода, молоко и подсолнечное масло. Необходимо рассчитать массы этих жидкостей, зная их плотности.

    2) Рядом с мензурками находятся пластмассовый и стальной шарики, электронные весы. По таблице из учебника выясните плотности этих веществ. Рассчитайте объемы шариков.

    V. Самостоятельное решение расчетных задач

    1. Масса чугунного шара 800 г, его объем 125 см3. Сплошной этот шар или полый?
    2. Чему равна масса железного листа длиной 1 м, шириной 80 см, толщиной 1 мм?
    3. Дубовый брусок имеет массу 800 г. Определите его объем, если плотность дуба  800 кг/м3.

    VI. Рефлексия (подведение итогов урока, оценки)

    Что ж, наше занятие подходит к завершению. Я надеюсь, что полученные вами знания вы сможете использовать не только на уроках по различным предметам, но и будете применять их в повседневной жизни. А теперь рубрика обратной связи. На экране вы видите высказывания. Выберите любое и закончите его (устно).

    Предлагаются варианты ответов: Сегодня я узнал… Было интересно… Было трудно… Я понял, что… Я научился… Меня удивило…

    VII. Д/з

    1) § 23 – учить

    2) Придумать 3 интересные задачи на расчет плотности, массы или объема тел, применяемых в быту (оформить на А4 с решением и иллюстрациями, принести эти тела в класс для решения некоторых задач)

    Калькулятор объема жидкости — Академия калькуляторов

    Конвертер единиц измерения

    Введите плотность жидкости и массу жидкости в калькулятор, чтобы определить объем жидкости. Этот калькулятор также может определить либо плотность, либо массу жидкости, если известны другие переменные.

    • Все калькуляторы объема
    • Калькулятор объемного отношения
    • Калькулятор объемного расхода
    • Калькулятор молярного объема
    • Калькулятор времени удерживания
    • Калькулятор объема колонки
    • Калькулятор емкости на входе

    Формула объема жидкости

    Ниже приведена формула объема жидкости:

    LV = M/d

    • Где LV — масса объема жидкости

      7 M 9 жидкость

    • d плотность жидкости

    Чтобы рассчитать объем жидкости, разделите массу жидкости на плотность жидкости.

    Определение объема жидкости

    Объем жидкости просто определяется как общий объем любой жидкости или комбинации жидкостей.

    Вы можете измерить объем жидкости?

    Один из способов измерения объема жидкости состоит в том, чтобы налить жидкость в ограниченный контейнер, измерить высоту, на которой она заполняет контейнер, а затем определить объем жидкости путем вычисления содержащегося объема и умножения на отношение высоты жидкости к общей высоте .

    Изменяют ли жидкости объем?

    В большинстве случаев такие жидкости, как вода, считаются несжимаемыми, а это означает, что их объем не изменяется при изменении давления или температуры. Есть некоторые исключения из этого правила, но в целом оно верно.

    Имеет ли жидкость объем?

    Да, жидкости абсолютно имеют объем, но если жидкость не определена в контейнере, было бы невозможно измерить объем указанной жидкости обычными методами. Один из способов определить объем — использовать приведенную выше формулу.

    Объем жидкости равен емкости?

    Объем жидкости может совпадать с емкостью только в том случае, если емкость определена таким образом, что объем жидкости на 100 % контейнера указан как емкость.

    Почему объемы жидкости следует считывать на уровне глаз?

    Жидкости, такие как вода, обладают свойством, называемым адгезией. Это притяжение воды к другим поверхностям. Глядя на емкость, наполненную водой, важно читать измерения на уровне глаз, потому что вода будет прилипать к стенкам и может казаться немного выше, чем она есть на самом деле, если вы смотрите на нее под углом.

    Какой прибор измеряет объем жидкости?

    Наиболее распространенным инструментом, используемым для измерения объема жидкости, являются градуированные цилиндры. Это контейнеры с измерениями объема жидкости, когда жидкость наливается в химический стакан. 93.

  • Наконец, рассчитайте объем жидкости по формуле LV = M/d.
  • вопросов и ответов — Сколько атомов в голове человека?

    Предыдущий вопрос

    (Есть ли способ узнать, сколько атомов содержится в объекте?)

    Вопросы и ответы
    Основной индекс

    Следующий вопрос

    (Сколько атомов в теле человека?)

    Предыдущий вопрос

    (Есть ли способ узнать, сколько атомов содержится в объекте?)

    Вопросы и ответы Основной индекс

    Следующий вопрос

    (Сколько атомов в теле человека?)

    Вопросы и ответы Основной индекс

    Сколько атомов в голове человека?

    Мы можем подсчитать количество атомов в вашей голове, если мы знаем плотность и константу, называемую числом Авогадро. Это всего лишь оценка, но она будет хорошей. Уравнение довольно простое. Количество атомов ЛЮБОГО вещества в объеме:

    # атомов = N * (плотность) * объем / (молекулярная масса).

    N — константа, называемая числом Авогадро, равная 6,022*10 23 атомов/моль. Это также могут быть молекулы на моль. В приведенной выше формуле плотность, умноженная на объем, — это просто масса. Если вы знаете, насколько что-то тяжелое или каковы его объем и плотность, вы можете легко это сделать.

    Давайте начнем с простой задачи. В литре воды 1000 кубических сантиметров. Вода легкая, потому что каждый кубический сантиметр имеет 1 грамм массы. Вода состоит из 2 атомов водорода и одного атома кислорода. Водород имеет атомный вес 1, а кислород имеет атомный вес 16. Молекулярный вес воды 18. В литре воды 1000 граммов. Количество молей равно 1000/18 = 55,556 молей. Таким образом, число молекул равно 6,022*10 23  * 55,556 = 3,346*10 25 молекул. Количество атомов в 3 раза больше, потому что каждая молекула состоит из трех атомов, поэтому в литре воды 1,0038*10 26 атомов.

    Теперь мы знаем достаточно, чтобы ответить на ваш вопрос. Типичная человеческая голова весит около 10-12 фунтов. Мы большей частью состоим из воды. Когда вы идете плавать, вы, наверное, замечали, что почти все плавают только частью головы над водой. Это наблюдение приведет вас к выводу, что наша плотность очень близка к плотности воды. Вооружившись всем этим, мы можем оценить количество атомов в вашей голове. Фунт равен 454 граммам, поэтому человеческая голова весом 10 фунтов весит 4540 граммов. Если я предположу, что в среднем мы в основном состоим из воды, потому что наша средняя плотность примерно равна плотности воды, то я могу использовать приведенную выше информацию о воде, чтобы получить ответ.

    Моль на голову = (4540 г) / (18 г/моль) = 252,22 моль
    Молекул на моль = 6,022*10 23 * 252,22 моль = 1,519*10 26 молекул
    атомов на голову = 3 *с на голову = 4,56*10 26

    Это 456 триллионов триллионов атомов!

    Завершу историческую справку.

    Знак объема в физике: Меры объёма — урок. Физика, 7 класс.

    Физики предложили новый метод решения задачи трёх тел / Хабр

    Израильские учёные из Института физики им. Рака при Еврейском университете предложили новый способ упрощения задачи трёх тел, позволяющий точно оценить вероятность покидания системы любым из этих тел. Работа опубликована в журнале Celestial Mechanics and Dynamical Astronomy.

    Задача трёх тел подразумевает описание поведения небесных тел в трёхмерном пространстве, взаимно притягивающихся друг к другу. Ею впервые заинтересовался Исаак Ньютон, пытавшийся в XVII веке описать движения планет Солнечной системы. Он попробовал вывести формулу, описывающую движение Луны вокруг Земли с учётом того, что Земля движется вокруг Солнца. Однако оказалось, что это одна из тех задач, которую легко сформулировать и трудно решить.

    Проблемой в разное время занимались такие величайшие математики, как Эйлер, Лагранж, Якоби и Пуанкаре. Анри Пуанкаре в XIX веке обнаружил, что поведение системы отличается крайней чувствительностью к начальным условиям – первоначальным положениям и скоростям тел. Таким образом, система является хаотичной и не имеет детерминистских решений.

    Появление вычислительных мощностей в XX веке позволило подступиться к этой задаче при помощи численного моделирования. Оказалось, что при неких достаточно общих допущениях в системе трёх тел хаотичные периоды движения сменяются периодическими, пока, наконец, она не распадается на пару тел, вращающихся относительно центра их масс, и третье тело, удаляющееся от них.

    Из-за хаотичного поведения системы нельзя было рассчитывать на то, что компьютеры выдадут точные и долгосрочные прогнозы поведения такой системы. Однако в 1976 году появилась идея о возможности статистического решения этой задачи, предсказывающего вероятность убегания одного из тел.

    Даже такая задача оказалась настолько сложной, что её удалось решить только 45 лет спустя. Доктор Николас Стоун из Еврейского университета со своими коллегами использовали новый метод расчётов, и впервые получили замкнутое математическое выражение для статистического решения задачи.

    Предыдущие методы опирались на т.н. фазовое пространство – набор всех положений и скоростей частиц, составляющих систему. К примеру, фазовое пространство единственной частицы, которая может двигаться по отрезку длиной метр со скоростями не более двух метров в секунду, будет представлять собой прямоугольник с шириной в 1 м и длиной в 4 м/с – поскольку скорость может иметь как положительный, так и отрицательный знак.

    Обычно физики имеют дело с объёмом фазового пространства, или, сокращённо, с фазовым объёмом. Например, вероятность найти частицу в левой половине отрезка связывается с объёмом левой половины прямоугольника в фазовом пространстве, составляющим половину объёма всего прямоугольника.

    Поскольку три тела не ограничены в пространстве, а сила гравитации действует на любом расстоянии, объём фазового пространства в этой задаче бесконечен, и, следовательно, вероятности тоже бесконечны. Поэтому все предыдущие методы принимали некий объём пространства за «область сильного взаимодействия», игнорируя всё остальное.

    В новой работе предлагается изучать «поток фазового объёма». Если мы представим себе ёмкость с газом, в стенке которой есть пара отверстий, то вероятность того, что конкретная молекула вылетит через одно из них, будет пропорциональной потоку газа через каждое из отверстий.

    Поток конечен даже в бесконечном пространстве, поэтому новый подход одновременно избегает бесконечных вероятностей и не прибегает к искусственному ограничению объёма. В итоге теория позволяет предсказать вероятность убегания любого из тел. По словам профессора Кола, одного из авторов работы, миллионы компьютерных симуляций показывают, что предсказания теории очень хорошо соответствуют численным компьютерным симуляциям.

    Ожидается, что исследование повлияет как на решение различных задач астрофизики, так и на понимание целого класса проблем механики.

    Абстрактная математика помогает физике

    Иногда на первый взгляд совершенно абстрактные математические теории помогают физикам-теоретикам понять, как устроен наш мир.

    В год окончания Первой мировой войны двое немецких математиков геттингенской выучки опубликовали работы, имеющие огромное значение для теоретической физики. Одна из самых блестящих алгебраистов XX века Эмми Нётер представила доказательства двух знаменитых ныне теорем, связывающих законы сохранения различных величин (энергии, импульса, углового момента, заряда и т. д.) с симметриями уравнений, описывающих физическую систему.

    Эти теоремы стали мощным и универсальным средством выявления подобных законов в ньютоновской и релятивистской механиках, в теории тяготения, электродинамике, квантовой теории поля и физике элементарных частиц.

    Статья Германа Вейля «Гравитация и электричество», опубликованная не в Геттингене, а в Берлине, известна гораздо меньше. Между тем она и ее продолжение, вышедшее годом позже, положили начало чрезвычайно эффективному подходу к конструированию теорий микромира, который сформировался уже во второй половине XX века. С его помощью была создана объединенная теория трех фундаментальных взаимодействий, сильного, слабого и электромагнитного, которую назвали Стандартной моделью.

    РЕКЛАМА – ПРОДОЛЖЕНИЕ НИЖЕ

    От сил к потенциалам

    Как обычно и бывает, у Вейля имелись предшественники. В начале XIX века работы нескольких математиков, прежде всего Гаусса и Пуассона, преобразовали математический аппарат ньютоновской теории тяготения. В новой интерпретации она предстала как силовое поле, пронизывающее Вселенную. Это поле стали описывать гравитационным потенциалом — скалярной функцией, зависящей от пространственных координат, но не от времени. При этом сила тяготения в любой точке полностью определяется тем, насколько резко изменяется вблизи нее этот потенциал (то есть его градиентом).

    Это нововведение обогатило математический аппарат небесной механики и других разделов физики, где приходится иметь дело с тяготением, но ввело в описание гравитации некую неопределенность. В законе Ньютона фигурируют силы тяготения, которые можно измерять непосредственно, и определяются они однозначно (в выбранной системе единиц). А вот значения гравитационного потенциала можно изменить на любую постоянную величину — градиент останется тем же. В те времена это выглядело тривиальным следствием математического формализма, не имеющим отношения к реальной физике.

    В калибровочных теориях существует очень обширная симметрия, которая неодинаково проявляет себя в разных точках пространства и времени. Поэтому при математическом описании симметрий такого типа появляются параметры, которые зависят от пространственно-временных координат. И вот оказывается, что существование калибровочных симметрий накладывает весьма сильные ограничения на свойства объектов, которые эти теории описывают.
    «Для примера возьмем квантовую электродинамику, — объясняет академик Валерий Рубаков. – Электромагнитные взаимодействия переносят частицы с единичным спином – фотоны. Спин фотона может быть ориентирован только в двух направлениях, вдоль или против его движения. В первом случае мы говорим о правой поляризации, во втором — о левой. Но если строить теорию фотонов чисто формально, ни о чем не задумываясь, появятся еще две поляризации с нулевыми проекциями спина на направление движения. Если такое допустить, теория рассыпется, потеряет самосогласованность. А в теории с правильно подобранной калибровочной симметрией эта проблема не возникает, лишние поляризации оттуда уходят. Аналогичная ситуация имеет место и в теории глюонного поля, переносящего сильные взаимодействия, и в теории слабого взаимодействия, переносящего промежуточные векторные бозоны. Все эти частицы имеют единичный спин, и у всех возникают неприемлемые состояния, которые не исчезают сами по себе, однако изгоняются калибровочной симметрией».

    Столетием позже таким же образом переписали классическую электродинамику. В первоначальной форме она была представлена уравнениями Максвелла, куда входят измеряемые на опыте напряженности электрического и магнитного поля. Эти уравнения тоже удобно выразить через потенциал, только более сложный, чем у ньютоновской гравитации (помимо скалярной части, в него входит вектор, определяющий величину магнитного поля).

    Уравнения электродинамики в такой записи выглядят очень элегантно и естественно встраиваются в пространство-время специальной теории относительности. Однако они становятся неоднозначными, поскольку одному и тому же полю могут соответствовать разные потенциалы. Например, к векторному потенциалу можно добавить любой постоянный вектор, а к скалярному — любое число.

    Более того, эти добавки могут меняться и в пространстве, и во времени, лишь бы они были правильно связаны друг с другом, так что произвол в выборе электромагнитных потенциалов существенно больше, чем в случае ньютоновской гравитации. Физики и математики начала прошлого века прекрасно видели эту неоднозначность, но, как и предшественники, не придавали ей особого значения.

    Калибровочные преобразования

    Это свойство электромагнитных потенциалов имеет глубокий физический смысл. Их взаимные изменения компенсируют друг друга точно таким образом, чтобы сохранить в прежнем виде уравнения Максвелла. Неоднозначность выбора фактически отражает неразрывную связь между электричеством и магнетизмом.

    Преобразования потенциалов, не меняющих уравнений электромагнитного поля, называют калибровочными (этот термин тоже восходит к статьям Вейля) — как говорят физики, эти уравнения инвариантны относительно калибровочных преобразований. В квантовой электродинамике такая инвариантность, в соответствии с теоремой Нётер, влечет за собой закон сохранения электрического заряда. Таким образом, калибровочная инвариантность, несмотря на свой вроде бы формальный характер, открывает возможность заключений, имеющих прямой физический смысл!

    И не только в отношении электромагнетизма. Принцип эквивалентности, на котором базируется общая теория относительности (ОТО), утверждает, что поле тяготения вызывает такие же физические эффекты, как и ускорение. Если недалеко от звездолета с работающим двигателем поместить тяготеющие массы, то в принципе можно полностью скомпенсировать импульсы двигателя и создать в кабине зону невесомости. Такая компенсация ускорений посредством переменного гравитационного потенциала аналогична взаимной компенсации изменений потенциалов электромагнитного поля. Это наводит на мысль, что уравнения ОТО должны подчиняться какому-то аналогу калибровочных преобразований.

    Такие рассуждения сейчас кажутся вполне естественными, но сто лет назад до них никто не додумался. Калибровочная инвариантность — и как идея, и как термин — пришла в теоретическую физику иным путем. Чтобы понять, как это произошло, обратимся к работам Вейля.

    Мир переменных масштабов

    Вейль записал уравнения гравитационного поля в пространстве с иной геометрией, чем та, которой воспользовался Эйнштейн. В итоге к ним добавились формулы, в которых Вейль увидел основные черты уравнений Максвелла. Этим путем он получил математическую конструкцию, которую счел единой теорией электричества и тяготения.

    Уравнения ОТО записываются в римановом пространстве, искривленном четырехмерном пространстве-времени с однозначной метрикой. В отличие от «плоского» евклидового пространства, где при перенесении произвольного вектора вдоль замкнутой кривой по возвращении в исходную точку он окажется в прежней позиции, в римановом пространстве такой перенос закончится поворотом вектора на ненулевой угол, который будет мерой кривизны пространства в этой точке. С другой стороны, длина вектора после переноса остается той же самой — в этом и состоит однозначность метрики.

    От этого ограничения и отказался Вейль. Он предположил, что уравнения тяготения не должны зависеть от масштабов, применяемых для измерения длины. В обыденной жизни можно с равным успехом пользоваться метрами, футами, аршинами и вершками. Численные значения длины любого отрезка зависят от единицы измерения, но отношения между ними строго сохраняются.

    Нечто подобное происходит и в геометрии Вейля, только масштабная единица непрерывно изменяется от точки к точке. Вслед за ней изменяются и длины, но отношения этих длин для любой пары векторов с общим началом остаются неизменными. Операцию смены масштабов Вейль назвал перекалибровкой. Она сохраняет уравнения гравитационного поля — это и есть калибровочная инвариантность в своей ранней исторической ипостаси.

    В теоретической физике инвариантность к определенным преобразованиям приводит к появлению специальных свойств. К примеру, ньютоновское уравнение движения инвариантно к трансляции (смещению на некоторое расстояние в пространстве), что приводит к закону сохранения импульса. Калибровочные преобразования на первый взгляд кажутся абстрактными, но они приводят к существованию различных калибровочных полей, связанных с математическим понятием групп симметрии. В группе U(1) только один фазовый угол, в Стандартной модели ему соответствует один бозон (электромагнитного взаимодействия – фотон), группа SU(2) имеет три фазовых угла (в СМ – три бозона слабого взаимодействия), SU(3) – восемь фазовых углов (восемь бозонов сильного взаимодействия — глюонов).

    Но причем здесь электричество? В ОТО длины векторов сохраняются, поэтому сравнить их не представляет проблемы. А вот Вейлю пришлось ввести математические правила, позволяющие выяснить, имеют ли два вектора в соседних точках одинаковую длину (хотя сама длина при этом не определена!).

    Эти правила он интерпретировал как уравнения Максвелла для электромагнитных потенциалов. Изменение длины вектора определяется именно этими потенциалами (подобно тому, как изменение его ориентации задается кривизной пространства, которая проявляется через гравитацию).

    Вейль отправил рукопись своей статьи Эйнштейну и попросил рекомендовать ее к публикации. Эйнштейн так и сделал, но отметил, что если теория Вейля верна, то частоты оптических спектров должны зависеть от истории излучающих атомов, а это явно противоречит эксперименту. Были выдвинуты и другие возражения, поставившие крест на вейлевском объединении электричества и гравитации. Изумительная по красоте модель оказалась физически несостоятельной.

    Однако гравитация, с которой все начиналось, в Стандартную модель не входит. По словам академика Рубакова, гравитация имеет свою специфику: «При квантовании поля тяготения возникают гравитоны. Это тоже бозоны, но уже не векторные — их спин равен не единице, а двойке. Однако теория гравитации опять-таки подчиняется калибровочной симметрии. Гравитон, подобно фотону, имеет лишь две поляризации, в то время как число математически возможных поляризаций у частицы со спином 2 равно пяти. Калибровочная симметрия гравитационного поля позволяет убрать лишние поляризации и тем самым сделать теорию непротиворечивой. Эту симметрию фактически нашел еще Эйнштейн, хотя в ОТО нет никаких гравитонов. Но там имеется симметрия пространства-времени относительно всех гладких преобразований координат, а это и есть калибровочная симметрия.
    Впрочем, калибровочные теории очень сильны, но все же не всемогущи. Сегодняшние теории элементарных частиц очень сложно объединить с гравитацией, и в этом их очевидная слабость. Все попытки создать квантовую теорию тяготения пока не увенчались успехом. Так что наши нынешние калибровочные модели — это, конечно, еще не вся правда. Я думаю, что для объединенного описания всех четырех фундаментальных взаимодействий придется изобрести новую теорию с еще более широкой калибровочной симметрией. Многие возлагают надежду на теории суперструн, но скорее всего, понадобится что-то еще шире. Но я не сомневаюсь, что в основе этой будущей теории окажутся какие-то калибровочные симметрии. Некоторые ее черты просматриваются уже сейчас, но когда она появится и какую примет форму, я предсказывать не берусь».

    Однако позднее стало ясно, что идея калибровочной инвариантности глубока и конструктивна, а Вейль ошибся лишь в ее конкретном приложении. В 1920-е годы это поняли несколько физиков, в том числе Фриц Лондон — впоследствии один из авторов первой квантовой теории сверхпроводимости. В 1927 году он предложил новую интерпретацию теории Вейля, сделавшую ее частью квантовой физики.

    Вся сила в фазе

    Вот как выглядит идея Лондона в современном выражении. Квантовые объекты описываются комплексной (в математическом смысле) волновой функцией. Измерить ее экспериментально (как и электромагнитные потенциалы!) невозможно. Опытным путем можно выявить лишь вероятности значений физических величин, которые определяются квадратом модуля этой волновой функции. Поэтому ее можно умножить на любое комплексное число с единичным модулем — вероятность от этого не изменится. Если записать такое число в виде экспоненты с чисто мнимым показателем, то операция его умножения на волновую функцию приведет к изменению ее фазы.

    Если на квантовую частицу не действуют никакие силы, изменение фазы не повлечет за собой значимых последствий. Движение заряженной частицы в электромагнитном поле в нерелятивистском случае описывается уравнением Шредингера, которое при умножении на фазовый множитель изменяет свой вид и становится неинвариантным. Это препятствие можно обойти, если одновременно изменить электромагнитные потенциалы с помощью того самого классического преобразования, которое после работ Вейля называется калибровочным. Если записать показатель экспоненты в виде произведения мнимой единицы на заряд частицы и скалярную функцию времени и координат, то эта функция как раз и будет задавать требуемое калибровочное преобразование потенциалов. Оно точно компенсирует те дополнительные члены в уравнении Шредингера, которые появляются после изменения фазы волновой функции.

    В чем физический смысл этой вроде бы чисто абстрактной математики? Состояния частицы, чьи волновые функции различаются лишь фазовыми множителями, с точки зрения эксперимента эквивалентны. Если частица заряжена и, следовательно, подчиняется действию электромагнитного поля, возможность произвольной смены фазового множителя обеспечивается соответствующим изменением электромагнитных потенциалов. Инвариантность уравнения движения частицы относительно выбора фазы волновой функции автоматически приводит к калибровочной инвариантности полевых уравнений.

    Если записать уравнение Шредингера для заряженной частицы без каких-либо электромагнитных потенциалов, найти его решение в виде волновой функции и умножить ее на фазовый множитель, в уравнении появятся добавочные члены. Следовательно, оно должно содержать какие-то компоненты, которые своими изменениями скомпенсируют эти добавки. В качестве таких компонент как раз и выступают электромагнитные потенциалы. Получается, что если волновые функции, различающиеся на произвольный фазовый множитель, описывают одно и то же состояние заряженной квантовой частицы, то должны существовать и электромагнитные поля, которые подчиняются уравнениям Максвелла.

    Таким образом, мы пришли к удивительному результату — фазовая инвариантность порождает электромагнетизм! Этого еще нет у Лондона, хотя логика его рассуждений подводит к такому выводу. Впервые его четко сформулировал Вейль в статье «Электрон и гравитация», опубликованной в 1929 году (хотя он использовал не уравнение Шредингера, а дираковское уравнение для релятивистского электрона). Умножение волновой функции на фазовый множитель у Вейля предстает как новое калибровочное преобразование, тесно связанное с преобразованием электромагнитных потенциалов.

    Инструмент предсказаний

    Идеи Вейля настолько привлекли Вольфганга Паули, что в 1933 году он пересказал их в статье «Волновая механика». В середине 1940-х годов ее прочел молодой китайский физик Янг Чжэньнин, которого очень заинтересовало доказательство связи между фазовой инвариантностью и сохранением электрического заряда. В 1953—1954 годах в Брукхейвенской национальной лаборатории Чжэньнин и аспирант Роберт Миллс применили эти идеи для анализа ядерных сил. Их совместная статья «Сохранение изотопического спина и обобщенная калибровочная инвариантность» сыграла огромную роль в развитии теоретической физики.

    Янг и Миллс первыми показали, что на основе калибровочной симметрии можно предсказывать существование ранее неизвестных физических полей и, как следствие, еще не открытых частиц (Паули пришел к сходным выводам за год до Янга и Миллса, однако воздержался от их публикации). В 1960—1970-е годы этот росток дал обильный урожай в виде Стандартной модели элементарных частиц.

    «Все фундаментальные взаимодействия, за исключением гравитации, переносятся векторными частицами, — говорит профессор МГУ и главный научный сотрудник Института ядерных исследований РАН, автор монографии о калибровочных полях академик Валерий Рубаков, — так уж устроен мир. А при таком раскладе просто необходимо пользоваться калибровочными симметриями, иначе получаются сплошные патологии. Физики шли к пониманию этих вещей очень разными путями. Калибровочная природа электромагнетизма известна еще со времен Вейля, больше 80 лет. Объединенная калибровочная теория слабых и электромагнитных взаимодействий была развита Стивеном Вайнбергом и Абдусом Саламом во второй половине 1960-х годов и окончательно доработана лишь в начале 1970-х. А потом настала очередь и внутриядерных сил. Как раз тогда экспериментаторы показали, что на очень малых дистанциях взаимодействие между кварками не растет, а слабеет. Это явление назвали асимптотической свободой, и поначалу оно не находило разумного объяснения. Однако трое физиков-теоретиков — Дэвид Гросс, Фрэнк Вильчек и Дэвид Политцер — вскоре показали, что в калибровочных моделях глюонных полей асимптотическая свобода возникает естественным образом. Отсюда было недалеко до объединения теорий электрослабых и сильных взаимодействий в единую теоретическую конструкцию, которую назвали Стандартной моделью».

    Символы физики — список физических символов и названий

    В физике существует большое количество физических величин, которые мы учитываем при выполнении вычислений. Чтобы сделать их более удобными для пользователей, а также более простыми в использовании и запоминании, мы часто используем обозначения/символы для представления этих физических величин. Эти обозначения/символы, которые мы используем для представления физических величин при решении связанных с ними задач или для других целей, являются символами.

    В физике все обозначается английским/греческим алфавитом, например, скорость света, длина волны, скорость и так далее.

    Предположим, женщина едет на своей машине со скоростью 30 км/ч и добирается до родного города за 2 часа, а если она едет со скоростью 50 км/ч, то добирается за 1,5 часа. Итак, если нам нужно представить эти единицы в виде символов, как мы можем это сделать?

    В этой статье вы найдете самые популярные физические символы, а также те, которые мы обычно используем в физике, с их названиями, типом величин и соответствующими единицами измерения в табличном формате.

    Примеры физических символов

    Кроме того, символы, используемые для физических величин, сильно различаются. Иногда символ может быть первой буквой физических величин, которые они представляют, например «d», что означает расстояние. В других случаях они могут быть совершенно не связаны с названием физических величин, например, c символизирует скорость света. Они также могут быть в виде греческих символов, таких как λ, что означает длину волны.

    Ниже приведен подробный список наиболее часто используемых символов в физике с их единицами СИ. Обратите внимание, что один и тот же символ может относиться к нескольким величинам.

    Symbols for Physical Quantities Related to Space and Time

    70030

    2923232323232323232323232323232323232323232323232323. величина и направление, но не является векторной величиной.

    (Не подчиняется закону коммутативности)

    Метр0027

    Scalar 

    003

    Symbols

    Quantity/ Coefficients

    S.I. Unit

    Physical Quantity (Scalar/Vector)

    R

    Радиус, радиус Curvature

    Meter

    .

    Вектор

    d

    Расстояние

    Метр

    θ

    Агловое смещение,

    Radian

    x, y, z

    Декартовы координаты

    Unitless

    Scalar

    î, ĵ, k̂

    Cartesian unit vectors

    Unitless

    Vector

    r, θ, φ

    Сферические координаты

    метра/Radian

    Вектор

    R̂, θ̂, φ̂

    Spherical Vectors 9000

    . 0003

    Unitless

    Vector

    r, θ, z

    Cylindrical coordinates

    Metre/Radian

    Scalar

    r̂, θ̂, ẑ

    Цилиндрические векторы

    БЕЗСПОЛЬЗОВАНИЕ

    Вектор

    Нормальный вектор

    0030

    Unitless

    Vector

    Tangential unit vector

    Unitless

    Vector

    h

    Height, Depth

    Метр

    Скаляр

    л, л

    Длина

    t

    Time

    Second 

    Scalar

    D (= 2 r)

    Diameter

    Metre

    Scalar

    C

    Окружность

    МЕТР

    Скалер

    9002

    0002 A

    Area

    Square Metre

    Functions as both scalar and vector (like Area vector in a magnetic flux formula)

    V

    Volume 

    Cubic Метр (M3)

    Скаляр

    τ

    Постоянная времени

    секунда (S)

    Скалар

    (S)

    T

    Periodic time

    Second (s)

    Scalar 

    f

    Frequency 

    1/second or (1/s

    Scalar

    ω

    Угловая частота

    RAD/S

    Scalar

    0479

    Ниже приведены некоторые символы, часто используемые в физике, с их названиями, типом величин и соответствующими единицами СИ в табличном формате.

    Physics Symbols Related to Mechanics

    7 Вектор

    030003

    Symbols

    Quantity/ Coefficients

    S.I. Unit

    Physical Quantity (Scalar/Vector)

    в

    Velocity, speed

    metre/second (m/s)

    Speed ​​= Scalar

    Velocity = vector

    a

    Acceleration

    metre/square second (M/S2)

    Vector

    G

    Ускорение из -за гравитации

    Метр/Square Second

    вектор

    0003

    ac

    Centripetal/Centrifugal acceleration

    metre/square second

    Vector 

    m

    Mass

    Kilogram (kg)

    Скаляр 

    F

    Сила

    Ньютон (Н)

    W/Fg

    Force due to gravity/Weight

    Newton

    Vector

    Fg/ N

    Normal force

    N

    Vector

    FF

    Сила трения

    N

    Вектор

    µ

    Coefficient of friction

    Unitless

    Scalar

    p

    Momentum

    Kg. m/s

    Vector

    J

    Impulse

    N/S

    Vector

    E

    Энергия

    0002 Joule  (J)

    Scalar

    Kinetic energy

    J

    Scalar

    U

    Potential Energy

    J

    Скалар

    VG

    Гравитационный потенциал

    J/KG

    Скатар

    J/KG

    η

    Efficiency

    Unitless

    Scalar

    P

    Power

    Watt

    Scalar

    α

    Ускорение вращения

    Радиан на секунду в квадрате (Рад/с2)

    Вектор

    ω

    Rotational velocity 

    Rad/s

    Vector

    τ

    Torque

    N/m

    Vector

    L

    Угловой момент

    Килограмм-метр в секунду

    Кг. м2/с

    Вектор

    3

    0026

    ρ

    Density

    Volume 

    Mass density

    Kilogram per cubic meter

    Scalar 

    I

    Moment of inertia

    Kg.m2

    Скаляр

    Физические символы, относящиеся к гидромеханике

    030

    Символы

    Количество/коэффициенты

    S.I. Блок

    Физическая величина (скаляр/вектор)

    9003

    Скаляр

    σ

    Плотность площади

    Килограмм на квадратный метр (кг/м2)

    0003

    FB, B

    Buoyancy

    N

    Vector

    qm

    Mass flow rate

    kg/s

    Scalar

    QV

    Громкость Скорость потока

    M3/S

    Скаляр

    FD, R

    Drag or air resistance

    N

    Vector

    CD

    Drag Coefficient

    Unitless

    Scalar 

    η

    Вязкость

    Паскаль-секунда

    Скаляр

    v

    Кинематическая вязкость

    M2/S

    Скаляр

    σ

    . Номер

    Unitless

    Скаляр

    FR

    Froude Number

    БЕСПЛАТНЫЙ

    0030

    Scalar 

    Ma

    Mach number

    Unitless

    Scalar

    Symbols Related to Solid Mechanics

    030

    Символы

    Количество/Коэффициенты

    Единица СИ

    Физическая величина (скаляр/вектор)

    P

    Pressure 

    Pascal

    Or

    N/m2

    Scalar 

    σ

    Stress

    Pascal

    Scalar

    τ

    Пресс. 0027

    k

    Spring constant

    N/m

    Scalar

    E

    Young’s modulus of elasticity

    Pascal

    Scalar 

    G

    Модуль жесткости при сдвиге

    Паскаль

    Скаляр

    Linear strain

    Unitless 

    Scalar 

    γ

    Shear strain

    Unitless

    Scalar 

    θ

    Volume деформация

    Безразмерная

    Скалярная

    S

    Поверхностное натяжение

    N/m

    Scalar 

    K

    Bulk modulus of compression

    Pascal

    Scalar 

    Physical Quantities Related to Thermal Physics

    10027

    03

    Символы

    Количество/коэффициенты

    Единица измерения 09 S. 0.0.30030

    Physical Quantity (Scalar/Vector)

    k

    Thermal conductivity

    W/m.K

    Scalar 

    P

    Heat flow rate

    Ватт

    Скаляр

    N

    Номер частиц

    БЕСПЛАТНО

    Scalar 

    n

    Amount of substance

    Mole

    Scalar

    L

    Latent heat/specific latent heat

    J/ кг

    Скаляр

    c

    Удельная теплоемкость

    Дж/кг. K

    Скаляр 

    Q

    HEAT

    J

    Скаляр

    B

    70027. Кельвин)

    Скаляр 

    α

    Линейное расширение, коэффициент теплового расширения

    Scalar 

    T

    Temperature 

    Kelvin

    Scalar 

    Physical Symbol Related to Wave and Optics

    Символ

    Количество/Коэффициенты

    Единица S.I

    Физическая величина (скаляр/вектор)

    M

    Magnification

    Untiless 

    Scalar 

    f

    Focal length

    Metre 

    Scalar 

    n

    Показатель преломления

    Безразмерный

    Скалярный

    L

    Level

    Decibel (dB), decineper

    Scalar 

    I

    Intensity

    W/m2

    Scalar

    В, C

    Скорость волны

    м/с

    Скаляр

    λ

    Wavelend0003

    Metre (m)

    Scalar 

    P

    Power of a lens

    Dioptre (D)

    Scalar 

    Physics Symbols Related к электричеству и магнетизму

    003

    Символ

    Количество/коэффициенты

    9. 27 SI

    Physical Quantity (Scalar/Vector)

    Poynting vector, intensity

    W/m2

    Vector 

    η

    Energy Плотность

    J/M3

    Скаляр

    N

    Отчредитель на единицу длины

    1/M

    .0003

    Scalar 

    N

    Number of turns

    Unitless 

    Scalar

    φB

    Magnetic flux 

    Weber

    Вектор

    B

    Магнитное поле

    Тесла

    Вектор

    20030

    FB

    Magnetic force

    N

    Vector 

    σ

    Conductivity

    Siemens/m 

    (S/m)

    Scalar

    G

    Проводятся

    Siemens

    Скаляр

    0002 ρ

    Resistivity

    Ohm-m

    Scalar 

    R, r

    Electric resistance/internal resistance

    Ohm 

    Scalar 

    I

    Электрический ток

    Ампер (A)

    Скаляр

    ϵ

    ϵ

    0030

    Dielectric constant

    Unitless

    Scalar 

    Electromotive Force

    Volt (V)

    Scalar

    C

    Емкость

    Фарад (Ф)

    Скаляр 

    В

    Напряжение, электрический потенциал

    V

    Scalar 

    UE

    Electric potential energy

    J

    Scalar 

    φE

    Electric flux 

    Ньютон-метр в квадрате на кулон (Н/м2. C)

    Вектор

    E

    Электрическое поле

    N/C or V/m

    Vector 

    FE

    Electrostatic force

    N

    Vector 

    λ

    Линейная плотность заряда

    кг/м

    Скаляр

    σ

    Площадь Площадь.0003

    Scalar 

    ρ

    Volume charge density

    kg/m3

    Scalar 

    q, Q

    Electric charge

    Кулон (C)

    Скаляр 

    Символы, используемые в современной физике

    7 2 2 9029

    02 Symbol

    0027

    ψ (r, t), ψ (r) φ (t)

    . J

    Quantity/Coefficients

    S. I Unit

    Physical Quantity (Scalar/Vector)

    D

    Dose/ dose absorbed

    Gray (Gy)

    Scalar

    T1⁄2

    Полурочная жизнь

    Второй

    Вектор

    Вектор

    Волновая функция

    Unitless

    Scalar

    W.

    Scalar

    H

    Effective Dose

    Sievert

    Scalar

    Γ

    Фактор Лоренца/Гамма Лоренца

    Безразмерный

    Скаляр

    Из приведенного выше текста по физике для обозначения величин мы понимаем, что в физике мы используем различные символы или обозначения. Обозначения упрощают представление величин.

    Также было интересно увидеть, что некоторые физические символы очень близки (например, «d» для расстояния), а некоторые не связаны (например, «c» для скорости света или «λ» для длины волны). Кроме того, мы заметили, что конкретный символ связан с более чем одной величиной.

    общая теория относительности — Объемная форма в терминах символа Леви-Чивиты

    спросил

    Изменено 2 года, 3 месяца назад

    Просмотрено 936 раз

    $\begingroup$

    Сейчас я читаю книгу Шона Кэрролла по общей теории относительности, и в какой-то момент он пишет: 9{\ му_п}, \end{уравнение} поскольку и произведение клина, и символ Леви-Чивиты полностью антисимметричны.

    Как мне это заметить? Я изо всех сил пытаюсь понять, как тот факт, что оба объекта полностью антисимметричны, подразумевает приведенное выше уравнение. Означает ли это, что любой полностью антосимметричный тензор может быть записан как сокращение над символом Леви-Чивиты (надеюсь, это правильная терминология; поправьте меня, если я ошибаюсь)?

    • общая теория относительности
    • дифференциальная геометрия
    • тензорное исчисление
    • объем

    $\endgroup$

    $\begingroup$

    Полностью антисимметричный символ размерности $n$ с индексами $n$ имеет только одну независимую компоненту. Например. если $\rho_{a_1…a_n}$ — вполне антисимметричный символ, то $$\rho_{a_1…a_n}=\rho_{12…n}\tilde{\epsilon}_{a_1. ..a_n},$$, где $\tilde\epsilon$ — символ Леви-Чивиты. Это связано с тем, что в размерностях $n$ антисимметричный символ индексов $k$ имеет компоненты $\left(\begin{matrix}n \\ k\end{matrix}\right)$, а $\left(\begin{matrix }n \\ n\end{matrix}\right)=1$.

    Формула нахождения q в физике: А для чего формулы? Q=c*m*t , t=Q/cm и зачем в кДж переводить??

    Основные формулы по физике — ЭЛЕКТРИЧЕСТВО И МАГНЕТИЗМ

    Формулы электричества и магнетизма. Изучение основ электродинамики традиционно начинается с электрического поля в вакууме. Для вычисления силы взаимодействия между двумя точными зарядами и вычисления напряженности электрического поля, созданного точечным зарядом, нужно уметь применять закон Кулона. Для вычисления напряженностей полей, созданных протяженными зарядами (заряженной нитью, плоскостью и т.д.), применяется теорема Гаусса. Для системы электрических зарядов необходимо применять принцип

    При изучении темы «Постоянный ток» необходимо рассмотреть во всех формах законы Ома и Джоуля-Ленца При изучении «Магнетизма» необходимо иметь в виду, что магнитное поле порождается движущимися зарядами и действует на движущиеся заряды. Здесь следует обратить внимание на закон Био-Савара-Лапласа. Особое внимание следует обратить на силу Лоренца и рассмотреть движение заряженной частицы в магнитном поле.

    Электрические и магнитные явления связаны особой формой существования материи — электромагнитным полем. Основой теории электромагнитного поля является теория Максвелла.

    Смотрите также основные формулы оптики

    Таблица основных формул электричества и магнетизма

     Физические законы, формулы, переменные

     Формулы электричество и магнетизм

    Закон Кулона:
    где q1 и q2 — величины точечных зарядов, ԑ1  — электрическая постоянная;
    ε — диэлектрическая проницаемость изотропной среды (для вакуума ε = 1),
    r — расстояние между зарядами.

    Напряженность электрического поля:

    где Ḟ — сила, действующая на заряд q0 , находящийся в данной точке поля.

    Напряженность поля на расстоянии r от источника поля:

    1) точечного заряда

    2) бесконечно длинной заряженной нити с линейной плотностью заряда τ:

    3) равномерно заряженной бесконечной плоскости с поверхностной плотностью заряда σ:

    4) между двумя разноименно заряженными плоскостями

    1)

     

    2)

     

    3)

     

    4)

     

    Потенциал электрического поля:

    где W — потенциальная энергия заряда q0 .

    Потенциал поля точечного заряда на расстоянии r от заряда:

    По принципу суперпозиции полей, напряженность:

    Потенциал:

    где Ēi и ϕi — напряженность и потенциал в данной точке поля, создаваемый i-м зарядом.

    Работа сил электрического поля по перемещению заряда q из точки с потенциалом ϕ1 в точку с потенциалом ϕ2 :

    Связь между напряженностью и потенциалом

    1) для неоднородного поля:

    2) для однородного поля:

    1) 

     

    2) 

     

    Электроемкость уединенного проводника:

    Электроемкость конденсатора:

    где U = ϕ1 — ϕ2 — напряжение.

    Электроемкость плоского конденсатора:

    где S — площадь пластины (одной) конденсатора,

    d — расстояние между пластинами.

    Энергия заряженного конденсатора:

    Сила тока:

    Плотность тока:

    где S — площадь поперечного сечения проводника.

    Сопротивление проводника:

    ρ — удельное сопротивление;

    l — длина проводника;

    S — площадь поперечного сечения.

    Закон Ома

    1) для однородного участка цепи:

    2) в дифференциальной форме:

    3) для участка цепи, содержащего ЭДС:

       где ε — ЭДС источника тока,

       R и r — внешнее и внутреннее сопротивления цепи;

    4) для замкнутой цепи:

    1)

     

    2) 

     

    3) 

     

    4) 

     

    Закон Джоуля-Ленца

     1) для однородного участка цепи постоянного тока:
        где Q — количество тепла, выделяющееся в проводнике с током,
        t — время прохождения тока;

     2) для участка цепи с изменяющимся со временем током:

    1)

    2)

    Мощность тока:

    Связь магнитной индукции и напряженности магнитного поля:

    где B — вектор магнитной индукции,
    μ √ магнитная проницаемость изотропной среды, (для вакуума μ = 1),
    µ0 — магнитная постоянная ,
    H — напряженность магнитного поля.

    Магнитная индукция (индукция магнитного поля):
     1) в центре кругового тока
         где R — радиус кругового тока,

     2) поля бесконечно длинного прямого тока
         где r — кратчайшее расстояние до оси проводника;

     3) поля, созданного отрезком проводника с током
        где ɑ1 и ɑ2 — углы между отрезком проводника и линией, соединяющей концы отрезка и точкой поля;
     4) поля бесконечно длинного соленоида
         где n — число витков на единицу длины соленоида.

    1)

     

    2) 

     

    3) 

     

    4) 

     

    Сила Лоренца:

    по модулю
    где F — сила, действующая на заряд, движущийся в магнитном поле,
    v — скорость заряда q,
    α — угол между векторами v и B.

    Поток вектора магнитной индукции (магнитный поток через площадку S):
     1) для однородного магнитного поля ,
        где α — угол между вектором B и нормалью к площадке,
     2) для неоднородного поля

    1)

     

    2)

     

    Потокосцепление (полный поток):
    где N — число витков катушки.

    Закон Фарадея-Ленца:
    где ԑ— ЭДС индукции.

    ЭДС самоиндукции:
    где L — индуктивность контура.

    Индуктивность соленоида:

    где n — число витков на единицу длины соленоида,
    V — объем соленоида.


    Энергия магнитного поля:

    Заряд, протекающий по замкнутому контуру при изменении магнитного потока через контур:

    где ∆Ф = Ф2 – Ф1 — изменение магнитного потока, R — сопротивление контура.

    Работа по перемещению замкнутого контура с током I в магнитном поле:

    Количество теплоты: нагревание, охлаждение, плавление, кристаллизация, парообразование, конденсация, горение. Термодинамическая система

    Физика->Термодинамика->количество теплоты->

    Тестирование онлайн

    • Количество теплоты. Основные понятия

    • Количество теплоты

    Термодинамика

    Раздел молекулярной физики, который изучает передачу энергии, закономерности превращения одних видов энергии в другие. В отличие от молекулярно-кинетической теории, в термодинамике не учитывается внутреннее строение веществ и микропараметры.

    Термодинамическая система

    Это совокупность тел, которые обмениваются энергией (в форме работы или теплоты) друг с другом или с окружающей средой. Например, вода в чайнике остывает, происходит обмен теплотой воды с чайником и чайника с окружающей средой. Цилиндр с газом под поршнем: поршень выполняет работу, в результате чего, газ получает энергию, и изменяются его макропараметры.

    Количество теплоты

    Это энергия, которую получает или отдает система в процессе теплообмена. Обозначается символом Q, измеряется, как любая энергия, в Джоулях.

    В результате различных процессов теплообмена энергия, которая передается, определяется по-своему.

    Нагревание и охлаждение

    Этот процесс характеризуется изменением температуры системы. Количество теплоты определяется по формуле

    Удельная теплоемкость вещества с измеряется количеством теплоты, которое необходимо для нагревания единицы массы данного вещества на 1К. Для нагревания 1кг стекла или 1кг воды требуется различное количество энергии. Удельная теплоемкость — известная, уже вычисленная для всех веществ величина, значение смотреть в физических таблицах.

    Теплоемкость вещества С — это количество теплоты, которое необходимо для нагревания тела без учета его массы на 1К.

    Плавление и кристаллизация

    Плавление — переход вещества из твердого состояния в жидкое. Обратный переход называется кристаллизацией.

    Энергия, которая тратится на разрушение кристаллической решетки вещества, определяется по формуле

    Удельная теплота плавления известная для каждого вещества величина, значение смотреть в физических таблицах.

    Парообразование (испарение или кипение) и конденсация

    Парообразование — это переход вещества из жидкого (твердого) состояния в газообразное. Обратный процесс называется конденсацией.

    Удельная теплота парообразования известная для каждого вещества величина, значение смотреть в физических таблицах.

    Горение

    Количество теплоты, которое выделяется при сгорании вещества

    Удельная теплота сгорания известная для каждого вещества величина, значение смотреть в физических таблицах.

    Для замкнутой и адиабатически изолированной системы тел выполняется уравнение теплового баланса. Алгебраическая сумма количеств теплоты, отданных и полученных всеми телами, участвующим в теплообмене, равна нулю:

    Q1+Q2+…+Qn=0

    Электрический заряд — веб-формулы

    Электрический заряд определяется как:

    Q = I t

    Соответствующие единицы СИ:
    кулон (Кл) = ампер (А) ∙ секунда (с)

    Где I — электрический ток, а t — время (длительность).

    Блок

    • Заряд системы можно измерить, сравнив его с зарядом стандартного кузова.
    • Единица заряда в системе СИ — Кулон, записывается как C.
    • 1 Кулон — это заряд, протекающий по проводу за 1 секунду, если сила тока в нем равна 1А.
    • Заряд электрона равен -1,602 * 10 -19 Кл, а заряд протона положителен от этого значения.
    • Двумя важными свойствами заряда являются квантование и сохранение .

    (a)   Квантование заряда

    (i)                 Электрический заряд может существовать только как целое кратное заряду электрона (-e), т.е.

    q = ± ne , , где n — целое число.

     

    (ii)              Возможные значения электрического заряда: q = ± 1e; ± 2е; ± 3e

    (iii)            Заряд меньше, чем заряд электрона ( т.е. e = 1,6 * 10 -19 Кл) невозможен.

    (b)   Сохранение заряда

    (i)                В изолированной системе общий электрический заряд всегда остается постоянным.

    (ii)              Общий заряд тела равен алгебраической сумме всех имеющихся на нем зарядов. Каждый атом электрически нейтрален, так как содержит столько электронов, сколько в нем протонов.

    (iii)            Когда мы потираем стеклянную палочку о кусок шелка, положительный заряд, приобретаемый стеклянной палочкой, равен отрицательному заряду, приобретаемому кусочком шелка. Таким образом, заряды производятся равными и непохожими парами.

    Пример (1) : Какова возможная величина электрического заряда?

     

    (а)   1 X 1,6 X 10 -19 C

    (б)   2,4 X 1,6 X 10 -19 C

    (в)    -8 X 1,6 X 10 -19 C

    (г)   1 X 1,8 X 10 -19 C

     

    Решение: (а)

    Как мы знаем, электрический заряд может существовать только как целое кратное заряду электрона (-е), т.е.

     

    q = ± ne , где n — целое число. Итак, q = ± 1 х 1,6 х 10 -19 С

     

    Пример (2) : Если n = 2, какова будет величина электрического заряда? (Данный e = 1,6 X 10 -19 C)

    (а)   ±0,8 X 10 -19 С

    (б)   ±3,2 X 10 -19 С

    (в)    ±4,3 X 10 -19 С

    (г)   ±6,3 X 10 -19 С

     

    Решение: ( b )

    Мы знаем, что

    q = ± пе

        = 2 х 1,6 х 10 -19 С

        = ±3,2 X 10 -19   С

    Следовательно, вариант (б) правильный.

     

    Пример (3): Возможен ли заряд меньше заряда электрона (т. е. e = 1,6 X 10 -19 Кл)?

    (а)    Да      (б) Нет

     

    Решение: (b) Как мы знаем

    q = ± ne , где n — целое число, т. е. n = 1, 2, 3,…

     

    Пример 4): Каков общий заряд всех протонов в 1,00 кг углерода?

    (а)   4,82 X 10 7 С

    (б)   3,96 X 10 7 С

    (с)    4,82 X 10 9 С

    (г)   3,96 X 10 12 С

     

    Решение: (a) Мы можем найти количество кулонов положительного заряда в 1,00 кг углерода из Q = 6n c e , где n c — число атомов в 1,00 кг углерода, а коэффициент 6 учитывает присутствие 6 протонов в каждом атоме. Мы можем найти число атомов в 1,00 кг углерода, составив пропорцию, связывающую число Авогадро N A , массу углерода и молекулярную массу углерода с n c .

    Подстановка числовых значений и оценка Q:

    Пример 5): Определить электрический ток в электрической цепи, где общий электрический заряд равен 6 Кл за 5 секунд.

    Изолирующий I от Q = I t
    I = Q / t = 16 / = 16 /

    Формула электрического заряда — GeeksforGeeks

    Когда материя находится в электрическом или магнитном поле, она приобретает электрический заряд, что заставляет ее испытывать силу. Текущий электрический заряд создает магнитное поле, которое связано с электрическим полем. Электромагнитное поле состоит из комбинации электрического и магнитного полей. Электромагнитная сила, являющаяся основой физики, создается при взаимодействии зарядов. Давайте подробнее рассмотрим концепцию электрического заряда,

    Электрический заряд

    Свойство субатомных частиц, которое позволяет им испытывать силу при помещении в электрическое или магнитное поле, называется электрическим зарядом.

    Скалярная величина, электрический заряд. Величина, называемая вектором, должна удовлетворять законам сложения векторов, таким как закон сложения векторов треугольника и закон сложения векторов параллелограмма, в дополнение к величине и направлению; только тогда сумма называется векторной величиной. В случае электрического тока результирующий ток представляет собой алгебраическую сумму, а не векторную сумму, когда два тока встречаются на стыке. В результате электрический ток, хотя и имеет величину и направление, является скалярной величиной. Электрический заряд обозначается Q.

    Единица электрического заряда в системе СИ: Кулон и Другие единицы измерения: Фарадей, Ампер-час.

    Положительные и отрицательные электрические заряды переносятся протонами и электронами соответственно.

    • Положительно заряженные протоны
    • Отрицательно заряженные электроны
    • Нейтрон имеет нулевой заряд

    Формула электрического заряда

    Q = I × t

    Где,

    • Q = электрический заряд,
    • I = электрический ток,
    • t = время.

    Примеры вопросов

    Вопрос 1. По какой причине электрический заряд является скалярной величиной?

    Ответ :

    Результирующий ток двух токов, встречающихся на стыке, представляет собой алгебраическую сумму, а не векторную сумму. Таким образом, скалярная величина представляет собой электрический ток. Это свойство электрического заряда известно как KCL, также известное как закон тока Кирхгофа.

    Вопрос 2: Когда возникает отрицательный и положительный электрический заряд?

    Ответ :

    Считается, что заряд отрицательный, если в веществе больше электронов, чем протонов, и считается, что он имеет положительный заряд, если в веществе больше протонов, чем электронов.

    Вопрос 3: Цепь с током 150 мА работает в течение 2 минут. рассчитать количество заряда, протекающего по цепи.

    Решение:

    Дано: I = 150 мА = 150 × 10 -3 A, T = 2 мин = 2 × 60 = 120S

    С тех пор

    Q = I × T

    ∴. Q = 150 × 10 -3 × 120

    ∴ Q = 18 C

    Вопрос 4. Когда проводник с током подключен к внешнему источнику питания в течение 20 секунд, всего 6 × 10 46 электронов текут через него. Определить значение тока в проводнике.

    Решение:

    Дано: n = 6 × 10 46 Электроны, T = 20S, E = 1,6 × 10 -19 C

    С

    Q = I × T

    ∴ ∴ ∴ ∴ ∴ ∴

    . I = Q/T

    по токовой формуле,

    Q = NE

    ∴ I = NE/T

    ∴ I = 6 × 10 46 × 1,6 × 10 -19 /20

    ∴ I = 4,8 × 10 26 А

    Вопрос 5: По проводнику течет ток силой 0,6 А.

    © 2015 - 2019 Муниципальное казённое общеобразовательное учреждение «Таловская средняя школа»

    Карта сайта